eureka 2008

140
CONTEÚDO AOS LEITORES 2 XVIII OLIMPÍADA DE MATEMÁTICA DO CONE SUL 3 Enunciados e resultado brasileiro XIX OLIMPÍADA DE MATEMÁTICA DO CONE SUL 11 Enunciados e resultado brasileiro ARTIGOS JOGOS E FEIJOADA NO SÃO PAULO´S 13 Emanuel Carneiro SUBSTITUIÇÕES ENVOLVENDO NÚMEROS COMPLEXOS 17 Diego Veloso Uchôa INTEGRAIS DISCRETAS 25 Eduardo Poço PRODUTOS NOTÁVEIS 32 Onofre Campos OLIMPÍADAS AO REDOR DO MUNDO 38 COMO É QUE FAZ 48 SOLUÇÕES DE PROBLEMAS PROPOSTOS 50 PROBLEMAS PROPOSTOS 58 AGENDA OLÍMPICA 61 COORDENADORES REGIONAIS 62

Upload: demi-de

Post on 08-Mar-2016

356 views

Category:

Documents


12 download

DESCRIPTION

PROBLEMAS PROPOSTOS 58 JOGOS E FEIJOADA NO SÃO PAULO´S 13 Emanuel Carneiro COORDENADORES REGIONAIS 62 OLIMPÍADAS AO REDOR DO MUNDO 38 SUBSTITUIÇÕES ENVOLVENDO NÚMEROS COMPLEXOS 17 Diego Veloso Uchôa SOLUÇÕES DE PROBLEMAS PROPOSTOS 50 CONTEÚDO XIX OLIMPÍADA DE MATEMÁTICA DO CONE SUL 11 Enunciados e resultado brasileiro XVIII OLIMPÍADA DE MATEMÁTICA DO CONE SUL 3 Enunciados e resultado brasileiro ARTIGOS

TRANSCRIPT

Page 1: Eureka 2008

CONTEÚDO AOS LEITORES 2 XVIII OLIMPÍADA DE MATEMÁTICA DO CONE SUL 3 Enunciados e resultado brasileiro XIX OLIMPÍADA DE MATEMÁTICA DO CONE SUL 11 Enunciados e resultado brasileiro ARTIGOS JOGOS E FEIJOADA NO SÃO PAULO´S 13 Emanuel Carneiro SUBSTITUIÇÕES ENVOLVENDO NÚMEROS COMPLEXOS 17 Diego Veloso Uchôa INTEGRAIS DISCRETAS 25 Eduardo Poço PRODUTOS NOTÁVEIS 32 Onofre Campos OLIMPÍADAS AO REDOR DO MUNDO 38 COMO É QUE FAZ 48 SOLUÇÕES DE PROBLEMAS PROPOSTOS 50 PROBLEMAS PROPOSTOS 58 AGENDA OLÍMPICA 61 COORDENADORES REGIONAIS 62

Page 2: Eureka 2008

Sociedade Brasileira de Matemática

EUREKA! N°27, 2008

2

AOS LEITORES

É com grande alegria que comemoramos em 2008 os 10 anos da Revista EUREKA! e transmitimos aos leitores a nossa satisfação pela acolhida recebida neste período. Durante estes 10 anos de existência temos procurado atender ao leitor mais exigente, apresentando uma publicação específica que além de fornecer material atualizado e de alto nível acadêmico, tem tornado o estudo da matemática olímpica muito mais interessante e acessível a professores e jovens olímpicos de todo o Brasil.

Neste número especial da revista apresentamos quatro artigos, cujos

autores são todos ex-olímpicos de grande destaque, além de um bom número de novos problemas propostos por nossos leitores, que estão cada vez mais inspirados. Agradecemos também a valiosa ajuda dos alunos que trabalharam na revisão deste número da Eureka!: Álvaro Lopes Pedroso, Ana Luísa de Almeida Losnak, Custódio Moreira Brasileiro Silva, Elder Massahiro Yoshida, Guilherme Phillippe Figueiredo Hanon Guy Lima Rossi, Henrique Pondé de Oliveira Pinto, Illan Feiman Halpern, Marco Antonio Lopes Pedroso, Rafael Horimoto de Freitas, Renan Henrique Finder, Talita Alessandra da Silva, Thiago Saksanian Hallak e Thiago da Silva Pinheiro, e particularmente ao Prof. Carlos Yuzo Shine, que coordenou a revisão e que foi responsável pela seção “Como é que faz” deste número. Continuaremos contando com o entusiasmo e a colaboração dos nossos leitores para que a EUREKA! continue sendo um instrumento útil à formação matemática e à preparação olímpica do nosso público. Esperamos que gostem deste número. Divirtam-se!

Os editores

Page 3: Eureka 2008

Sociedade Brasileira de Matemática

EUREKA! N°27, 2008

3

XVIII OLIMPÍADA DE MATEMÁTICA DO CONE SUL Enunciados e Resultado Brasileiro

A XVIII Olimpíada de Matemática do Cone Sul foi realizada na cidade de Atlântida, Uruguai no mês de junho de 2007. A equipe brasileira foi liderada pelos professores Yuri Gomes Lima e Samuel Barbosa Feitosa, ambos da cidade de Fortaleza – CE. RESULTADOS DA EQUIPE BRASILEIRA BRA1 Renan Henrique Finder Medalha de Ouro BRA2 Marcelo Tadeu de Sá Oliveira Sales Medalha de Prata BRA3 Grazielly Muniz da Cunha Medalha de Prata BRA4 Thiago Ribeiro Ramos Medalha de Prata

PRIMEIRO DIA

PROBLEMA 1 Achar todos os pares de inteiros (x, y) que satisfazem

3 22 .x y x y xy xy+ + = +

SOLUÇÃO DE MARCELO TADEU DE SÁ OLIVEIRA SALES (SALVADOR – BA) De 3 22x y x y xy xy+ + = + temos:

3 2 2 22 ( 1 2 ) |x y x xy xy y x x y y y y x y+ − − = − ⇒ + − − = − ⇒ 2 3 32 ( 2 1 ) |xy xy y x y x y x xy x x y x+ − − = ⇒ + − − = ⇒

Então |x y e |y x com exceção de x = 0 ou y = 0. Nesses dois casos temos que ambos têm que ser 0. Assim (x, y) = (0, 0) é a nossa primeira solução. Se |x y então x y≤ (eu já desconsiderei x = 0 e y = 0) e se |y x então ,y x≤ daí

.x y= Assim temos dois casos: Primeiro caso: x = y Substituindo temos 4 2 32 2 .x x x x+ = + Como 0x ≠ e 0y ≠ então podemos simplificar. Assim, 3 22 2x x x+ = + e daí | 2x , então { 2, 1,1,2}.x∈ − − Desses valores, o único que não admite solução é x = – 2 então para esse caso ( , ) ( 1, 1);(1,1);(2,2).x y = − −

Page 4: Eureka 2008

Sociedade Brasileira de Matemática

EUREKA! N°27, 2008

4

Segundo caso: x = – y. Substituindo temos 4 2 32 .x x x− = − +

Como 0x ≠ temos 4 2 3 22 1 2x x x x x− = − + ⇔ − = − + e daí 2 82

x − ±= que não

é inteiro, então não há solução para esse caso. Assim as soluções são ( , ) ( 1, 1);(0,0);(1,1)x y = − − e (2, 2). PROBLEMA 2 Considere 100 inteiros positivos tais que sua soma é igual ao seu produto. Determinar a quantidade mínima de números 1 que podem existir entre os 100 inteiros. SOLUÇÃO DE RENAN HENRIQUE FINDER (JOINVILLE – SC) Seja K o número de 1´s que aparecem. Sejam 1 2 100, ...a a a os números, com

1 2 ... 1.Ka a a= = = =

1 2 100 1 2 100... ...a a a a a a+ + + =

1 100 1 2 100... ...K K KK a a a a a+ + ++ + + = Vamos minimizar 1 2 100 1 2 100... ... .K K K Ka a a a a a+ + + +− − − − Para isso, suponha

2.ja ≥

Note que 1 100 1 100 1 1 100 1... ... ... ... 2 ... ... 2 ...K j K j K j j Ka a a a a a a a a a a+ + − + +− − − − − ≥ − − −

1 1 100 1 100 1 1 1 100... ... ... 2 ... ... 2j j K j j K j ja a a a a a a a a a a− + + + − +− − − − ⇔ − ≥ − ⇔

1 1 1 100... ... ( 2) 2,K j j j ja a a a a a+ − + − ≥ − o que ocorre de fato. Então, a diferença é

mínima quando 1 2 100... 2.K Ka a a+ += = = = Logo, 100 100

1 2 100 1 100... ... 2 2 (100 ) 0 2 200K KK K KK a a a a a K K− −+ + += − − − ≥ − ⋅ − ⇒ ≥ − + ⇒

1002 200 .K K− ≤ − Se 93 200 107K K≤ − ≤ e 100 72 2 128,K− ≥ = o que obriga 94.K ≥ Note que há um exemplo para K = 95:

1 2 45... 1a a a= = = =

96 95 2a a= =

98 99 100 3a a a= = = A soma é 195 2 2 3 3 95 4 9 108+ ⋅ + ⋅ = + + = e o produto é 2 32 3 108.⋅ = Resta o caso K = 94, isto é, o caso 94 .a b c d e f abcdef+ + + + + + = Supondo , 3,a b ≥ minimizemos .abcdef a b c d e f− − − − − − Temos

Page 5: Eureka 2008

Sociedade Brasileira de Matemática

EUREKA! N°27, 2008

5

3 3 ( 3) 3,abcdef a b c d e f bcdef b c d e f a bcdef a− − − − − − ≥ − − − − − − ⇔ − ≥ −o que ocorre. Além disso,

2 2 ( 2) 2,abcdef a b c d e f abcde a b c d e abcde f f− − − − − − ≥ − − − − − − ⇔ − ≥ −o que ocorre, pois 1,f ≠ logo 2.f ≥ Concluímos que a expressão é mínima se a = 3 e f = 2. Analogamente, ela é mínima quando b = 3 e c = d = e = 2. Então,

2 43 2 4 2 3 2 144 14 130 94.abcdef a b c d e f− − − − − − ≥ − ⋅ − ⋅ = − = > Então, é impossível 94abcdef a b c d e f− − − − − − = se duas das variáveis forem 3≥ . Já se só uma for 3≥ (digamos que a, teremos b = c = d = e = f = 2, logo

52 10 94 31 104,a a a− − = ⇒ = absurdo, pois 31 | 104./ Se todas as variáveis forem iguais a 2, obtemos também 31 104 31 2 104,a = ⇒ ⋅ = absurdo. Então K = 95 é o máximo que podemos obter. PROBLEMA 3 Seja ABC um triângulo com todos os seus ângulos agudos, de alturas AD, BE e CF (com D em BC, E em AC e F em AB). Seja M o ponto médio do segmento BC. A circunferência circunscrita ao triângulo AEF corta a reta AM em A e X. A reta AM corta a reta CF em Y. Seja Z o ponto de encontro entre as retas AD e BX. Demonstrar que as retas YZ e BC são paralelas. SOLUÇÃO DA BANCA Observemos que AFHE é inscritível, pois 90 .AEH AFH∠ =∠ = ° Daí que 90 .AXH∠ = ° Seja A´ um ponto sobre a semireta AM tal que AM = MA´. O quadrilátero ABA´C é um paralelogramo, onde

´ ´A BH A BC CBH∠ =∠ +∠ (90 )ACB ACB= ∠ + ° −∠

90= ° ´ ,A XH= ∠

Ou seja que o quadrilátero BHXA´ é inscritível. Alem disso BHCA´ também é inscritível já que

´ (180 ) 180 .BHC BA C BAC BAC∠ +∠ = ° −∠ +∠ = ° Desta forma os pontos B, H, X, C são concíclicos, onde

Page 6: Eureka 2008

Sociedade Brasileira de Matemática

EUREKA! N°27, 2008

6

´XBM XBC XA C BAM∠ =∠ =∠ =∠ (1) Seja .T AB XH= ∩ Notemos que H também é o ortocentro do triângulo ATY, uma vez que TY AY⊥ e .YF AT⊥ Daí que ,AH TY⊥ então // .TY BC Se provamos que ,Z TY∈ o problema estará terminado. Seja então ´ .Z AD TY= ∩ Vamos mostrar que .́Z Z= Agora, ´ 90HZ Y HXY∠ =∠ = ° então HXYZ´ é inscritível, e portanto,

´XZ Y XHY FAX BAM∠ =∠ = ∠ −∠ (2) Das relações (1) e (2) segue-se que ´ ,XZ Y XBM∠ =∠ ou seja, que os pontos B, Z´ e X são colineares. Porém então ´Z AD BX Z∈ ∩ = e assim ´Z Z= , como queríamos.

B

A

E

F H X

Y

D M

Z

C

PROBLEMA 4 Considere um tabuleiro 2007 2007.× São pintadas algumas casas do tabuleiro. Dizemos que o tabuleiro é charrua se nenhuma linha está totalmente pintada e nenhuma coluna está totalmente pintada. a) Qual é o número máximo k de casas pintadas que um tabuleiro charrua pode ter? b) Para tal número k, calcular o número de tabuleiros charruas distintos que existem.

Page 7: Eureka 2008

Sociedade Brasileira de Matemática

EUREKA! N°27, 2008

7

SOLUÇÃO DE GRAZIELLY MUNIZ DA CUNHA (FORTALEZA – CE) a) Note que todas as colunas têm que ter no máximo 2006 casas pintadas. Como são 2007 colunas então o número de casas pintadas é no máximo 2007 2006,× número que é atingido pintando todas as casas, exceto uma diagonal, como na figura abaixo

2007

2007

b) como para k no máximo iremos pintar 2006 casas em cada coluna, então temos que escolher qual casa ficará sem ser pintada. E note que não podemos ter duas casas sem serem pintadas em uma mesma linha, pois se não terá uma linha que ficará toda preenchida. Logo para a primeira coluna poderemos escolher qualquer uma das 2007 casas para não ser pintada, na segunda coluna podemos escolher qualquer uma de 2006 casas, pois não podemos escolher uma casa que esteja na mesma linha qua a que foi escolhida na primeira coluna, na terceira coluna temos 2005 escolhas, na quarta 2004 escolhas e assim sucessivamente, logo são 2007 2006 2005... 2007!× × = maneiras de escolher, logo são 2007! tabuleiros charruas distintos, com o número k. PROBLEMA 5 Seja ABCDE um pentágono convexo que satisfaz as seguintes condições:

• Existe uma circunferência Γ tangente a cada um de seus lados. • As medidas de todos os seus lados são números inteiros. • Ao menos um dos lados do pentágono mede 1. • O lado AB mede 2.

Seja P o ponto de tangência de Γ com o lado AB. a) Determinar as medidas dos segmentos AP e BP.

Page 8: Eureka 2008

Sociedade Brasileira de Matemática

EUREKA! N°27, 2008

8

b) Dar um exemplo de um pentágono que satisfaz as condições estabelecidas. SOLUÇÃO DE RENAN HENRIQUE FINDER (JOINVILLE – SC) a) Sejam Q, R, S e T os pontos de tangência de Γ em BC, CD, DE e EA, respectivamente, como a seguir:

E

D

C B Q

R P

S T

A

Seja AP = x. Temos: AB = 2 ⇒ BP = 2 – x QB = BP ⇒ QB = 2 – x QC = BC – QB = BC – 2 + x CR = QC ⇒ CR = BC – 2 + x DR = CD – CR = CD – BC + 2 – x DS = DR ⇒ DS = CD – BC + 2 – x ES = DE – DS = DE – C D + BC – 2 + x ET = ES ⇒ ET = DE – CD + BC – 2 + x AE = AT + TE = x + DE – CD + BC – 2 + x ⇒ 2x = AE – DE + CD – BC *2 ++ ∈ Como 2 2 4,x AB x< = ⇒ < pode-se ter 2 3,2 2x x= = ou 2 1.x = O item b)

mostra uma configuração para 32

x = e o caso 12

x = é obviamente análogo

(troque A por B e C por E). Resta ver o que acontece se x = 1. Temos 1 1.AT AE= ⇒ > Então 2,AE ≥ porque AE∈ . Como 1,ET AE= − vale que e 1.ET ET∈ ≥ Assim, 1,SE ≥ pois .SE TE= Desse modo, 2.DE SE DE> ⇒ ≥ Como DS DE SE= − tem-se 1,DS ≥ mas então 1 1.DR DC≥ ⇒ > Logo 2,DC ≥ uma vez que .DC∈ Mas então 1,CR ≥ já que .CR DC DR DC DS= − = − ∈ E também 1CQ ≥ e ,CQ∈ já que .RC QC= Deste modo, 1.BC CQ BC> ⇒ > E

1.AB AP> = Então todos os lados são maiores que 1: absurdo.

Page 9: Eureka 2008

Sociedade Brasileira de Matemática

EUREKA! N°27, 2008

9

Obs. No desenho do item b), de fato 3 ,2

AP = pois 1 312 2 2wAP x= + = + = (o

ponto P bissecta o segmento de medida w pois esse segmento é o lado de um hexágono regular). Por outro lado, na verdade já provamos que só podíamos ter

32

AP = ou 1 .2

AP = Isso mostra que 32

AP = e 12

BP = é uma possibilidade (e

que, analogamente, 32

BP = e 12

AP = também é).

b) Tome um hexágono regular de lado 1, seu incírculo Γ , dois de seus lados não opostos e não adjacentes e os prolongue. Seja A a intersecção obtida. O pentágono ABCDE é o fecho convexo da união dos pontos do hexágono e do ponto A, como a seguir:

A

C D

B E

P w

x

z

y

120° 120°

Note que o triângulo de lados x e y, com vértice em A, tem dois ângulos de 180 120 60 ,° − ° = ° logo é eqüilátero, e de lado 1 (seu terceiro lado é lado do hexágono L )! Como w e z são lados do hexágono, w = x = y = z = 1. Então:

• ABCDE é circunscrível • BC = CD = DE = 1 • AB = EA = w + x = y + z = 1 + 1 = 2

Todos os lados são inteiros, como queríamos. PROBLEMA 6 Demonstrar que, para cada inteiro positivo n, existe um inteiro positivo k tal que a representação decimal de cada um dos números k, 2k,..., nk contém todos os dígitos 0, 1, 2, 3, 4, 5, 6, 7, 8, 9.

Page 10: Eureka 2008

Sociedade Brasileira de Matemática

EUREKA! N°27, 2008

10

SOLUÇÃO DE MARCELO TADEU DE SÁ OLIVEIRA SALES (SALVADOR – BA) Lema: Para todo ,n existe *k +∈ tal que nk contém todos os dígitos 0, 1, 2, ..., 9, onde * .n +∈ Demonstração: Ao fatorarmos n temos que n é da forma 2 5a b q⋅ ⋅ onde q é o produto dos outros fatores primos de n. Seja c = máx (a, b). Vou mostrar que para 10cn q= ⋅ o lema é válido. Temos que 2 5a b q⋅ ⋅ é divisor de 10 ,c q⋅ e por Bézout existe um x tal que 110 10 (mod 10 )c c cqx +⋅ ≡ ⋅ , pois

( ,10) 1mdc q = Assim, se multiplicarmos 10c qx⋅ por 2, 3, ..., 9 ele dará restos 2 10 ,...,9 10c c⋅ ⋅ , ou seja, aparecerem os dígitos que nós quisermos na base decimal. Considere

2 810 ( 2 10 3 10 ... 9 10 )c p p pqx qx qx qx+ ⋅ + ⋅ + + ⋅ onde x é o número tal que 110 10 (mod 10 )c c cqx +⋅ ≡ ⋅ e p é um inteiro tal que 10 9 10 .p c qx> ⋅ ⋅

Então k 2 810( 2 10 3 10 ... 9 10 )p p px x x x= + ⋅ ⋅ + ⋅ ⋅ + + ⋅ ⋅ satisfaz as nossas condições pois ao multiplicarmos k por 10c q⋅ temos o seguinte: Em 10c q x⋅ ⋅ vai aparecer um dígito 1 porque 110 10 (mod10 ).c c cq x +⋅ ⋅ ≡ Em 10 2c p q x+ ⋅ ⋅ vai aparecer um dígito 2. Em 810 9c p q x+ ⋅ ⋅ vai aparecer um dígito 9 e eu multipliquei tudo por 10 para aparecer o 0. Assim k é o que queríamos. Agora vou terminar o problema por indução Casos iniciais 1n→ = e 1234567890k =

n = 2 e 1234567890617283945ou 6172839450k =

Passo indutivo → suponha que até n é verdadeira então existe um k tal que k, 2k, 3k,...,nk têm todos os dígitos 0,1,...,9. Observe que se multiplicarmos k por 10 para *

+∈ continua sendo verdade para ,2 , caso, .k k nk Pelo lema, temos que existe um r tal que (n + 1)r tem todos os dígitos 0,1,...,9. Assim, fazendo 10s k r= + , onde é um inteiro tal que 10 ( 1) ,n r> + esse s satisfaz as condições do enunciado para 1 até (n + 1) pois de 1 até n teremos que

,2 ,...,s s ns terão todos os dígitos porque 10 ,...,10k nk têm (e porque é grande o suficiente para que 10 ).nr < E para n + 1 temos que ( 1) 10n r+ < terá todos os dígitos. Assim s satisfaz as condições. Portanto para todo n inteiro positivo existe tal inteiro k.

Page 11: Eureka 2008

Sociedade Brasileira de Matemática

EUREKA! N°27, 2008

11

XIX OLIMPÍADA DE MATEMÁTICA DO CONE SUL Enunciados e Resultado Brasileiro

A XIX Olimpíada de Matemática do Cone Sul foi realizada na cidade de Temuco, Chile no mês de maio de 2008. A equipe brasileira foi liderada pelos professores Cícero Thiago Magalhães e Bruno Holanda, ambos da cidade de Fortaleza – CE. RESULTADOS DA EQUIPE BRASILEIRA BRA1 Gustavo Lisbôa Empinotti Medalha de Bronze BRA2 Marcelo Tadeu de Sá Oliveira Sales Medalha de Prata BRA3 Matheus Araújo Marins Medalha de Bronze BRA4 Matheus Secco Torres da Silva Medalha de Prata

PRIMEIRO DIA PROBLEMA 1 Defina I(n) como o resultado de inverter os números de um algarismo. Por exemplo, (123) 321,I = etc. Calcule todos os inteiros 1 10000n≤ ≤ tais que

( ) .2nI n =

PROBLEMA 2 Seja ABC um triângulo, P um ponto em seu interior e X, Y e Z pontos em BC, AC e AB respectivamente tais que .PZB PXC PYA∠ =∠ =∠ Considere os pontos U, W e V sobre BC, AC e AB (ou seus prolongamentos, se necessário) tais que PV = 2PY; PU = 2PX e PW = 2PZ. Sabendo que a área de XYZ é 1, calcule a área de UVW. PROBLEMA 3 Dois amigos A e B devem resolver a seguinte adivinha: cada um deles recebe um número do conjunto {1, 2, ..., 250} mas não vê o número que o outro recebeu. O objetivo é que cada amigo descubra o número do outro. O procedimento que devem seguir é anunciar, por turnos, números inteiros positivos não necessariamente distintos: primeiro A diz um número, em seguida B diz um número, depois novamente A, etc., de modo que a soma de todos os números

Page 12: Eureka 2008

Sociedade Brasileira de Matemática

EUREKA! N°27, 2008

12

anunciados seja 20. Demonstrar que existe uma estratégia de modo que, através de um acordo prévio A e B possam atingir o objetivo, sem importar quais números cada um receba no começo da adivinha. SEGUNDO DIA PROBLEMA 4 Qual é o maior número de casas que se pode colorir num tabuleiro 7 × 7 de maneira que todo subtabuleiro 2 × 2 tenha no máximo 2 casas coloridas? PROBLEMA 5 Seja ABC um triângulo isósceles de base AB. Uma semicircunferência C com centro no segmento AB e tangente aos lados iguais AC e BC. Considera-se uma reta tangente a C que corta os segmentos AC e BC em D e E, respectivamente. Suponha que as retas perpendiculares a AC e BC, traçadas respectivamente por D e E, se cortam em P interior ao triângulo ABC. Seja Q o pé da perpendicular à

reta AB que passa por P. Demonstrar que 1 .2

PQ ABCP AC

= ⋅

PROBLEMA 6 Dizemos que um número é capicua se ao inverter a ordem de seus algarismos obtivermos o mesmo número. Achar todos os números que tem pelo menos um múltiplo não-nulo que seja capicua.

Page 13: Eureka 2008

Sociedade Brasileira de Matemática

EUREKA! N°27, 2008

13

JOGOS E FEIJOADA NO SÃO PAULO´S Emanuel A. S. Carneiro

♦ Nível Iniciante

Bem próximo ao Robert Lee Moore hall, sede do Departamento de Matemática da Universidade do Texas em Austin, fica o celebrado restaurante brasileiro São Paulo´s. Comida muito boa (definitivamente a melhor feijoada da cidade), além do velho e bom guaraná Antártica são apenas alguns dos fatores que nos levam (a comunidade brasileira aqui em Austin) a almoçar regularmente no São Paulo´s.

Certo dia eu estava a almoçar com dois professores do departamento e algo me fez lembrar dos meus tempos de olimpíada de Matemática. Fernando Rodriguez-Villegas, argentino, professor na área de teoria dos números e Tamás Hausel, húngaro, que trabalha nas áreas de geometria algébrica e topologia. Enquanto saboreávamos as nossas feijoadas (eu e o Tamás, o Fernando no bobó de camarão), conversávamos sobre jogos matemáticos. Dr. Rodriguez-Villegas, um matemático extraordinário e super simpático, que além de fazer pesquisa do mais alto nível em teoria dos números é bastante interessado em jogos e puzzles matemáticos, nos explicava tópicos do curso que estava a ensinar nesse semestre (Math, Puzzles and Computers) além de outras idéias de jogos que ele próprio havia inventado.

Entre as torres de Hanói, Nim, Resta um, e coisas do tipo escritas em guardanapos do São Paulo´s, Tamás lembrou-se de algo e me perguntou: – Emanuel, você conhece o jogo do “15 out of 3” (15 de 3)? Respondi que não. Ele então me deu a formulação do jogo: – (O jogo 15 out of 3) os números de 1 a 9 estão sobre a mesa. Dois jogadores alternadamente escolhem números para si (sem repetição) e ganha quem primeiro completar 15 somando três de seus números. (sugiro agora que os leitores joguem um pouquinho antes de prosseguir e “desvendar” o mistério). Pensei comigo mesmo: “Hummm... isso não me parece estranho...”. Eu disse: – Bem, minha intuição me leva a crer que o primeiro jogador está em melhor situação para negociar do que o segundo, pois vai receber mais números ao final... Tomás foi adiante e disse:

Page 14: Eureka 2008

Sociedade Brasileira de Matemática

EUREKA! N°27, 2008

14

– Sim, você está correto. Vamos jogar! Você começa, escolha o seu primeiro número. – Está bem. Eu escolho o 5. Nesse momento ele parou e me olhou curioso. – “Por que você escolheu o 5?” perguntou. Eu disse: – Não sei exatamente o que é, mas algo me faz lembrar um quadrado mágico, e como o 5 sempre está no meio, achei que tinha mais chances de ganhar... – Sua intuição mais uma vez está correta. Muito bem. Naturalmente, você deve saber qua a menos de rotações e reflexões a configuração do quadrado mágico é única. Por que a gente não desenha um quadrado mágico 3 × 3 aqui e tenta jogar olhando para ele? Após um minuto tentando lembrar como se faz um quadrado mágico, desenhamos no guardanapo:

6 7 2

1 5 9

8 3 4

Quando vi os números na mesa matei a charada. Um quadrado mágico de 3 × 3 como visto acima é uma disposição dos números 1, 2, 3, ..., 9, sem repetição, de modo que cada linha, coluna ou diagonal some a mesma quantidade. Nesse caso a soma comum será 15 e o que vemos acima são todas as maneiras possíveis de se escrever 15 como soma de 3 números: 15 = 9 + 1 + 5 = 9 + 2 + 4 = 8 + 1 + 6 = 8 + 2 + 5 = = 8 + 3 + 4 = 7 + 2 + 6 = 7 + 3 + 5 = 6 + 5 + 4. Se pensarmos então que o primeiro jogador marca X sobre os números do quadrado mágico e o segundo marca O, o objetivo do jogo passa a ser completar uma linha, coluna ou diagonal com seus símbolos. O jogo 15 out of 3 que ele me propôs nada mais é do que uma formulação equivalente, belíssima e engenhosa,

Page 15: Eureka 2008

Sociedade Brasileira de Matemática

EUREKA! N°27, 2008

15

do milenar jogo da velha (em inglês “tic-tac-toe”). Fiquei pasmo, havia ganho o meu dia. Por alguns momentos não consegui parar de pensar na beleza e no poder da matemática, presente até nos mínimos detalhes da nossa vida. Senti-me orgulhoso de poder ser um pesquisador que tenta compreender essa ciência e pequenos fatos como esse me fazem, a cada dia, ter mais consciência de que ela é muito maior do que nós. Observamos: A história acima se passou no dia 02 de maio de 2007. Fiquei com o guardanapo como recordação. O jogo 15 out of 3 é um belo exemplo para se mostrar como uma pessoa que sabe matemática realmente pode levar vantagem sobre uma pessoa menos interessada pelo assunto. Todos sabemos que o jogo da velha não admite estratégia vencedora, mas mesmo assim o professor Tamás Hausel jogava o 15 out of 3 com seus alunos e ganhava na maioria das vezes. Naturalmente, ele sempre tinha seu quadrado mágico para consultas. Até que um dia ele esqueceu-se do quadrado mágico em casa e foi derrotado por uma aluna.

Guardanapo da discussão no São Paulo´s

No verso do guardanapo acima, há outras discussões também belíssimas sobre as torres de Hanói e versões relacionadas (de formas mais engenhosas do que a

Page 16: Eureka 2008

Sociedade Brasileira de Matemática

EUREKA! N°27, 2008

16

analogia acima) inventadas pelo Dr. Rodríguez-Villegas. Ele ainda está buscando a melhor formulação para seu jogo para poder patenteá-lo e disponibilizá-lo ao público em geral. Isso então vai ficar para uma outra história. PROBLEMA 1: Prove que em um quadrado mágico 3 × 3, como foi descrito acima:

(a) a soma comum deve ser 15. (b) o número do centro deve ser 5.

PROBLEMA 2: Usando o problema anterior, prove que só existe um quadrado mágico 3 × 3 (a menos de rotações e reflexões). Verifique também que não há estratégia vencedora para o jogo da velha, em outras palavras, se os dois jogadores jogam certo, sempre dá empate. REFERÊNCIAS: [1] Para ver outras discussões sobre jogos e invariantes, há outras listas em minha página pessoal: http://www.math.utexas.edu/users/ecarneiro na seção math olympiads. [2] Para mais informações sobre os trabalhos e o curso (Math, Puzzles and Computers) do Dr. Rodriguez-Villegas sua página pessoal é www.math.utexas.edu/users/villegas/S07. Um dos jogos que ele criou está descrito no paper: Rodriguez Villegas, F.; Sadun, L.: Voloch, J.F. Blet: a mathematical puzzle. Amer. Math. Monthly 109 (2002), no. 8, 729-740.

[3] URL: página pessoal do Dr. Hausel é http://www.math.utexas.edu/users/hausel

Page 17: Eureka 2008

Sociedade Brasileira de Matemática

EUREKA! N°27, 2008

17

SUBSTITUIÇÕES ENVOLVENDO NÚMEROS COMPLEXOS Diego Veloso Uchôa

♦ Nível Avançado

É bastante útil em problemas de olimpíada onde temos igualdades ou queremos encontrar um valor de um somatório fazermos substituições por números complexos aliada a outras ferramentas. Para alguns problemas que possuam equações com funções seno e co-seno é importante saber a fórmula de Euler que escreve um número complexo na forma polar o que simplifica quando fazemos multiplicações ou somatórios. Um número complexo pode se escrever na sua forma trigonométrica

(cos sen )iρ θ θ+ ou na sua forma polar θρ ie⋅ de onde temos que

cos sen ii e θθ θ+ = (Fórmula de Euler). Segundo essa equação podemos fazer αθ = ou αθ −= , de onde temos:

cos senie iα α α= + (I) cos senie iα α α− = − (II)

Somando I com II, temos:

2cos

αα

αii ee −+

=

Subtraindo I de II, temos:

sen2

i ie ei

α α

α−−

=

Segundo a fórmula de Euler podemos verificar imediatamente a fórmula de De Moivre: Para todo n natural temos que ( )cos sen cos( ) sen( )ni n i nα α α α+ = + . No seguinte problema da OIMU, quais idéias imediatas poderíamos ter sem conhecer a fórmula de Euler? PROBLEMA 1: (OIMU – 2001) Calcule:

∑∞

=

1cos)1(cos...2coscos

n nn

nn

nnππππ

Page 18: Eureka 2008

Sociedade Brasileira de Matemática

EUREKA! N°27, 2008

18

SOLUÇÃO: Seja

=

nn

nn

nnPn

ππππ cos)1(cos...2coscos , observe

de imediato que se 2n k= então 2 0kP = pois 02

cos =

π

kk

. Portanto

considere 2 1n k= + então

++

+

+

+=+ 12

)12(cos12

2cos...12

2cos12

cos12 kk

kk

kkP k

ππππ

Observe que 2 1cos cos2 1 2 1

i k ik k

π π+ − = − ⇒ + +

12

12 )1(12

cos...12

2cos12

cos ++ −⋅

+

+

+= k

k kk

kkP πππ

,

Considere

+

+−

+

+=+

12cos

12)1(cos...

122cos

12cos12

~

kk

kk

kkP k

ππππ

Fazendo 2 1i

kw eπ+= então j

jjj

wwww

kj

21

212cos

2 +=

+=

+

π portanto

2 4 2~2 1 2

1 1 1... .2 2 2

k

k kw w wP

w w w+

+ + +=

Faça então a seguinte multiplicação no

numerador e no denominador

2 4 2 3 2 1~2 1 2 3 2 1

1 1 1 ( 1)( 1)...( 1)... ,2 2 2 ( 1)( 1)...( 1)

k k

k k kw w w w w wP

w w w w w w

+ −

+ + + + + +=

+ + +

agrupando no numerador os termos tais que

jkjjjkkjkj wwwwwww 21222212122122 )1()1)(1( −+−++−+ +=+++=++ , já que 112 −=+kw , com j variando de 1 até k. Agrupando agora os termos do

denominador podemos ver que

Page 19: Eureka 2008

Sociedade Brasileira de Matemática

EUREKA! N°27, 2008

19

2 2 1 4 2 3 2~2 1 2 2 2 1 1

1 ( )( )...( ) ,2 ( )( )...( )

k k k

k k k k k kw w w w w wPw w w w w w

− −

+ − +

+ + +=

+ + + e usando que 112 −=+kw

podemos simplificar a expressão para 2 2 4 4 2 2~

2 1 1 2 21 ( )( )...( ) .2 ( )( )...( )

k k

k k k kw w w w w wP

w w w w w w

− − −

+ − − −

− − −=

− + − − + Agora, olhando para o

numerador, podemos escolher os termos )( 22 jj ww −− tais que 2 j k> que são

m termos (para algum m) e substituí-los por 2 1 2 (2 1 2 )( 1)( )k j k jw w+ + − + −− − de forma que o numerador e o denominador serão iguais a menos de um sinal (e do fator

k2 ), i.e,

km

km

kk

kk

kk PwwwwwwwwwwwwP

21)1()1(

))...()(())...()((

21

12~

221

221

12~

−=⇒−+−−+−+−−+−

= +−−−

−−−

+

Portanto temos que 2~

1 1 12 12 1 2

1 1( 1) ( 1) ( 1) ,2 4

k k kkk k kP P+ + +++

= − = − = −

e então

como 0nP = para todo n par,

2 11 0 1

1 1 11 ...4 4 16

k

n kn k k

P P∞ ∞ ∞

+= = =

= = − − = − + − +

∑ ∑ ∑ (soma de P.G infinita), e assim

1

1 4 .1 514

nn

P∞

=

−= = −

+∑

Observação: Esse problema pede para demonstrarmos um resultado relacionado aos polinômios de Chebyshev do segundo tipo.

PROBLEMA 2: (OBM – U 2001)

Seja senxexf x ⋅= −)( . Calcule (2001) (0)f . (Denotamos por ( ) ( )nf x a derivada de ordem n no ponto x; assim, (2) ''( ) ( )f x f x= ).

SOLUÇÃO: =⋅= − senxexf x)(ieee

ixixx

2

−− −⋅ ⇒−=

+−−

ie

ie xixi

22

)1()1(

após n

derivações teremos

Page 20: Eureka 2008

Sociedade Brasileira de Matemática

EUREKA! N°27, 2008

20

( ) ( 1) ( 1)1( ) ( 1) ( 1 )2

n n x i n i xf x i e i ei

− − + = − ⋅ − − − ⋅ ; para n = 2001 e x = 0, temos

que (2001) 2001 20011(0) ( 1) ( 1 )2

f i ii = − − − − sendo que 4 4( 1) ( ( 1)) 4.i i− = − + = −

Assim

( ) ( )500500 500(2001) 4 4 10001 4(0) ( 1) ( 1) ( ( 1)) ( 1 ) (2 ) 2 .

2 2f i i i i i

i i = − ⋅ − − − + ⋅ − − = =

Ou

tro caminho possível para a solução desse problema seria: após 4 derivações de f perceber um ciclo e assim calcular (2001) (0)f , método esse mais trabalhoso do que o apresentado. PROBLEMA 3: (IMO – 1963)

Prove que 21

73cos

72cos

7cos =

+

πππ

SOLUÇÃO: Fazendo 7πi

ew = o problema se torna equivalente a demonstrar que:

⇔=+

++

−+

⇔=+

++

−+ −−−

111121

222 3

6

2

4233221

ww

ww

wwwwwwww

01)1()()( 2345636524 =+−+−+−⇔=+++−+ wwwwwwwwwwww

. Veja que isso é a soma dos termos de uma P.G cujo primeiro termo é 1 e a razão é w− . Somando a P.G:

01

111

1)( 7

=−−

−=

−−−−

=ww

wSPG . Lembre que 77 1i

w e wπ

= ⇒ = −

e portanto a igualdade é realmente verdadeira.

O seguinte resultado (muito conhecido) tem por objetivo mostrar a importância dos números complexos em problemas de alto grau de dificuldade e que aparentemente não têm nenhuma conexão com números complexos.

Page 21: Eureka 2008

Sociedade Brasileira de Matemática

EUREKA! N°27, 2008

21

PROBLEMA 4: Prove que 6

1 2

12

π=∑

=n n

SOLUÇÃO: Sabemos que para 20 π

<< x a desigualdade xxsenx tan<< é

verdadeira. De onde segue que xx

x 22

2 cot11cot +<< . Agora fazendo

12 +=

mkx π

com mk ,...,2,1= e somando de 1=k até mk = nós obtemos

(i) ∑∑∑=== +

+≤+

≤+

m

k

m

k

m

k mkm

km

mk

1

2

122

2

1

2

12cot1)12(

12cot π

ππ

Observe que essa inequação está próxima da desejada, a idéia agora é tentar mostrar que quando m →∞ o termo central fica “imprensado” entre dois limites que convergem para um mesmo valor. Para isso vamos usar um truque que usa números complexos. Pela lei de De Moivre e usando binômio de Newton temos :

0

cos( ) ( ) (cos ) (cot( ) ) cot ( )n

n n n n k n k

k

nnt isen nt t sent sen t t i sen t i t

k−

=

+ = + = + = ⋅ ⋅

Fazendo 2 1n m= + e igualando as partes imaginárias, temos:

2 2 12 1

2 1 2 1((2 1) ) (cot ) (cot ) ... ( 1)1 3

m m mm

m msen m t t tsen t

−+

+ + += − + + −

. (*)

Agora podemos tratar essa igualdade por meio de um polinômio

12 1 2 1( ) ... ( 1)

1 3m m m

mm m

P x x x −+ + = − + + −

Substituindo 12 +=

mkt π

em (*) para 1 k m≤ ≤ nos dá 0)12

(cot 2 =+m

kPmπ

,

pois sen (2 1) 02 1

kmmπ + = +

e sen 0.2 1

kmπ ≠ +

Então,

Page 22: Eureka 2008

Sociedade Brasileira de Matemática

EUREKA! N°27, 2008

22

mkmkxk ,...,1,

12cot 2 =

+=

π são as “m” raízes de mP cuja soma é

3)12(

112

312

12cot

1

2 −=

+

+

=+∑

=

mmm

m

mkm

k

π(ii)

De (i) e (ii) segue

3)12(1)12(

3)12(

122

2 −+≤

+≤

− ∑=

mmmk

mmm m

Multiplicando essas desigualdades por 2

2

)12( +mπ

e fazendo m →∞ chegamos

ao resultado desejado.

Exercícios para treinamento: PROBLEMA 5: (IME 1990/1991)

Prove que

(2 1)sen1 2cos( ) cos(2 ) ... cos( )2 2sen

2

n

n

θ

θ θ θθ

+ + + + + =

PROBLEMA 6: (IME-2000/2001) Dois números complexos são ortogonais se suas representações gráficas forem perpendiculares entre si. Prove que dois números complexos 1Z e 2Z são ortogonais se e somente se:

02121 =⋅+⋅ ZZZZ

PROBLEMA 7: Prove que 1

sen cot2

n

k

kn nπ π

=

=

PROBLEMA 8: Prove a identidade trigonométrica:

( )1

1cos ( ) cos ( 2 ) .2

nn

nk

nn k

kθ θ

=

= −

PROBLEMA 9: (IME – 2005/2006) Sejam as somas 0S e 1S definidas por

Page 23: Eureka 2008

Sociedade Brasileira de Matemática

EUREKA! N°27, 2008

23

]3/[396300 ... n

nnnnn CCCCCS +++++= 1]3/)1[(310741

1 ... +−+++++= nnnnnn CCCCCS

Calcule os valores de 0S e 1S em função de n, sabendo que [r] representa o maior inteiro menor ou igual ao número r. PROBLEMA 10: (Putnam 1970) Prove que a série de potências de )cos(bxeax ⋅ (com a e b positivos) ou não tem nenhum coeficiente zero ou possui infinitos zeros. PROBLEMA 11: Ache uma fórmula geral para:

1

0

2( 1)cos .n

k

kknπ−

=

+

PROBLEMA 12: (OBM – Nível U 2004)

Calcule o valor de 0

1 .(3 1)(3 2)(3 3)k k k k

= + + +∑

PROBLEMA 13: (IMO 1974) Prove que o número 3

1

2 12

2 1

nk

K

nk=

+ ⋅ +

∑ não é divisível

por 5 para qualquer inteiro 0≥n .

PROBLEMA 14: Calcule o valor de 2(mod3)

.k

nk≡

PROBLEMA 15: (IMC 99) Atiramos um dado (com faces de número 1, 2,..., 6) n vezes. Qual é a probabilidades de que a soma dos valores obtidos seja múltiplo de 5? Admita que as faces sejam igualmente prováveis. Dica: Use a função

2 3 4 5 6

( ) .6

nx x x x x xf x

+ + + + +=

PROBLEMA 16: Mostre que dados n pontos no círculo unitário sempre existe um outro ponto no círculo unitário tal que o produto de suas distâncias aos n pontos dados é maior ou igual a 2. PROBLEMA 17: (OBM – Nível U 2007)

Page 24: Eureka 2008

Sociedade Brasileira de Matemática

EUREKA! N°27, 2008

24

Dados números reais 1 2, ,..., na a a não todos nulos, encontre o (menor) período da função

1

( ) cos( ).n

kk

f x a kx=

= ∑

PROBLEMA 18: (Miklós Schweitzer-1956) Ache o mínimo de { }21,1 zzmáx ++ se z percorre todos os números

complexos. PROBLEMA 19: (IMO – 1995) Seja p um primo ímpar. Ache o número de subconjuntos A de { }p2,...,2,1 tais que

a) A tem exatamente p elementos b) A soma de todos elementos de A é divisível por p

Dica: Use o polinômio 2 2( , ) (1 )(1 )...(1 ).pf x y xy x y x y= + + + BIBLIOGRAFIA [1] E. Lozansky. C. Rousseau, Wining Solutions, Springer Velrlag, New York, 1996.

[2] Contests in Higher Mathematics, Hungary 1949–1961: in memoriam Miklós Schweitzer, eds.: G. Szász, L. Gehér, I. Kovács and L. Pintér, Akadémiai Kiadó, Budapest, 1968.

[3] URL: http://www.ime.eb.br (Site do Instituto Militar de Engenharia)

[4] URL : http://www.obm.org.br (Site da Olimpíada Brasileira de Matemática)

Page 25: Eureka 2008

Sociedade Brasileira de Matemática

EUREKA! N°27, 2008

25

INTEGRAIS DISCRETAS Eduardo Poço

♦ Nível Avançado Integral discreta: dizemos que ( )F n é integral discreta de ( )f n se e somente se:

( 1) ( ) ( )F n F n f n+ − = , para n inteiro (a princípio). Da mesma forma, dizemos que ( )f n é a derivada discreta de ( )F n .

Notação: ( ) ( )n

f n F n=∑

Utilidade: conhecida a integral discreta ( )F n da função ( )f n , temos condições de fazer o somatório:

( ) ( 1) ( )b

k af k F b F a

=

= + −∑ , a e b inteiros

A integral discreta transforma uma soma em soma telescópica. Sabendo de algumas propriedades, é possível trabalhar dinamicamente com integrais discretas para obter fórmulas novas a partir de outras conhecidas. Aqui, não queremos provar que uma função dada é integral discreta de outra, pois essa verificação é simples. Queremos obter ferramentas que nos possibilitem ACHAR integrais discretas de forma rápida, para no final poder calcular o valor de um somatório que tenha surgido de algum problema. Em alguns casos, é suficiente saber a “cara” da integral discreta (ou seja, se é um polinômio, exponencial etc). Algumas integrais discretas (o exercício de verificação é simples):

n

c cn=∑ . ! !n

n n n=∑

1

nnn qq

q=

−∑ ( )log log 1 !n

a an n= −∑

Page 26: Eureka 2008

Sociedade Brasileira de Matemática

EUREKA! N°27, 2008

26

cos2sen

2sen2

nkkn

knk

− − =

∑ sen

2cos2sen

2

nkkn

knk

− =

( )2 sen 2 1sen

2 4sen1

n nnn−

= −∑

( )2 sen 2 1cos

2 4sen1

n nnn−

= +∑

1

n n nk k

= + ∑

1

n n k n kn n+ +

= − ∑

Propriedades 1) Assim como integrais contínuas (as primitivas), existem várias integrais discretas para uma dada função, e todas elas diferem por uma constante. Exemplo: 2n e 2 1n + são integrais discretas de ( ) 2nf n = . Verifique pela definição! 2) Integração discreta é uma transformação linear:

[ ]. ( ) . ( ) ( ) ( )n n n

a f n b g n a f n b g n+ = +∑ ∑ ∑ , para constantes a e b.

A igualdade nos fornece uma integral discreta para a função do lado esquerdo,

lembre-se que podemos somar constantes do lado direito e continuar com uma integral discreta.

3) Integral discreta do produto (por partes): sendo ( ) ( )n

f n F n=∑ e

( ) ( )n

g n G n=∑ , então:

Page 27: Eureka 2008

Sociedade Brasileira de Matemática

EUREKA! N°27, 2008

27

( ) ( ) ( ) ( ) ( ) ( 1)n n

F n g n F n G n f n G n= − +∑ ∑

Exemplo: Calcule senn

n n∑ e 2 senn

n n∑ , comparando com o cálculo de

senx xdx∫ e 2 senx xdx∫

4) Sendo ( , )f x n uma função das variáveis x e n, derivável na variável x, então:

( , ) ( , )n n

f x n f x nx x∂ ∂

=∂ ∂∑ ∑

Podemos usar a própria variável n, se a função tiver derivada nessa variável:

( ) ( )n nd df n f n

dn dn=∑ ∑

Exemplo: Calcule n

nnx∑ , com x uma constante em relação a n.

5) Seguindo um caminho análogo, temos que:

( )( , ) ( , )n n

f x n dx f x n dx Cn = +

∑ ∑∫ ∫

Para alguma constante C. Essa constante é encontrada através de valores iniciais conhecidos das funções.

Exemplo: Prove que 01

1 1

( 1) ln 21

k nn

k

x dxk x

+

= −

−= +

−∑ ∫

Aplicação: Soma de potências consecutivas. Seja a seguinte função:

Page 28: Eureka 2008

Sociedade Brasileira de Matemática

EUREKA! N°27, 2008

28

1( ) 1 2 ...

nm m m m

mk

S n k n=

= = + + +∑

Há uma fórmula recursiva em que podemos calcular ( )mS n a partir de valores anteriores (tente prová-la como exercício):

11

0

1( 1) ( ) ( 1) 1 ( )

mm

m kk

mm S n n S n

k

−+

=

+ + = + − −

O problema dessa fórmula é a praticidade: precisamos de todas as funções anteriores, e ainda assim faremos um trabalho algébrico grande. Com integrais discretas, conseguimos obter ( )mS n a partir de 1( )mS n− apenas com um trabalho aritmético.

Inicialmente, se queremos ( )mS n , queremos sua integral discreta n

mn∑ . Usando

a propriedade que nos permite trocar a integral discreta com a contínua (escolhendo a própria variável n como variável de integração contínua):

( )1 1n n

m mn dn n dn Cn− − = +

∑ ∑∫ ∫

A integral contínua pode ser realizada sem problemas:

1mn n

mn n dn Cnm

− = +

∑ ∑∫

Renomeando a constante a ser encontrada:

1n n

m mn m n dn Cn− = +

∑ ∑∫

Essa constante pode ser encontrada pela diferença entre integrais discretas quando 0n = , fornecendo o oposto da soma dos outros coeficientes já obtidos pela integração contínua. Resumindo:

Page 29: Eureka 2008

Sociedade Brasileira de Matemática

EUREKA! N°27, 2008

29

Se 1 1 21 2 1...

nm m m

m mn a n a n a n a n− −−= + + + +∑ , então:

1 2

1 2 1...n

m m mm mn b n b n b n b n++= + + + +∑

Com 1k kmb ak −= , para 1,2,..., 1k m= + , e

1

01

m

kk

b b+

=

= −∑ .

Alguns valores:

1n

n=∑

2

2 2

n n nn = −∑

3 22

3 2 6

n n n nn = − +∑

4 3 23

4 2 4

n n n nn = − +∑

5 4 34

5 2 3 30

n n n n nn = − + −∑

Aplicação: Soma de potências multiplicadas por progressão geométrica

Agora procuraremos n

m nn x∑ , com x uma constante em relação a n. Observe:

( )1 1ln lnn n n n

m n m n m n m n m nd n x mn x n x x m n x x n xdn

− −= + = +∑ ∑ ∑ ∑

Das formas iniciais de n

m nn x∑ , encontramos uma função da forma:

( )1 1 2 21 2 2 1...

nm n n m m

m mn x x a n a n a n a n− − −− −= + + + +∑

com as constantes ka sendo funções de x, mas não dependendo de n. É natural procurar uma integral discreta com a seguinte forma:

Page 30: Eureka 2008

Sociedade Brasileira de Matemática

EUREKA! N°27, 2008

30

( )1 21 2 1...

nm n n m m

m mn x x b n b n b n b n−−= + + + +∑

Essa forma pode ser encontrada, e os coeficientes satisfazem 1k kmb ak −= ,

1,2,...,i m= e 011

m

kk

xb bx =

=− ∑ .

Alguns valores: ( )

( ) ( )( )

1

2 2

11

1 1

n n nnn n x nx xnx x n x

x x

+− −= = − − − −

∑ , 1x ≠

( )2 2 2n

n nn n= −∑

( )2 22 2 4 6n

n nn n n= − +∑

( )3 3 22 2 6 18 26n

n nn n n n= − + −∑

( )4 4 3 22 2 8 36 104 150n

n nn n n n n= − + − +∑

Problemas 1- Calcule as seguintes integrais discretas:

a) 2

3

n nn

∑ e) 2 ( 1)( 1)!

nn nn

−+∑

b) 32

nn n

∑ f) 2

1n

n n+∑

c) 1( 2) !

n

n n+∑ g) 11

n

n n+ +∑

d) 2

2

n n

∑ h) 1cos cos( 1)

n

n n +∑

Page 31: Eureka 2008

Sociedade Brasileira de Matemática

EUREKA! N°27, 2008

31

2- Calcule: 2

1

senlimn

n k

kn→∞

=∑

3- Calcule

4

1

3

1

.2lim

.2

nk

knn n

k

k

k

=

→∞

=

4- Calcule

1

1 1lim

mnm

kmn

nkm

n

+

=

→∞

−+∑

, com m inteiro positivo.

5- Prove que ( )2 321 1

2 1 1n

n n

n hnn n

∞ ∞

= =

+=

+∑ ∑ , sendo

1

1n

nk

hk=

=∑ .

6- Ache a derivada (contínua) da função gama ( )nΓ para n inteiro positivo, sabendo que '(1) γΓ = − , a constante de Euler (um valor conhecido) e (1) 1Γ = . A função gama satisfaz ( 1) ( )x x xΓ + = Γ , para todo x real, assim ( ) ( 1)!n nΓ = − para n inteiro positivo. 7- (OBM2002) O diâmetro de um conjunto S R⊂ é definido como sendo

( ) max( ) min( )D S S S= − . O conjunto vazio, por definição, tem diâmetro igual a zero. Calcule a soma dos diâmetros de todos os subconjuntos de { }1,2,3,...,A n= , em função de n. REFERÊNCIAS: [1] Uma referência sobre somatórios e algumas considerações históricas sobre o raciocínio humano e implementação de algoritmos em computadores: “A = B”, Marko Petkovsek, Herbert S. Wilf, Doron Zeilberger.

Page 32: Eureka 2008

Sociedade Brasileira de Matemática

EUREKA! N°27, 2008

32

PRODUTOS NOTÁVEIS Uma lista de problemas

Onofre Campos ♦ Nível Iniciante

1. Se x é um número real tal que tal que 1 5,xx

+ = determine o valor de 22

1 .xx

+

Solução: Elevando ambos os membros da equação 1 5xx

+ = ao quadrado,

obtemos: 2

2

1 12 25,x xx x

+ ⋅ + =

e daí, 22

1 23.xx

+ =

2. Fatore a expressão 3 25 5.E x x x= − − + Solução: Temos

3 25 5E x x x= − − + 2 ( 5) ( 5)x x x= − − − 2( 5)( 1)x x= − − ( 5)( 1)( 1).x x x= − − +

3. Simplifique a expressão 2 2 2

.( )( ) ( )( ) ( )( )

x y zAx y x z y z y x z x z y

= + +− − − − − −

Solução: Note que podemos escrever a expressão acima da seguinte forma:

2 2 2

.( )( ) ( )( ) ( )( )

x y zAx y x z x y y z x z y z

= − +− − − − − −

Assim, reduzindo a expressão ao mesmo denominador comum vem: 2 2 2( ) ( ) ( ) .

( )( )( )x y z y x z z x yA

x y y z x z− − − + −

=− − −

Por outro lado, desenvolvendo o denominador, obtemos: 2( )( )( ) ( )( )x y y z x z xy xz y yz x z− − − = − − + −

2 2 2x y xyz x z xz= − − + 2 2 2xy y z xyz yz− + + − 2 2 2( ) ( ) ( ).x y z y x z z x y= − − − + −

Page 33: Eureka 2008

Sociedade Brasileira de Matemática

EUREKA! N°27, 2008

33

Portanto: 2 2 2

2 2 2

( ) ( ) ( ) 1.( ) ( ) ( )

x y z y x z z x yAx y z y x z z x y

− − − + −= =

− − − + −

4. Se 0,x y z+ + = mostre que 3 3 3 3 .x y z xyz+ + = Solução: Observe que

3 3 3 30 ( ) 3( )( )( ).x y z x y z x y y z x z= + + = + + + + + + Como ,x y z y z x+ = − + = − e ,x z y+ = − então:

3 3 3 3 3 33( )( )( ) 0 3 .x y z y x y x y z xyz+ + + − − − = ⇒ + + =

5. Calcule o valor da expressão 3 3 3(2004) (1003) (1001) .

2004 1003 1001S

− −= ⋅ ⋅

Solução: Vamos tomar x = 1003 e y = 1001. Dessa forma, a expressão S se reduz a:

3 3 3( ) .( )

x y x ySxy x y

+ − −=

+

Mas, como sabemos, 3 3 2 2 3( ) 3 3 .x y x x y xy y+ = + + + Dessa forma, obtemos:

2 23 3 3 ( ) 3.( ) ( )

x y xy xy x ySxy x y xy x y

+ += = =

+ +

6. Sabendo que x, y e z são reais satisfazendo xyz = 1, calcule o valor da expressão:

1 1 1 .1 1 1

Ax xy y yz z xz

= + ++ + + + + +

Solução: Como xyz = 1, então 0,x ≠ 0y ≠ e 0.z ≠ Assim,

1(1 ) (1 ) 1

z xAz x xy x y yz z xz

= + ++ + + + + +

11

z xz xz xyz x xy xyz z xz

= + ++ + + + + +

Page 34: Eureka 2008

Sociedade Brasileira de Matemática

EUREKA! N°27, 2008

34

11 1 1

z xz xz x xy z xz

= + ++ + + + + +

11 1 1

z xzz xz z xz z xz

= + ++ + + + + +

11

z xzz xz

+ +=

+ +1.=

7. Se ab = 1 e 2 2 3,a b+ = determine 2 2

2 2 2.a bb a

+ +

Solução: Temos: 2 2 4 2 2 4

2 2 2 2

22a b a a b bb a a b

+ ++ + =

2 2 2

2

( )( )

a bab+

= 9.=

8. Prove que se 1x y za b c+ + = e 0,a b c

x y z+ + = então

2 2 2

2 2 2 1.x y za b c

+ + =

Solução: Elevando a equação 1x y za b c+ + = ao quadrado, obtemos:

2 2 2

2 2 2 2 1,x y z x y y z x za b c a b b c a c

+ + + + + =

ou seja, 2 2 2

2 2 2 2 1.x y z xyc xzb yzaa b c abc

+ + + + + =

Por outro lado, da equação 0,a b cx y z+ + = temos 0.xyc xzb yza+ + = Logo,

2 2 2

2 2 2 1.x y za b c

+ + =

9. Se a, b e c são três números distintos e satisfazem as equações: 3

3

3

000,

a pa qb pb qc pc q

+ + =

+ + = + + =

calcule a + b + c. Solução: Multiplicando a segunda equação por – 1 e somando com a primeira, obtemos:

3 3 ( ) 0,a b p a b− + − = ou ainda,

Page 35: Eureka 2008

Sociedade Brasileira de Matemática

EUREKA! N°27, 2008

35

2 2( )( ) ( ) 0,a b a ab b p a b− + + + − = 2 2( )( ) 0.a b a ab b p− + + + =

Como 0,a b− ≠ pois os números são distintos, obtemos: 2 2 0. (*)a ab b p+ + + =

Analogamente, multiplicando a terceira equação por – 1 e somando com a primeira equação, obtemos:

2 2 0. (**)a ac c p+ + + = Agora, multiplicando (**) por –1 e somando com (*), obtemos:

2 2 0, ab ac b c− + − = ( ) ( )( ) 0,a b c b c b c− + − + =

( )( ) 0.b c a b c− + + = Daí, como 0,b c− ≠ segue que a + b + c = 0. 10. Sejam a, b e c números reais distintos e não nulos. Se a + b + c = 0, mostre que

9.a b b c c a c a bc a b a b b c c a− − − + + + + = − − −

Solução: Façamos , e .a b b c c ax y zc a b− − −

= = =

Assim, devemos provar que 1 1 1( ) 9,x y zx y z

+ + + + =

ou seja,

9,x y z x y z x y zx y z

+ + + + + ++ + =

ou ainda,

1 1 1 9y z x z x yx y z+ + +

+ + + + + = 6.y z x z x yx y z+ + +

⇒ + + =

Mas, x y a b b c b

z c a c a+ − − = + −

2 2a ab bc c bac c a

− + −= ⋅

−2 2( ) ( )a c b a c b

ac c a− − −

= ⋅−

( )( ) ( )a c a c b a c bac c a

− + − −= ⋅

Page 36: Eureka 2008

Sociedade Brasileira de Matemática

EUREKA! N°27, 2008

36

( )( )a c a c b bac c a

− + −= ⋅

−( )b b b

ac− −

= −22 .b

ac=

Analogamente, concluímos que 22y z c

x ab+

= e 22 .x z a

y bc+

= Logo, pelo exercício

4, segue que 2 2 22 2 2y z x z x y a b c

x y z bc ac ab+ + +

+ + = + +3 3 3

2 a b cabc

+ +=

32 abcabc

= ⋅

6,=

como queríamos provar. Exercícios Propostos

1. Fatore a expressão 4 2 1.S x x= + + 2. Determine a expressão que deve ser multiplicada por 3 32 2x x+ para obtermos 22 ( 4).x x + 3. Calcule o valor da expressão

2 22 2 2 2 2 2

3 2 3 2

( 1) ( 1) ( 1) ( 1) .( 1) ( 1)

x x x x x xSx x

+ − + + + += ⋅ − +

4. Se 2 2 3 ,x y xy+ = calcule 1 1 .x yy x

+ +

5. Simplifique

( ) ( ) ( )2 22 2 2 2 2 2 .x y z xy yz xz x y z x y z+ + + + + − + + + +

6. Fatore as seguintes expressões: (a) 3 25 3 9;x x x+ + − (b) 3 3 3( ) ( ) ( ) ;x y z x z y y z x− − − + − (c) 2 2( 3)( 4) 12;x x x x+ + + + − (d) 4 44 ;x y+ (e) 3 3 3( ) ( ) ( ) ;x y y z z x− + − + − (f) 3 3 3 3( ) ;x y z x y z+ + − − − (g) 3 3 3( 2 3 ) ( 2 3 ) ( 2 3 ) .a b c b c a c a b+ − + + − + + − 7. Simplifique as expressões:

(a) 2 4 8

1 1 1 1 1 ;1 1 1 1 1x x x x x

− − − −− + − − −

Page 37: Eureka 2008

Sociedade Brasileira de Matemática

EUREKA! N°27, 2008

37

(b) 1 1 1 ;( )( ) ( )( ) ( )( )x y x z y x y z z x z y

+ +− − − − − −

(c) 2 2 3 2 2 3 2 2 3

3 3 3

( ) ( ) ( ) .( ) ( ) ( )

x y y z z xx y y z z x− + − + −− + − + −

8. Prove que se 0,x y zy z z x x y

+ + =− − −

então

2 2 2 0.( ) ( ) ( )

x y zy z z x x y

+ + =− − −

9. Para que os valores de a∈ a expressão 4 4a + é um número primo? 10. Prove que se a + b + c = 0 então

5 5 5 3 3 3 2 2 2

.5 3 2

a b c a b c a b c+ + + + + += ⋅

11. Mostre que 7 7 7 2 2 2( ) 7 ( )( ) .a b a b ab a b a ab b+ − − = + + + 12. Prove que se a + b + c = 0, então

7 7 7 5 5 5 2 2 2

.7 5 2

a b c a b c a b c+ + + + + += ⋅

13. Se a, b e c são reais não nulos que satisfazem a + b + c = 0, calcule 3 3 3 2 4 4 4

5 5 5 2

( ) ( ) .( )

a b c a b ca b c

+ + + ++ +

14. Prove que se x, y e z são racionais distintos então a expressão

2 2 2

1 1 1( ) ( ) ( )y z z x x y

+ +− − −

é um quadrado perfeito. 15. Fatore 3 3 3 38( ) ( ) ( ) ( ) .x y z x y y z x z+ + − + − + − +

Page 38: Eureka 2008

Sociedade Brasileira de Matemática

EUREKA! N°27, 2008

38

OLIMPÍADAS AO REDOR DO MUNDO

Apresentamos, como sempre, questões que não são encontradas facilmente na Internet. Divirtam-se e enviem as suas soluções.

Continuamos à disposição na OBM para aqueles que estiverem interessados na solução de algum problema particular. Para tanto, basta contactar a OBM, através de carta ou e-mail.

Bruno Holanda Carlos Augusto David Ribeiro

Primeiramente vamos aos problemas propostos deste número 224.(Balcânica Junior - 2007) Seja a um real positivo tal que 3 6( 1).a a= + Prove

que a equação 2 2 6 0x ax a+ + − = não possui solução real. 225.(Bulgária - 2007) Ache todos os inteiros positivos x, y tais que o número

2 2( )( )x y y x+ + é a quinta potência de um primo. 226.(Inglaterra - 2007) Seja ABC um triângulo acutãngulo com AB AC> e

60 .BAC∠ = ° Seja O o circuncentro e H o ortocentro. A reta OH encontra AB em P e AC em Q. Prove que PO = HQ. 227.(Austria - 2007) Sejam 0 1 6690 , ,..., 1x x x< < reais distintos. Mostre que existem , {0,1,...,669}i j∈ para os quais

10 ( ) .2007i j j ix x x x< − <

228.(Bulgária - 2008) Para cada inteiro positivo n, seja ( )nτ a quantidade de divisores de n maiores que 2008. Defina 0na = se ( )nτ é par e 1na = caso

contrário. O número 1 20, ... ...na a aα = é racional? 229.(Bielorrússia - 2001) No losango ABCD, 60 .A∠ = ° Os pontos F, H, e G estão sobre os segmentos AD, DC e AC de modo que DFGH é um paralelogramo. Prove que FBH é um triângulo eqüilátero.

Page 39: Eureka 2008

Sociedade Brasileira de Matemática

EUREKA! N°27, 2008

39

230.(Rússia - 2007) Sejam a, b, c números reais. Prove que pelo menos uma das três equações

2 ( ) ( ) 0,x a b x b c+ − + − = 2 ( ) ( ) 0,x b c x c a+ − + − = 2 ( ) ( ) 0,x c a x a b+ − + − =

possui solução real. 231.(Bulgária - 2007) No triângulo ,ABC∆ com 60 ,ACB∠ = ° sejam 1AA e 1BB

1 1( , )A BC B AC∈ ∈ as bissetrizes de BAC∠ e .ABC∠ A reta 1 1A B encontra o

circuncírculo do triângulo ABC∆ nos pontos 2A e 2.B (a) Sejam O e I o circuncentro e o incentro do ABC∆ , respectivamente, prove que OI é paralelo a 1 1A B . (b) Se R é o ponto médio do arco AB, não contendo o ponto C, P e Q são os pontos médios de 1 1A B e 2 2A B , respectivamente, prove que RP = RQ.

232.(Romênia - 2007) Encontre todos os conjuntos A de pelo menos dois inteiros positivos, tais que para quaisquer dois elementos distintos ,x y A∈ tenhamos

.( , )

x y Amdc x y

+∈ Aqui mdc(x, y) denota o máximo divisor comum de x e y.

233.(Romênia - 2007) Determine todas as progressões aritméticas infinitas de inteiros positivos, com a seguinte propriedade: existe ,N ∈ tal que para qualquer primo p, p > N, o p-ésimo termo da seqüência também é primo. 234.(Bulgária - 2007) O incírculo do triângulo acutângulo ABC∆ toca os lados AB, BC e CA nos pontos P, Q e R respectivamente. O ortocentro H do triângulo

ABC∆ está sobre o segmento QR. (a) Prove que .PH QR⊥ (b) Sejam I e O o incentro e o circuncentro do ABC∆ , respectivamente, e N o ponto comum entre o lado AB e ex-incírculo relativo a este lado. Prove que os pontos I, O e N são colineares.

235.(Olimpíada Checa e Eslovaca – 2007) Se x, y, z são números reais no intervalo (–1, 1) satisfazendo xy + yz + zx = 1, mostrre que:

2 2 2 236 (1 )(1 )(1 ) 1 ( ) .x y z x y z− − − ≤ + + +

Page 40: Eureka 2008

Sociedade Brasileira de Matemática

EUREKA! N°27, 2008

40

236.(Romênia – 2007) Um conjunto de pontos no plano é livre se não existe triângulo eqüilátero cujos vértices estão entre os pontos do conjunto. Mostre que qualquer conjunto de n pontos no plano contém um subconjunto livre com pelo menos n pontos. SOLUÇÃO DE PROBLEMAS PROPOSTOS NOS NÚMEROS ANTERIORES: 211. (Baltic Way – 2004) Uma seqüência a1, a2, ... de números reais não-negativos satisfaz, para n = 1, 2, ..., as seguintes condições: (a) an + a2n ≥ 3n. (b) )1(21 +≤++ nana nn . (i) Prove que an ≥ n para todo n = 1, 2, ... (ii) Dê exemplo de uma tal seqüência. SOLUÇÃO DE ESTILLAC LINS MACIEL BORGES FILHO (BELÉM – PA) (i) Utilizando a desigualdade das médias, temos:

112

12)1(2 111 +≤⇒++≤+⇒

++≤+≤+ +++ nnnn

nnn aanana

nanana

Supondo válido que kaa nkn +≤+ , temos que:

111 ++≤+≤ +++ kaaa nknkn E como a desigualdade vale para 1=k , fica provado por indução que

kaa nkn +≤+ , k∀ ∈ . Em particular, para nk = , temos:

naaanaa nnnnn +≤+⇒+≤ 222

Finalmente, como naa nn 32 ≥+ , temos:

nananaaan nnnnn ≥⇒≥⇒+≤+≤ 2223 2

(ii) Uma seqüência que satisfaz as condições é 1+= nan . De fato, temos:

(1) nnnnaa nn 3231212 ≥+=+++=+

(2) )1(2)1(2)1)(1(2)1(2221 +≤+=++=+=+=++ nanannnnna nnn 212. (Baltic Way – 2004) Seja P um polinômio com coeficientes não-negativos. Prove que se P(1/x)P(x) ≥ 1 para x = 1, então tal desigualdade se verifica para todo real positivo x.

Page 41: Eureka 2008

Sociedade Brasileira de Matemática

EUREKA! N°27, 2008

41

SOLUÇÃO DE GELLY WHESLEY (FORTALEZA – CE) Para 0x > temos ( ) 0.P x >

Da condição dada, temos 2( (1)) 1.P ≥

Agora, denote 10 1( ) ... .n n

nP x a x a x a−= + + + Então:

( )1

10 1 0 1

1 1 1( ) ... ...n n

n nn nP x P a x a x a a a a

x x x

−−

= + + + ⋅ + + +

≥ (Por Cauchy-Schwarz) 2

10 10 1

1

...n nn n

n n

a aa x a x a ax x

⋅ + ⋅ + + ⋅ =

2 2

0 1( ... ) ( (1)) 1.na a a P+ + + = ≥ 213. (Baltic Way – 2004) Ache todos os conjuntos X, consistindo de ao menos dois inteiros positivos, tais que para todos m, n ∈ X, com n > m, exista um elemento k de X tal que n = mk2. SOLUÇÃO DE ESTILLAC LINS MACIEL BORGES FILHO (BELÉM – PA) Suponha que o número 1 .X∈ Logo, seja 1 com , >∈ pXp , temos que deve

existir Xk ∈ , tal que 2kp = , isto é, p é quadrado perfeito. Obviamente, 1>k e com raciocínio análogo, concluímos que k também deve ser quadrado

perfeito e sua raiz quadrada deve pertencer a X . Ou seja, estendendo o

raciocínio, todas as potências de p da forma np 21

, com n∈ também devem

pertencer a X , o que é impossível, dado que 1

2np ∉ para algum valor de n . Logo, X∉1 . Vamos então tentar montar o conjunto X . Para tal, vamos supor, inicialmente que X possua somente dois elementos 1p e 2p , com 112 >> pp . Desta

forma, temos que deve existir Xk ∈ , tal que 212 kpp = . Obviamente, 2pk <

e, portanto, a única alternativa é 1pk = e, portanto, 312 pp = . De fato, todo

conjunto { }3, nnX = , com Nn∈ e 1>n , satisfaz as condições do problema e somente tais conjuntos de dois elementos satisfazem, conforme verificado. Vamos então supor que o conjunto X tenha mais do que dois elementos, isto é,

{ }rpppppX ,...,,,, 4321= , com Npp r∈,...,1 . Suponha, sem perda de

Page 42: Eureka 2008

Sociedade Brasileira de Matemática

EUREKA! N°27, 2008

42

generalidade, que 1... 1231 >>>>>> − ppppp rr . Com um raciocínio

análogo ao parágrafo anterior, concluímos que 312 pp = . Seguindo o raciocínio,

temos que: (1) 2

113 kpp =

(2) 22

313 kpp =

onde Xkk ∈21 , . Obviamente, temos que 31 pk < , 32 pk < e 21 kk ≠ . Portanto, como 3p é o

terceiro menor elemento do conjunto, só restam as possibilidades

=

=3

12

11

pk

pk ou

==

12

311

pkpk

. Porém, no primeiro caso, temos que 23

13 ppp == : absurdo! Já no

segundo caso, temos que: (1) 7

123

113 )( pppp ==

(2) 51

21

313 pppp ==

Portanto, temos que 1ou 0 117

15

1 ==⇒= pppp . Absurdo! Logo, não é possível que o conjunto X possua mais de 2 elementos. E assim, todos os conjuntos que satisfazem o enunciado são:

{ }3, nnX = , com n∈ e 1>n . 223. (Bielorússia – 2005) Seja H o ponto de interseção das alturas BB1 e CC1 do triângulo acutângulo ABC. Seja uma reta passando por A, tal que .AC⊥ Prove que as retas BC, B1C1 e possuem um ponto em comum se e somente se H for o ponto médio de BB1.

Page 43: Eureka 2008

Sociedade Brasileira de Matemática

EUREKA! N°27, 2008

43

SOLUÇÃO DE DAVI LOPES ALVES DE MEDEIROS (FORTALEZA – CE)

A

M C1

B1

B

C

P

Seja P o ponto de interseção de BC e . É suficiente mostrarmos que 1 1, ,B C P

são colineares 1.BM MB⇔ =

i) 1, ,C M C colineares ⇒ por Menelaus no 1 11

1 1

: 1,CB AC BMBABAC C B MB

∆ ⋅ ⋅ = (I)

ii) 1BB AC⊥ e 1//AC BB⊥ ⇒ e daí 1,ACP BCB∆ ∆∼ donde

1

1

ABPBBC CB

= (II) e 1

PC CBAC CB

= (III)

iii) 1,P B e 1C são colineares 1 1

1 1

1AB C BPCPB CB AC

⇔ ⋅ ⋅ = (IV)

Multiplicando (I) e (IV) membro a membro, temos que 1,P B e 1C são colineares

1 1 1 1 1

1 1 1 1 1

1 1CB AC AB C B ABBM PC BM PCAC C B MB PB CB AC MB AC PB

⇔ ⋅ ⋅ ⋅ ⋅ ⋅ = ⇔ ⋅ =

(V)

Mas de (II) : 1 1AB CBPB BC

= e multiplicando este resultado por (III)

1 1

1

1AB CBPC CBAC PB CB CB

⋅ = ⋅ = (*)

Page 44: Eureka 2008

Sociedade Brasileira de Matemática

EUREKA! N°27, 2008

44

Substituindo (*) em (V), temos que 1 1, ,P B C são colineares

11

1 .BM BM MBMB

⇔ = ⇔ = c.q.d.

224. (Bielorússia – 2005) Ache todas as funções f : N → N satisfazendo

)()())(( nfmfnfnmf +=+− ,

para todos m, n ∈ N. SOLUÇÃO DE ESTILLAC LINS MACIEL BORGES FILHO (BELÉM – PA) Primeiramente, seja k∈ . Fazendo knm == , temos:

)(2))(()()())(( kfkffkfkfkfkkf =⇒+=+− , k∀ ∈ Em seguida, façamos 0=m e )(kfn = , temos:

)(2)0())(2)(())(()0()))(()(( kffkfkffkfffkffkff +=+−⇒+=+−0)0()(2)0())(( =⇒+=⇒ fkffkff

Agora, seja x∈ , tal que )1(fx = . Tomando 1+= km e 1=n , temos: )1()( ++=+ kfxxkf , k∀ ∈

Seja t∈ , com 1≥t . Substituindo sucessivamente na equação anterior k por )1()( −+− jxjt , com tj ≤≤1 , temos:

)1)1(()( +−+= xtfxtxf )2)2(()1)1(( +−+=+− xtfxxtf )3)3(()2)2(( +−+=+− xtfxxtf

))(()1)1(( jxjtfxjxjtf +−+=−++−

)1()22( −++=−+ txfxtxf

)()1( tfxtxf +=−+ Somando as equações anteriores, temos:

)()( tftxtxf += , t∀ ∈ Com este resultado em mãos, vamos provar, por indução, que kxkf =)( ,

k∀ ∈ . De fato, temos: (1) Se 0=k , temos kxfkf === 0)0()( (2) Se 1=k , temos kxxfkf === )1()(

Page 45: Eureka 2008

Sociedade Brasileira de Matemática

EUREKA! N°27, 2008

45

(3) Se txtf =)( , para t∈ , temos: ( 1 ) ( ) ( 1) ( ( )) (( ) ) ( 1) ( 1)f t x x f t x tx t x f f t i x f t i x t x f t− + = + = + = + ⇒ − + = + = + + +

2 ( 1 ) ( 1) ( 1)f t x t x f t⇒ − + = + + +xttfxttfxt )1()1()1(2)1()1( +=+⇒+=+++⇒

Portanto, nos resta descobrir quais os valores possíveis para )1(fx = . Para isso, vamos utilizar o último resultado encontrado na relação inicial proposta no problema, para 0≠n :

2)())(( nxnxmxxnxnmnfnmf +−=+−=+− nxmxnfmf +=+ )()(

2 2 2

02 2 ou

2

xmx nx nx mx nx nx nx x x

x

=⇒ − + = + ⇒ = ⇒ = ⇒ =

Logo, as funções possíveis são: 0)( =yf e yyf 2)( = . É fácil ver que ambas satisfazem as condições e, portanto, são todas as funções procuradas. 227. (Bulgária – 2005) Ivo escreve todos os inteiros de 1 a 100 (inclusive) em cartas e dá algumas delas para Iana. Sabe-se que para quaisquer duas destas cartas, uma de Ivo e outra de Iana, a soma dos números não está com Ivo e o produto não está com Iana. Determine o número de cartas de Iana sabendo que a carta 13 está com Ivo. SOLUÇÃO DE ESTILLAC LINS MACIEL BORGES FILHO (BELÉM – PA) Primeiramente, notamos que a carta de número 1 deve estar com Iana. De fato, se a carta 1 estivesse com Ivo, para qualquer carta y de Iana, o produto dos números também estaria com Iana, contradizendo a hipótese. Isso implica que ou a carta 1 pertence a Iana ou Iana não possui cartas, o que não é verdade por hipótese. Sendo assim, Iana possui a carta 1 e, dada qualquer carta x de Ivo, temos que Iana deverá possuir a carta 1+x . Continuando o raciocínio, temos que Iana também deverá possui as cartas

12 +x , 13 +x , ..., 1+kx , com 1 100kx + ≤ e k∈ . Além disso, Iana também deve possuir a carta 1−x , pois caso contrário, Iana deveria possui a carta xx =+− 11 , o que não é verdade. E logo, Iana também deve possuir as cartas 12 −x , ..., 1−kx , com 1 100kx − ≤ e k∈ . Assim, sabendo que Ivo possui a carta de número 13 , já sabemos que Iana possui as cartas:

92 ,90 ,79 ,77 ,66 ,64 ,53 ,51 ,40 ,38 ,27 ,25 ,14 ,12 ,1

Page 46: Eureka 2008

Sociedade Brasileira de Matemática

EUREKA! N°27, 2008

46

Notemos agora que a carta número 2 deve pertencer à Iana. De fato, se a carta pertencesse a Ivo, teríamos pelo raciocínio anterior que Iana deveria possuir todas as cartas ímpares, o que não é possível, já que Ivo possui a carta número 13 . Como a carta 2 pertence a Iana e a carta 13 pertence a Ivo, temos que a carta 26 deve pertencer a Ivo e, conseqüentemente, a carta 52 também. Portanto, até o momento, temos: Ivo: 52 ,26 ,13 Iana: 92 ,90 ,79 ,77 ,66 ,64 ,53 ,51 ,40 ,38 ,27 ,25 ,14 ,12 ,2 ,1 Novamente, temos que a carta 3 deve pertencer a Iana, pois caso pertencesse a Ivo, a carta 6 3 2= × também pertenceria a Ivo, assim como a carta 12 6 2= × , o que não é verdade. Analogamente, a carta 4 também pertence a Iana, pois se pertencesse a Ivo, também pertenceria a Ivo a carta 12 4 3= × . Isso implica que as cartas 39 e 78 pertencem a Ivo. Portanto, até o momento: Ivo: 78 ,52 ,39 ,26 ,13 Iana: 92 ,90 ,79 ,77 ,66 ,64 ,53 ,51 ,40 ,38 ,27 ,25 ,14 ,12 ,4 ,3 ,2 ,1 Se Ivo possuísse o número 5 , também deveria possuir o número 10 5 2= × e assim, deveria possuir o número 40 10 4= × , o que não é verdade. Logo Iana possui o número 5. Já os números 6 e 7 também não podem estar com Ivo, pois neste caso, os números 12 6 2= × e 14 7 2= × não poderiam estar com Iana, o que não acontece. Logo, Iana também possui os números 6 e 7 . Assim, até o momento: Ivo: 78 ,52 ,39 ,26 ,13 Iana: 92 ,90 ,79 ,77 ,66 ,64 ,53 ,51 ,40 ,38 ,27 ,25 ,14 ,12 ,7 ,6 ,5 ,4 ,3 ,2 ,1 Analogamente, temos que se 11 e 10 ,9 ,8 pertencessem a Ivo, 40 8 5= × , 27 9 3= × , 40 10 4= × e 77 11 7= × também deveriam pertencer a Ivo, o que não é verdade. Logo, 11 e 10 ,9 ,8 também pertencem a Iana. Neste momento, temos que: Ivo: 78 ,52 ,39 ,26 ,13 Iana: 92 ,90 ,79 ,77 ,66 ,64 ,53 ,51 ,40 ,38 ,27 ,25 ,14 ,12 ,11 ,10 ,9 ,8 ,7 ,6 ,5 ,4 ,3 ,2 ,1 Agora notamos que Ivo possui todos os múltiplos de 13 menores que 100 . Todos os demais números menores que 100 devem então ter a forma tk +13 , com ,k t∈ e 130 << t . Como Iana possui todos os números t∈ com

130 << t , temos que todos os número menores que 100 que não são múltiplos de 13 devem pertencer a Iana, pois k13 pertence a Ivo e a soma não pode pertencer a Ivo.

Page 47: Eureka 2008

Sociedade Brasileira de Matemática

EUREKA! N°27, 2008

47

Logo, Ivo possui apenas 7 números e Iana possui os 93 restantes. 230. (Eslovênia – 2005) Denote por I o incentro do triângulo ABC. Sabe-se que AC + AI = BC. Encontre a razão entre as medidas do ângulos ∠BAC e ∠CBA. SOLUÇÃO DE RAFAEL ALVES DA PONTE (FORTALEZA – CE) Denote por I o incentro do triângulo ABC. Sabe-se que AC + AI = BC. Encontre a razão entre as medidas dos ângulos BAC∠ e .CBA∠

C B

A

I β θ θ

α α

Construa ´AI em AC de modo que ´AI AI= , conforme a figura acima. Sejam ´BAI IAC α∠ = ∠ = e .IBA IBC θ∠ = ∠ = Note que ´BI C∆ é isósceles, e sendo ´ ,I BA β∠ = ´BI A∠ mede 2β θ+ e, pelo teorema do ângulo externo,

2 2 ,BAC β θ∠ = + donde vem α β θ= + [*]. Veja que ´ 180 ,I AI α∠ = °− e

visto que ´I BI α∠ = (por [*]), ´I BIA é inscritível. Como as cordas AI e ´AI são congruentes, ,β θ= daí

2 2 2 2 2 2BACBAC CBACBA

α θ α θ ∠= ⇔ = ⋅ ⇔ ∠ = ∠ ⇔ =

Page 48: Eureka 2008

Sociedade Brasileira de Matemática

EUREKA! N°27, 2008

48

COMO É QUE FAZ?

PROBLEMA PROPOSTO POR WILSON CARLOS DA SILVA RAMOS (BELÉM – PA) Dado um triângulo ABC com incentro I, considere uma reta variável l passando por I que intersecta o lado AB em P, o lado AC em N e a reta suporte do lado BC

em M. Prove que o valor de AB AC BCPA PB NA NC MB MC

+ −⋅ ⋅ ⋅

independe da escolha de l.

SOLUÇÃO: Suponha, sem perda de generalidade, que B está entre M e C.

β/2

A

B C

N

P

M

I

α/2 α/2

θ

β/2 γ/2

γ/2

ε φ

Primeiro, note que PBPAPBPA

PBPAPBPA

AB 11+=

⋅+

=⋅

,

NCNANCNANCNA

NCNAAC 11

+=⋅+

=⋅

e MCMBMCMB

MBMCMCMB

BC 11−=

⋅−

=⋅

, de

modo que queremos provar que

MCMBNCNAPBPA111111

+−+++

não depende da reta l. Isso é um trabalho para a lei dos senos! De fato, nos triângulos AIP e AIN, e

lembrando que, sendo r o inraio de ABC, 2sen α

rAI = ,

Page 49: Eureka 2008

Sociedade Brasileira de Matemática

EUREKA! N°27, 2008

49

( )( )θθ

θθ

αα

α sensensen1

sensen22

2 rPAAIAP +

=⇔+

= e

( )( )θθ

θθ

αα

α sensensen1

sensen22

2 rNAAINA −

=⇔−

=

Somando PA1 e

NA1 , obtemos

( ) ( )( )θ

θθ ααα

sensensensen11 222

rNAPA−++

=+

Utilizando a fórmula de Prostaférese ( ) ( )22 cossen2sensen yxyxyx −+=+ ,

( ) ( ) ( ) ( )( )

rrrNAPAα

θθ

θ

θθθθαα

ααααα

sensen

cossen2sensen

2cos

2sen2sen

11 22

22222

==

−−+

−++

=+

Sorte grande! Esse valor não depende da escolha de l, já que r e α só dependem do triângulo ABC. Podemos concluir, analogamente, que

rMCNCγsen11

=+

também não depende de l.

Já o caso de MBPB11

− , como era de se esperar, é um pouquinho diferente, mas

só um pouquinho: pela lei dos senos nos triângulos MIB e PIB,

( )( )φ

φ

φφ

ββ

β sensensen1

sensen22

2rMB

BIMB −=⇔

−= e

( )( )φ

φ

φφ

ββ

β sensensen1

sensen22

2rPB

BIPB +=⇔

+=

Note a mudança de sinal de 2αθ − para φβ

−2

: “destrocando” o sinal, obtemos

( )φφ ββ

sensensen1 22

rMB−

=− e aí podemos trabalhar como nos demais casos, obtendo

rPBMBβsen11

=+−

A soma pedida é, então, igual a r

γβα sensensen ++ , que não depende de l.

Page 50: Eureka 2008

Sociedade Brasileira de Matemática

EUREKA! N°27, 2008

50

SOLUÇÕES DE PROBLEMAS PROPOSTOS Publicamos aqui algumas das respostas enviadas por nossos leitores.

110) Um conjunto finito de inteiros positivos é chamado de Conjunto DS se cada elemento divide a soma dos elementos do conjunto. Prove que todo conjunto finito de inteiros positivos é subconjunto de algum conjunto DS. SOLUÇÃO DE ZOROASTRO AZAMBUJA NETO (RIO DE JANEIRO – RJ) Basta provar que, para todo n inteiro positivo, existe um conjunto DS que contém {1,2,...,n}. Para 3n ≤ , isso segue do fato de {1,2,3} ser um conjunto DS. Vamos mostrar, por indução em n, que, para todo 3n ≥ , existe um conjunto DS,

1 2 ( ){ , ,..., },n k nX a a a= com ja j= para 1 .j n≤ ≤ Note que a soma dos seus

elementos ( )

1

k n

n jj

S a=

= ∑ é par (pois 2 2 2 | ).n na X S= ∈ ⇒

Dado 3n ≥ e um conjunto nX como acima, podemos tomar

1 2 3 ( )( 1)( 2) ( 1)( 2) ( 1)( 2)1, 2,3,..., , 1, , ,...,

2 2 2n k nn n n n n nX n n a a a++ + + + + + = +

A soma de seus elementos é

( )

1 12

1

( 1)( 2) ( 1)( 2) ( 1)( 2)1 2 ... ( 1) ( )2 2 2

( 1)( 2) , pois 1.2

k n

n j nj

n

n n n n n nS n n a S a

n n S a

+=

+ + + + + += + + + + + + = + − =

+ += ⋅ =

Como nS é par, 1nS + é múltiplo de n + 1. Como nS é múltiplo de ja para todo j,

1( 1)( 2)

2n nn nS S+

+ += ⋅ é múltiplo de

( 1)( 2)2 j

n n a+ +⋅ para todo j, e logo

1nX + é um conjunto DS. 111) Prove que existem infinitos múltiplos de 7 na seqüência ( )na abaixo:

1 11999, ( ), 2n na a a p n n−= = + ∀ ≥ , onde p(n) é o menor primo que divide n.

Page 51: Eureka 2008

Sociedade Brasileira de Matemática

EUREKA! N°27, 2008

51

SOLUÇÃO DE JOSÉ DE ALMEIDA PANTERA (RIO DE JANEIRO – RJ) Seja N um número da forma 510510 2 3 5 7 11 13 17 ,r r= ⋅ ⋅ ⋅ ⋅ ⋅ ⋅ ⋅ onde r é um inteiro positivo. Temos então ( 2) 2,p N + = ( 3) 3,p N + = ( 4) 2,p N + =

( 5) 5,p N + = ( 6) 2,p N + = ( 7) 7,p N + = ( 8) 2,p N + = ( 9) 3,p N + =( 10) 2,p N + = ( 11) 11,p N + = ( 12) 2,p N + = ( 13) 13,p N + =( 14) 2,p N + = ( 15) 3,p N + = ( 16) 2,p N + = e ( 17) 17.p N + = Assim,

2 1 3 12(mod 7), 5(mod 7),N N N Na a a a+ + + +≡ + ≡ + 4 1(mod 7),N Na a+ +≡

5 1 5(mod 7),N Na a+ +≡ + 6 1(mod 7),N Na a+ +≡ 7 1(mod 7),N Na a+ +≡

8 1 2(mod 7),N Na a+ +≡ + 9 1 5(mod 7),N Na a+ +≡ + 10 1(mod 7),N Na a+ +≡

11 1 4(mod 7),N Na a+ +≡ + 12 1 6(mod 7),N Na a+ +≡ + 13 1 5(mod 7),N Na a+ +≡ +

14 1(mod 7),N Na a+ +≡ 15 1 3(mod 7),N Na a+ +≡ + 16 1 5(mod 7)N Na a+ +≡ + e

17 1 1(mod 7),N Na a+ +≡ + e portanto sempre há um múltiplo de 7 em

1 2 3 11 12 15 17{ , , , , , , },N N N N N N Na a a a a a a+ + + + + + + pois 1,Na + 2 ,Na + 3,Na + 11,Na + 12 ,Na +

15 ,Na + 17Na + percorrem todas as classes de congruência módulo 7. 112) Determine todos os inteiros positivos n tais que existe uma matriz n × n com todas as entradas pertencentes a { –1, 0, 1} tal que os 2n números obtidos como somas dos elementos de suas linhas e de suas colunas são todos distintos. SOLUÇÃO DE ASDRÚBAL PAFÚNCIO SANTOS (BOTUCATU – SP) Vamos mostrar que existe uma matriz como no enunciado se e somente se n é par. Se n é par, digamos n = 2k, podemos construir uma matriz 1 , 2( )ij i j kA a ≤ ≤=

com 1ija = se 1 , , 1iji j k a≤ ≤ = − se 1 , 2 , 1ijk i j k a+ ≤ ≤ = se i k≤ e

,j k i≥ + 0ija = se i k≤ e 1 , 1ijk j k i a+ ≤ < + = − se 1,i k j k≥ + ≤ e

,j k i+ > e 0ija = se 1i k≥ + e .j k i+ ≤ É fácil ver que os 2n números obtidos como somas dos elementos das linhas e das colunas de A é { , 1, 2,...,1,0, 1,..., ( 2), ( 1)}.n n n n n− − − − − − − Suponha agora que exista uma matriz 1 ,( )ij i j nA a ≤ ≤= como no enunciado, com

{ 1,0,1}, , .ija i j n∈ − ∀ ≤ Claramente permutar linhas ou colunas não altera as

propriedades do enunciado. Como há 2 1n + elementos em { , ( 1),..., 1,0,1,..., 1, },n n n n− − − − − um desses elementos, digamos c, não é valor da soma dos elementos de nenhuma linha ou coluna de A. Podemos supor (trocando o sinal de todos os elementos de A, se necessário) que esse elemento c

Page 52: Eureka 2008

Sociedade Brasileira de Matemática

EUREKA! N°27, 2008

52

que falta é menor ou igual a 0. Podemos supor (permutando linhas e colunas, se necessário) que os valores 1, 2,..., n – 1, n são obtidos como somas dos elementos das primeiras k linhas e das primeiras n – k colunas de A, para certo .k n≤ Sejam

11

,iji kj n k

x a≤ ≤≤ ≤ −

= ∑ 1

,iji k

n k j n

y a≤ ≤− < ≤

= ∑ 1

ijk i n

j n k

z a< ≤

≤ ≤ −

= ∑ e

.ijk i n

n k j n

w a< ≤− < ≤

= ∑ Temos então

( 1) 1 2 ... ( ) ( ) 2 ,2

n n n x y x z x y z+= + + + = + + + = + + e

( 1)2

n n c+− − =

( ) ( ) 2 .z w y w w y z= + + + = + + Portanto, 22 2 ( 1) ,x w n n c n− = + + ≥ pois ,c n≥ − e, como claramente temos ( )x k n k≤ − e ( ), 4 ( )w k n k k n k≥ − − − ≥

2 2 2 22 2 0 4 4 ( 2 ) ,x w n n kn k n k≥ − ≥ ⇒ ≥ − + = − donde 2 0,n k− = e portanto n é par. Note que, nesse caso, o elemento c que falta deve ser necessariamente igual a –n (ou n, se for positivo). 114) Sabendo que 0sen x sen y sen z sen w+ + + = e cos cos cos cos 0,x y z w+ + + = mostre que

2003 2003 2003 2003 0.sen x sen y sen z sen w+ + + = SOLUÇÃO BASEADA NAS SOLUÇÕES ENVIADAS POR SAMUEL LILÓ ABDALLA E DOUGLAS RIBEIRO SILVA De cos cos cos cos 0x y z w+ + + = e sen sen sen sen 0,x y z w+ + + = obtemos

(cos sen ) (cos sen ) (cos sen ) (cos sen ) 0.ix iy iz iwe e e e x i x y i y z i z w i w+ + + = + + + + + + + =Vamos supor sem perda de generalidade 0 2 .x y z w π≤ ≤ ≤ ≤ < Temos

y x π− ≤ e ,w z π− ≤ senão os quatro números complexos , ,ix iy ize e e e iwe pertenceriam a um mesmo semicírculo do círculo unitário, e sua soma não poderia ser 0. Temos (cos cos ) (sen sen )ix iye e x y i x y+ = + + + =

22cos cos 2cos .2 2 2 2

x yiy x x y x y y xsen i e+

− + + − = + =

Analogamente, 22cos .2

w ziiz iw w ze e e

+ − + =

Como ( ) ,ix iy iz iwe e e e+ = − + obtemos

Page 53: Eureka 2008

Sociedade Brasileira de Matemática

EUREKA! N°27, 2008

53

2 22cos 2cos .2 2

x y w zi iy x w ze e+ +

− − = −

Temos ainda 0

2 2y x π−

≤ ≤ e

02 2

w z π−≤ ≤ , donde cos 0

2y x− ≥

e cos 0.

2w z− ≥

Temos agora dois

casos:

i) cos 0.2

y x− =

Nesse caso, devemos ter também cos 0,2

w z− =

e,

portanto ,y x w z π− = − = sen seny x= − e sen sen ,w z= − donde segue imediatamente o resultado.

ii) cos 0.2

y x− >

Nesse caso, devemos ter cos cos 02 2

w z y x− − = >

e

2 2 ,x y w zi

e e+ +

= − donde

2 2w z x y π+ +

= + e .2 2

w z y x− −= Somando,

obtemos w y π= + e, substraindo, obtemos .z x π= + Assim, senw seny= − e senz senx= − , donde segue o resultado, como antes. 116) Seja ABC um triângulo e sejam X, Y e Z as reflexões de A, B e C em relação às retas BC, CA e AB, respectivamente. Prove que x, y e z são colineares se e somente se cos cos cos 3 8.A B C⋅ ⋅ = − SOLUÇÃO ENVIADA POR DOUGLAS RIBEIRO SILVA COM CONTRIBUIÇÕES DE CARLOS EDDY ESAGUY NEHAB E MARCIO COHEN Construam o triângulo ABC e as reflexões X, Y e Z de seus vértices A, B e C. Se, dados U, V, W no plano, (UVW) denota a área (orientada) do triângulo UVW, temos S(XYZ) = [S(ABC) + S(CBX) + S(ACY) + S(BAZ)] – S(AZY) – S(BXZ) – S(CYX). (*) Ver nota abaixo. Temos S(ABC) = S(CBX) = S(ACY) = S(BAZ), por construção. As áreas de AZY, BXZ e CYX podem ser somadas ou subtraídas, dependendo de os ângulos ˆ ˆ 3 , 3YAZ A ZBX B= = e ˆ 3XCY C= (onde A, B e C denotam os ângulos internos do triângulo ABC) serem maiores ou menores que 180 graus. Valerá a igualdade se usarmos as expressões S(AZY) = bc · sen(3A)/2, S(BXZ) = ac · sen(3B)/3 e S(CYX) = ab · sen(3C)/2. Então a relação passa a ser S(XYZ) = 4S(ABC) – bc · sen(3A)/2 – ac · sen(3B)/3 – ab · sen(3C)/2.

Page 54: Eureka 2008

Sociedade Brasileira de Matemática

EUREKA! N°27, 2008

54

Agora substituímos sen(3θ ) = – 4·(sen(θ )) 3 + 3 · sen(θ ) para θ = A, B, C e substituímos também bc/2 ac/2 e ab/2 respectivamente por S(ABC)/senA, S(ABC)/senB e S(ABC)/senC, devido à fórmula de área em função dos lados e do ângulo para o triângulo original. Fazendo as devidas substituições acima, simplificamos os senos e basta trocar (sen(θ ))2 por 1 – (cos(θ ))2 para θ = A, B, C para chegar em S(XYZ) = S(ABC) · [7 – 4((cosA)2 + (cosB)2 + (cosC)2)]. Para que os três pontos estejam alinhados, a área do triângulo XYZ deve ser igual a zero, donde 7 – 4((cosA)2 + (cosB)2 + (cosC)2) = 0 Façamos z = (cosA)2 + (cosB)2 + (cosC)2 Dai, como cos2x = 2(cosx)2 – 1 temos z = (1 + cos2A)/2 + (1+cos2B)/2 + (cosC)2 = 1 + (cos(2A) +cos(2B))/2 + (cosC)2. Mas cos(2A) + cos(2B) = 2cos(A + B)cos(A – B) = –2cosC cos(A – B). Substituindo em z: z = 1 – cosC [cos(A – B) – cosC] = 1 – cosC [cos(A – B) + cos(A + B)]. Daí temos: z = 1 – cosC. [2cosA.cosB] = 1 – 2cosA.cosB.cosC. Substituindo este z na expressão anterior, chegamos na desejada expressão do enunciado: 0 = 7 – 4 ((cosA)2 + (cosB)2 + (cosC)2) = 7 – 4 (1 – 2cosA.cosB.cosC) = 3+8cosA.cosB.cosC. Logo, chegamos na esperada relação cosA·cosB·cosC = –3/8. Nota: A igualdade(*) pode ser mostrada por meio de algumas figuras, considerando alguns casos, mas daremos a seguir uma prova algébrica dela. A área (orientada) de um triângulo UVW no plano 2 é igual à metade do determinante det (V – U , W– U) da matriz cujas linhas coincidem com os vetores V – U e W – U. Se identificarmos cada vetor 2( , )x y ∈ com 3( , ,0) ,x y ∈ o produto vetorial (V – U) × (W– U) é igual ao vetor (0,0, det (V – U, W – U)) = (0,0, 2 S (UVW)). Basta provar então que (Y – X) × (Z – X) = (B – A) × (C – A) + (B – C) × (X – C) + (C – A) × (Y – A) + (A – B) × (Z – B) – (Z – A) × (Y – A) – (X – B) × (Z – B) – (Y – C) × (X – C), mas o lado direito é igual a (B – A) × (C – A) + (B – Y) × (X – C) + × (C – Z) × (Y – A) + (A – X) × (Z – B), que, desenvolvendo (e usando a desigualdade U × V = – V × U, para quaisquer U, V), é igual a B × C – B × A – A × C + B × X – B × C – Y × X + Y × C + C × Y – C × A – Z × Y + Z × A + A × Z – A × B – X × Z + X × B = – Y × X – Z × Y – X × Z = Y × Z – Y × X – X × Z = (Y – X) × (Z – X).

Page 55: Eureka 2008

Sociedade Brasileira de Matemática

EUREKA! N°27, 2008

55

117) Sejam r e s duas retas reversas (i.e., não contidas num mesmo plano) e A, B, C, D, , , ,A B C D pontos tais que , , , , , , , ,A B A B r C D C D s∈ ∈ AB AB= e

.CD CD= Prove que os tetraedros ABCD e ABCD têm o mesmo volume. SOLUÇÃO DE DOUGLAS RIBEIRO SILVA (RECIFE - PE) Note que o que o problema pede é equivalente demonstrar que o volume do tetraedro só depende da medida de duas arestas reversas e da distância entre as retas-suporte dessas duas arestas. Na figura que segue, as arestas reversas são AB e CD. A distância entre as retas suporte é EF. O volume do tetraedro será calculado a partir da área da base ABC e a altura relativa a D V = (Área (ABC) · HD)/3 Note que a área de ABC pode ser definida como AB · EC/2, pois como EC está no mesmo plano de EF e EF é perpendicular a AB, EC também é (Teorema das 3 perpendiculares). V = ((AB · EC/2) · HD)/3 Conservando a área do triangulo retangulo EFC temos: EF · FC/2 = EC · FG/2. Logo FG = EF · FC/EC Pela semelhança dos triangulos FGC e DHC tiramos o valor de HD: HD/CD = FG/FC Logo HD = FG · CD/FC = (EF · FC/EC) · (CD/FC) Logo HD = EF · CD/EC Finalizando, temos que V = (AB · EC/2) · (EF · CD/EC)/3 Logo V = AB · CD · EF/6 Assim, provamos que o volume de um tetraedro não depende da posição dos segmentos AB e CD nas suas retas-suporte, mas sim, unicamente dos tamanhos dos segmentos e da distância entre as retas-suporte dos mesmos.

Page 56: Eureka 2008

Sociedade Brasileira de Matemática

EUREKA! N°27, 2008

56

D

F

G H

A

E

B

C

118) Considere a seqüência 1( )n na ≥ dada por 1 1a = e 12 9 , 1.3 9

nn

n

aa na+

+= ∀ ≥

+

Prove que ( )na converge e calcule o seu limite. SOLUÇÃO DE ESTILLAC LINS MACIEL BORGES FILHO (BELÉM – PA)

Primeiramente, iremos provar que 3

31+>na , para todo n∈ , utilizando

indução. De fato:

(1) 3

3111+

>=a

(2) Supondo 3

31+>na , temos 3231 +>+ na . Portanto:

331)

321(

311)

311(

311

9392

1+

=+

−>+

−=++

=+nn

nn aa

aa

Em seguida, provaremos que, se 3

31+>na , temos que nn aa <+1 . De fato:

3)13(3133

31 2 >−⇒>−⇒+

> nnn aaa

Page 57: Eureka 2008

Sociedade Brasileira de Matemática

EUREKA! N°27, 2008

57

Portanto,

093

)13(39392 2

1 <+

−−=−

++

=−+n

nn

n

nnn a

aa

aa

aa

Logo, )( na é uma seqüência estritamente decrescente limitada inferiormente por

331+

, o que garante sua convergência. Vamos mostrar agora que

331lim +

=na . Seja 1lim lim .n nx a a += = Temos então

12 9 2 9lim lim ,3 9 3 9

nn

n

a xx aa x+

+ += = =

+ +donde 29 6 2 0.x x− − = Como 0,x ≥ devemos ter

3 27 1 3 .9 3

x + += =

Obs: a solução acima é bastante artificial. Ela é construída já se sabendo de antemão qual o provável limite da seqüência. Este provável limite é obtido facilmente fazendo xaa nn ==+1 e resolvendo a equação

331

9392 +

=⇒++

= xxxx , pois 0.x ≥

Continuamos aguardando as soluções dos problemas a seguir: 113) 1 2 3, , ,...a a a formam uma seqüência de inteiros positivos menores que 2007

tais que m n

m n

a aa +

+ é inteiro, para quaisquer inteiros positivos m, n.

Prove que a seqüência (an) é periódica a partir de um certo ponto. 115) Suponha que ABC é um triângulo com lados inteiros a, b e c com

60BCA = ° e ( , ) ( , ) ( , ) 1.mdc a b mdc a c mdc b c= = = Prove que 1(mod 6)c ≡ .

Page 58: Eureka 2008

Sociedade Brasileira de Matemática

EUREKA! N°27, 2008

58

PROBLEMAS PROPOSTOS Convidamos o leitor a enviar soluções dos problemas propostos e sugestões de novos

problemas para próximos números. 119. Mostre que não existem inteiros positivos a e b tais que tais que (36 )(36 )a b b a+ + seja uma potência de 2. 120. Sejam a, b, c números reais e soma nS definida como ,n n n

nS a b c= + + para qualquer n inteiro não negativo, Sabe-se que 1 2,S = 2 6S = e 3 14,S = mostre que

21 1 8n n nS S S− +− ⋅ = para todo inteiro n > 1.

121. Na figura abaixo o lado do quadrado vale 4, obter o valor da altura h para que a área da região 1 seja igual a área da região 2.

h

4

4 4

4

1

2

122. Dado um triângulo ABC tal que AB AC a b= = + e BC a= , traça-se uma ceviana partindo de B determinando em AC um ponto D tal que DA a= e DC b= . Sabendo que 10ABD = ° , determine os ângulos internos desse triângulo. 123. Determine todas as funções : * *f → tais que

2 2 3 2 22 ( ) ( ) ( ) ( ) ( ) ,f m n f m f n f m f n+ = + para quaisquer , *m n∈ distintos. Obs. * {1, 2,3,...}= é o conjunto dos inteiros positivos.

Page 59: Eureka 2008

Sociedade Brasileira de Matemática

EUREKA! N°27, 2008

59

124. Considere a seqüência 1( )n na ≥ definida por 1 2 3 4 1a a a a= = = = e 2

1 3 2

4, 5.n n n

nn

a a aa n

a− − −

+= ∀ ≥

Prove que na é um inteiro positivo, para todo inteiro positivo n. 125. Considere dois naturais 2 m ≥ e 2,n ≥ e as seqüências

0 1 2( , , ,..., ), {0,1}.mn ia a a a a ∈ As seqüências de tipo m satisfazem as condições:

• 0,k k ma a + = para todo k; • Se 1 1k ka a + = então m divide k

As seqüências de tipo n são definidas analogamente. Prove que existem tantas seqüências do tipo m quanto do tipo n. 126. As circunferências ,0 5,i iΓ ≤ ≤ são tangentes a uma circunferência Γ nos pontos iA . Além disso, iΓ é tangente a 1i+Γ para 0 5i≤ ≤ e 5Γ é tangente a 0.Γ Prove que 0 3 1 4 2 5, ,A A A A A A são concorrentes. 127. Determine todos os inteiros positivos k tais que existem inteiros positivos x,

y, z com 2 2 2

.x y z kxyz

+ +=

128. Barango Joe era um sapo de mútiplos talentos que habitava a Terra das Chances Diminutas, localizada no alto de uma montanha. Após sua maioridade, Barango Joe decidiu tentar a vida no Reino das Grandes Oportunidades, localizado no cume da montanha vizinha. Para isso, ele atravessaria a extensa ponte de madeira por cima do Desfiladeiro da Morte. Entretanto, a ponte era guardada pela Esfinge Vegas, exímia jogadora que sempre desafiava os viajantes para algum jogo. O viajante vitorioso tinha a passagem franqueada; e o perdedor era lançado ao abismo. Assim chegando à cabeceira da ponte, Barango Joe foi desafiado a uma partida de “Pachang” jogo que lembra o “Black Jack” ou “Vinte e um”, mas é jogado por 2 oponentes da seguinte maneira: Os jogadores, designados por “banca” e “apostador”, utilizam um dado gerador de números aleatórios reais uniformemente distribuídos no intervalo [0,1] Inicialmente, a banca sorteia um número X. Se não estiver satisfeita com o número obtido, pode descartá-lo e então sortear um novo número. Este procedimento pode ser executado 2 vezes, Isto é, pode haver até 3 sorteios na definição do número X da banca.

Page 60: Eureka 2008

Sociedade Brasileira de Matemática

EUREKA! N°27, 2008

60

Então, o apostador sorteia quantos números forem necessários até que a soma de seus números ultrapasse o número X da banca. Neste momento, se esta soma for inferior a 1, o apostador ganha; caso contrário, perde. Ou seja, para ganhar, o apostador precisa “chegar mais próximo” de 1 que a banca, sem no entanto “estourar o limite” de 1. Após explicar as regras do Pachang, a Esfinge Vegas deu uma opção ao sapo: - Você prefere ser a banca ou o apostador? O que o Barango Joe deveria responder? Obs. Utilize lápis, papel, e uma calculadora científica simples. 129. Um coelho está numa rua infinita dividida em quadrados numerados pelos inteiros, e começa no quadrado 0. Se num dado momento ele está no quadrado k,

ele escolhe, com probabilidade 12

, pular para o quadrado k + 2 ou, também com

probabilidade 12

, pular para o quadrado k – 1. Ele continua esse processo

indefinidamante. Dado m∈ , determine a probabilidade de, em algum momento, o coelho pisar no quadrado m.

Problema 119 proposto por Adriano Carneiro, problemas 120 e 121 proposto por Samuel Liló Abdalla, de Sorocaba – SP, problema 122 proposto por Renan Lima Novais, do Rio de Janeiro – RJ, problema 123 proposto por Wilson Carlos da Silva Ramos, de Belém – PA, problemas 124, 125, 126 e 127 propostos por Anderson Torres, de São Paulo – SP, problema 128 proposto por Rogério Ponce da Silva, do Rio de Janeiro – RJ, problema 129 proposto por Nicolau Corção Saldanha e Zoroastro Azambuja Neto, do Rio de Janeiro – RJ. Agradecemos também o envio das soluções e a colaboração de: Gelly Whesley Fortaleza – CE Evandro A. dos Santos Campinas – SP Davi Lopes Alves de Medeiros Fortaleza – CE Rafael Alves da Ponte Fortaleza – CE André Felipe M da Silva Rio de Janeiro – RJ Carlos Alberto da Silva Victor Nilópolis – RJ

Page 61: Eureka 2008

Sociedade Brasileira de Matemática

EUREKA! N°27, 2008

61

AGENDA OLÍMPICA

XXX OLIMPÍADA BRASILEIRA DE MATEMÁTICA

NÍVEIS 1, 2 e 3 Primeira Fase – Sábado, 14 de junho de 2008

Segunda Fase – Sábado, 13 de setembro de 2008 Terceira Fase – Sábado, 25 de outubro de 2007 (níveis 1, 2 e 3)

Domingo, 26 de outubro de 2008 (níveis 2 e 3 - segundo dia de prova).

NÍVEL UNIVERSITÁRIO Primeira Fase – Sábado, 13 de setembro de 2008

Segunda Fase – Sábado, 25 e Domingo, 26 de outubro de 2008

XIV OLIMPÍADA DE MAIO 10 de maio de 2008

XIX OLIMPÍADA DE MATEMÁTICA DO CONE SUL Temuco – Chile

18 a 23 de junho de 2008

XLIX OLIMPÍADA INTERNACIONAL DE MATEMÁTICA 10 a 22 de julho de 2008

Madri – Espanha

XIV OLIMPÍADA INTERNACIONAL DE MATEMÁTICA UNIVERSITÁRIA 25 a 31 de julho de 2008

Blagoevgrad, Bulgária

XXIII OLIMPÍADA IBEROAMERICANA DE MATEMÁTICA 18 a 28 de setembro de 2008

Salvador, Bahia – Brasil ♦

XI OLIMPÍADA IBEROAMERICANA DE MATEMÁTICA UNIVERSITÁRIA

Page 62: Eureka 2008

Sociedade Brasileira de Matemática

EUREKA! N°27, 2008

62

COORDENADORES REGIONAIS

Alberto Hassen Raad (UFJF) Juiz de Fora – MG Américo López Gálvez (USP) Ribeirão Preto – SP Amarísio da Silva Araújo (UFV) Viçosa – MG Andreia Goldani FACOS Osório – RS Antonio Carlos Nogueira (UFU) Uberlândia – MG Ali Tahzibi (USP) São Carlos – SP Benedito Tadeu Vasconcelos Freire (UFRN) Natal – RN Carlos Alexandre Ribeiro Martins (Univ. Tec. Fed. de Paraná) Pato Branco - PR Carmen Vieira Mathias (UNIFRA) Santa María – RS Claus Haetinger (UNIVATES) Lajeado – RS Cleonor Crescêncio das Neves (Inst. de Tec. e Educ. Galileo da Amazônia) Manaus – AM Cláudio de Lima Vidal (UNESP) S.J. do Rio Preto – SP Denice Fontana Nisxota Menegais (UNIPAMPA) Bagé – RS Edson Roberto Abe (Colégio Objetivo de Campinas) Campinas – SP Élio Mega (Faculdade Etapa) São Paulo – SP Eudes Antonio da Costa (Univ. Federal do Tocantins) Arraias – TO Fábio Brochero Martínez (UFMG) Belo Horizonte – MG Florêncio Ferreira Guimarães Filho (UFES) Vitória – ES Francinildo Nobre Ferreira (UFSJ) São João del Rei – MG Genildo Alves Marinho (Centro Educacional Leonardo Da Vinci) Taguatingua – DF Ivanilde Fernandes Saad (UC. Dom Bosco) Campo Grande– MS Jacqueline Rojas Arancibia (UFPB)) João Pessoa – PB Janice T. Reichert (UNOCHAPECÓ) Chapecó – SC João Benício de Melo Neto (UFPI) Teresina – PI João Francisco Melo Libonati (Grupo Educacional Ideal) Belém – PA José Luiz Rosas Pinho (UFSC) Florianópolis – SC José Vieira Alves (UFPB) Campina Grande – PB José William Costa (Instituto Pueri Domus) Santo André – SP Krerley Oliveira (UFAL) Maceió – AL Licio Hernandes Bezerra (UFSC) Florianópolis – SC Luciano G. Monteiro de Castro (Sistema Elite de Ensino) Rio de Janeiro – RJ Luzinalva Miranda de Amorim (UFBA) Salvador – BA Mário Rocha Retamoso (UFRG) Rio Grande – RS Marcelo Rufino de Oliveira (Grupo Educacional Ideal) Belém – PA Marcelo Mendes (Colégio Farias Brito, Pré-vestibular) Fortaleza – CE Newman Simões (Cursinho CLQ Objetivo) Piracicaba – SP Nivaldo Costa Muniz (UFMA) São Luis – MA Osvaldo Germano do Rocio (U. Estadual de Maringá) Maringá – PR Raul Cintra de Negreiros Ribeiro (Colégio Anglo) Atibaia – SP Ronaldo Alves Garcia (UFGO) Goiânia – GO Rogério da Silva Ignácio (Col. Aplic. da UFPE) Recife – PE Reginaldo de Lima Pereira (Escola Técnica Federal de Roraima) Boa Vista – RR Reinaldo Gen Ichiro Arakaki (UNIFESP) SJ dos Campos – SP Ricardo Amorim (Centro Educacional Logos) Nova Iguaçu – RJ Sérgio Cláudio Ramos (IM-UFRGS) Porto Alegre – RS Seme Gebara Neto (UFMG) Belo Horizonte – MG Tadeu Ferreira Gomes (UEBA) Juazeiro – BA Tomás Menéndez Rodrigues (U. Federal de Rondônia) Porto Velho – RO Valdenberg Araújo da Silva (U. Federal de Sergipe) São Cristovão – SE Vânia Cristina Silva Rodrigues (U. Metodista de SP) S.B. do Campo – SP Wagner Pereira Lopes (CEFET – GO) Jataí – GO

Page 63: Eureka 2008

CONTEÚDO XXIX OLIMPÍADA BRASILEIRA DE MATEMÁTICA 2 Problemas e Soluções da Primeira Fase XXIX OLIMPÍADA BRASILEIRA DE MATEMÁTICA 15 Problemas e Soluções da Segunda Fase XXIX OLIMPÍADA BRASILEIRA DE MATEMÁTICA 35 Problemas e Soluções da Terceira Fase XXIX OLIMPÍADA BRASILEIRA DE MATEMÁTICA 57 Problemas e Soluções da Primeira Fase – Nível Universitário XXIX OLIMPÍADA BRASILEIRA DE MATEMÁTICA 62 Problemas e Soluções da Segunda Fase – Nível Universitário XXIX OLIMPÍADA BRASILEIRA DE MATEMÁTICA 73 Premiados AGENDA OLÍMPICA 77 COORDENADORES REGIONAIS 78

Page 64: Eureka 2008

Sociedade Brasileira de Matemática

EUREKA! N°28, 2008

2

XXIX OLIMPÍADA BRASILEIRA DE MATEMÁTICA Problemas e soluções da Primeira Fase

PROBLEMAS – NÍVEL 1 01) Observe as multiplicações a seguir:

101 11 1111101 111 11211101 1111 112211101 11111 1122211

× =× =× =× =

Qual é a soma dos algarismos do número obtido quando multiplicamos 101 pelo número 43421 …

1algarismos2007

1111111 ?

A) 1001 B) 2007 C) 2009 D) 4008 E) 4014

02) Quantos números inteiros positivos de três algarismos têm a soma de seus algarismos igual a 4? Observação: lembre-se de que zeros à esquerda não devem ser contados como algarismos; por exemplo, o número 031 tem dois algarismos. A) 4 B) 6 C) 7 D) 10 E) 12

03) Juntando dois retângulos iguais lado a lado, sem sobreposição, podemos formar dois tipos de figura: um quadrado de área igual a 144 cm2 ou um retângulo de largura diferente do comprimento. Qual é o perímetro deste último retângulo, em cm? A) 12 B) 24 C) 48 D) 60 E) 72 04) A figura ao lado é formada por dois quadrados de área 100 cm2 cada um, parcialmente sobrepostos, de modo que o perímetro da figura (linha mais grossa) é igual 50 cm. Qual é a área da região comum aos dois quadrados, em cm2 ? A) 20 B) 25 C) 30 D) 40 E) 50

Page 65: Eureka 2008

Sociedade Brasileira de Matemática

EUREKA! N°28, 2008

3

05) A soma de todos os números positivos ímpares até 2007 menos a soma de todos os números positivos pares até 2007 é igual a: A) 1003 B) 1004 C) 2005 D) 2006 E) 2007 06) Sílvia pensou que seu relógio estava atrasado 10 min e o acertou, mas na verdade o relógio estava adiantado 5 min. Cristina pensou que seu relógio estava adiantado 10 min e o acertou, mas na verdade o relógio estava atrasado 5 min. Logo depois, as duas se encontraram, quando o relógio de Sílvia marcava 10 horas. Neste momento, que horas o relógio de Cristina indicava? A) 9h 30min B) 9h 50min C) 10h D) 10h 5min E) 10h 15min

07) A fração ab

, onde a e b são inteiros

positivos, representa um número entre 0 e 1, na posição indicada no desenho ao lado. Qual é um possível valor para a soma ?a b+ A) 1 B) 2 C)3 D) 4 E) 5

0 1

ab

08) Em uma prova de olimpíada, 15% dos estudantes não resolveram nenhum problema, 25% resolveram pelo menos um problema, mas cometeram algum erro, e os restantes, 156 estudantes, resolveram todos os problemas corretamente. O número de estudantes que participaram da olimpíada foi: A) 200 B) 260 C) 93 D) 223 E) 300

09) Em uma certa cidade, a razão entre o número de homens e mulheres é 2 : 3 e entre o número de mulheres e crianças é 8 : 1. A razão entre o número de adultos e crianças é: A) 5 : 1 B) 16 : 1 C) 12 : 1 D) 40 : 3 E) 13 : 1 10) Na figura, o lado AB do triângulo eqüilátero ABC é paralelo ao lado DG do quadrado DEFG. Qual é o valor do ângulo x? A) 80o B) 90o C) 100o

D) 110o E) 120o

A

B C E

D

x F

G

Page 66: Eureka 2008

Sociedade Brasileira de Matemática

EUREKA! N°28, 2008

4

11) Uma loja de CD`s realizará uma liquidação e, para isso, o gerente pediu para Anderlaine multiplicar todos os preços dos CD`s por 0,68. Nessa liquidação, a loja está oferecendo um desconto de: A) 68% B) 6,8% C) 0,68% D) 3,2% E) 32%

12) Esmeralda e Pérola estão numa fila. Faltam 7 pessoas para serem atendidas antes de Pérola e há 6 pessoas depois de Esmeralda. Duas outras pessoas estão entre Esmeralda e Pérola. Dos números abaixo, qual pode ser o número de pessoas na fila? A) 9 B) 11 C) 13 D) 14 E) 15 13) Preenchemos as casas vazias da tabela ao lado com o produto dos números que estão sombreados na mesma linha e na mesma coluna da casa vazia a ser preenchida. Quantas dessas casas conterão números primos? A) 6 B) 7 C) 12 D) 14 E) 26

1 2 3 5 7 11 1312357

1113

x

14) O conteúdo de uma garrafa de refrigerantes enche três copos grandes iguais e mais meio copo pequeno ou 5 desses copos pequenos iguais mais a metade de um daqueles grandes. Qual é a razão entre o volume de um copo pequeno e o de um grande?

A) 25 B) 3

7 C) 710 D) 5

9 E) 35

15) Um código de barras é formado por barras verticais pretas de três larguras diferentes. Duas barras pretas sempre são separadas por uma barra branca, também com três larguras diferentes. O código começa e termina com uma barra preta, como no exemplo ao lado. Considere um código S, formado por uma barra preta fina, duas médias e uma grossa, separadas por barras brancas finas. Quantos códigos S diferentes podem ser assim formados? A) 4 B) 6 C) 12 D) 24 E) 36

Page 67: Eureka 2008

Sociedade Brasileira de Matemática

EUREKA! N°28, 2008

5

16) No quadriculado ao lado, cada quadradinho tem 1 cm2. Os segmentos inclinados ligam pontos médios dos lados dos quadradinhos ou um vértice ao centro de um quadradinho. Qual é a área ocupada pela sigla OBM, em cm2? A) 28 B) 32 C) 33 D) 34 E) 35

17) Lina e Lana brincam da seguinte maneira: a primeira a jogar pensa em um número de 10 a 99 e diz apenas a soma dos algarismos do número; a segunda tem então que adivinhar esse número. Qual é o maior número de tentativas erradas que a segunda pessoa pode fazer? A) 7 B) 8 C) 9 D) 10 E) 11 18) Anita imaginou que levaria 12 minutos para terminar a sua viagem, enquanto dirigia à velocidade constante de 80 km/h, numa certa rodovia. Para sua surpresa, levou 15 minutos. Com qual velocidade constante essa previsão teria se realizado? A) 90 km/h B) 95 km/h C) 100 km/h D) 110 km/h E) 120 km/h 19) O gráfico ao lado mostra o percentual de acertos numa prova de 60 testes de seis candidatos finalistas de um concurso. Qual foi o número médio de questões erradas por esses candidatos nessa prova? A) 14 B) 24 C) 30 D) 32 E) 40

A B C D E F

10%20%30%40%50%60%70%

20) Ao efetuar a soma 1 2 3 2006 200713 13 13 13 13+ + + + +L obtemos um número inteiro. Qual é o algarismo das unidades desse número? A) 1 B) 3 C) 5 D) 7 E) 9

Page 68: Eureka 2008

Sociedade Brasileira de Matemática

EUREKA! N°28, 2008

6

PROBLEMAS – NÍVEL 2 01) Veja o problema No. 1 do Nível 1. 02) Veja o problema No. 7 do Nível 1. 03) Veja o problema No. 10 do Nível 1. 04) Em uma certa cidade, a razão entre o número de homens e mulheres é 2 : 3 e entre o número de mulheres e crianças é 8 : 1. A razão entre o número de adultos e crianças é: A) 5 : 1 B) 16 : 1 C) 12 : 1 D) 40 : 3 E) 13 : 1

05) Veja o problema No. 8 do Nível 1. 06) Se N é o quadrado do quadrado de um número inteiro e tem 12 como fator,

o menor valor para 12N

é:

A) 3 B) 12 C) 36 D) 54 E) 108

07) O jardim da casa de Maria é formado por cinco quadrados de igual área e tem a forma da figura abaixo. Se AB = 10 m, então a área do jardim em metros quadrados é:

A

B

A) 200 B) 10 5 C) 100 D) 500

3 E)

1003

Page 69: Eureka 2008

Sociedade Brasileira de Matemática

EUREKA! N°28, 2008

7

08) Sejam , ,a b c e k números reais diferentes de zero satisfazendo as relações a b c

kb c c a a b

= = =+ + +

. Qual é o número de possíveis valores que k pode

assumir? A) 0 B) 1 C) 2 D) 3 E) 4

09) Doze pontos estão sobre um círculo. Quantos polígonos convexos podemos formar com vértices nesses 12 pontos? A) 4017 B) 220 C) 4095 D) 66 E) 3572 10) De quantas maneiras diferentes podemos escrever o número 2007 como soma de dois ou mais números inteiros positivos e consecutivos? A) 1 B) 2 C) 3 D) 4 E) 5

11) As equações do 2o grau 22007 2008 1 0x x+ + = e 2 2008 2007 0x x+ + = têm uma raiz comum. Qual é o valor do produto das duas raízes que não são comuns? A) 0 B) 1 C) 2007 D) 2008 E) 2007

12) Qual é o máximo valor que o número ( ) ( )a b c b a c+ − + pode assumir se

,a b e c , são inteiros satisfazendo 1 10a≤ ≤ , 1 10b≤ ≤ e 1 10c≤ ≤ ? A) 80 B) 81 C) 84 D) 90 E) 100 13) A quantidade de inteiros x com três dígitos tais que 6x e 7x possuem a mesma quantidade de dígitos é: A) 767 B) 875 C) 876 D) 974 E) 975 14) A figura abaixo é formada por três quadrados de lado 1 e um retângulo que os contorna.

A área do retângulo é: A) 3 2 B) 4 2 C) 6 D) 6 2 E) 8

Page 70: Eureka 2008

Sociedade Brasileira de Matemática

EUREKA! N°28, 2008

8

15) Se x é real positivo e 1 + (x2 + x)(x2 + 5x + 6) = 1812, então o valor de x(x + 3) é: A) 180 B) 150 C) 120 D) 182 E) 75

16) A figura abaixo mostra um retângulo, um pentágono, um triângulo e um círculo, com áreas respectivamente 121, 81, 49 e 25 centímetros quadrados. A diferença entre a área preta e a área cinza, em centímetros quadrados, é:

A) 25 B) 36 C) 49 D) 64 E) 81

17) As seguradoras de automóveis A e B cobram um valor anual (prêmio) mais um valor que o usuário deve pagar em caso de acidente (franquia). Jean quer fazer um seguro para seu automóvel e recebeu as seguintes propostas das seguradoras: Seguradora A: Prêmio anual de R$ 1500,00 e franquia de R$ 1400,00 Seguradora B: Prêmio anual de R$ 1700,00 e franquia de R$ 700,00 Para valer a pena Jean contratar a Seguradora A, ele não deve se acidentar com o carro por pelo menos N anos. O valor de N é: A) 2 B) 3 C) 4 D) 5 E) 6 18) O desenho abaixo mostra um dado comum cujas somas das pontuações em faces opostas é sempre igual a 7. Ele é colocado em uma mesa horizontal com a face “1” voltada para Leste. O dado é, então, movido quatro vezes.

Leste

Norte

Page 71: Eureka 2008

Sociedade Brasileira de Matemática

EUREKA! N°28, 2008

9

Um movimento consiste em uma rotação de °90 em relação a uma aresta. Depois do primeiro movimento a face em contato com a mesa passa a ser a “1”, depois a “2”, então a “3” e, finalmente, a face “5”. Para que sentido está voltada a face “1” após esta seqüência de movimentos? A) Oeste B) Leste C) Norte D) Sul E) Cima 19) Uma avenida possui 100 prédios numerados de 1 a 100, onde prédios com numeração par se situam do lado direito da rua e prédios com numeração ímpar se situam no lado esquerdo. A quantidade de andares de cada prédio é igual à soma dos algarismos do número correspondente ao prédio. Assim, podemos afirmar que: A) A quantidade de prédios com mais de 10 andares é maior do lado direito da rua. B) A quantidade de prédios com menos de 5 andares é maior do lado direito da rua. C) Pelo menos metade dos prédios possui 10 ou mais andares. D) Em ambos os lados da rua há a mesma quantidade de prédios com exatos 8 andares. E) Pelo menos 25% dos prédios possui menos de 5 andares. 20) Qual o menor perímetro inteiro possível de um triângulo que possui um dos

lados com medida igual a 2

35 ?

A) 8 B) 9 C) 10 D) 11 E)12 21) Determine em qual dos horários abaixo o ângulo determinado pelos ponteiros de um relógio é o menor. A) 02h30 B) 06h20 C) 05h40 D) 08h50 E) 09h55 22) O máximo divisor comum entre os números 1221, 2332, 3443, 4554,........, 8998 é: A) 3 B) 33 C) 37 D) 11 E) 101 23) Uma mesa de bilhar tem dimensões de 3 metros por 6 metros e tem caçapas nos seus quatro cantos P, Q, R e S. Quando uma bola bate na borda da mesa, sua trajetória forma um ângulo igual ao que a trajetória anterior formava.

Page 72: Eureka 2008

Sociedade Brasileira de Matemática

EUREKA! N°28, 2008

10

P

QR

S

Uma bola, inicialmente a 1 metro da caçapa P, é batida do lado SP em direção ao lado PQ, como mostra a figura. A quantos metros de P a bola acerta o lado PQ se a bola cai na caçapa S após duas batidas na borda da mesa?

A) 1 B) 76

C) 43

D) 32

E) 53

24) Considere todos os números abc de três algarismos onde b = a2 + c2 e a ≠ 0 . A diferença entre o maior e o menor destes números é um número: A) Múltiplo de 3 B) Primo C) Com último algarismo igual a 7 D) Cuja soma dos algarismos é 10 E) Múltiplo de 7 25) Seja {an} uma seqüência na qual cada termo é definido como o dobro da soma dos algarismos do termo anterior, mais uma unidade. Por exemplo, se an = 234, então 1+na = 2(2 + 3 + 4) +1. Se, a1 = 1 o valor de a31 + a32 + a33 + a34 + a35 é igual a: A) 44 B) 54 C) 64 D) 77 E) 84

PROBLEMAS – NÍVEL 3 01) A figura mostra dois quadrados sobrepostos. Qual é o valor de x + y, em graus?

x

y

A) 270 B) 300 C) 330 D) 360 E) 390

Page 73: Eureka 2008

Sociedade Brasileira de Matemática

EUREKA! N°28, 2008

11

02) Um número de quatro dígitos é dito peroba se possui pelo menos dois dígitos vizinhos com a mesma paridade. Quantos números perobas existem? A) 8999 B) 8874 C) 7875 D) 8000 E) 7750 03) Veja o problema No. 15 do Nível 2. 04) Veja o problema No. 18 do Nível 2. 05) Os números 72, 8, 24, 10, 5, 45, 36, 15 são agrupados em duplas de modo que o produto de cada dupla é o mesmo. Qual número fica com o 10? A) 36 B) 45 C) 24 D) 15 E) 72 06) Tintas pretas opacas absorvem 97% da luz, refletindo o restante. Cientistas desenvolveram uma nova cobertura superpreta que é “dez vezes mais preta” que tintas pretas opacas, querendo dizer que ela reflete 1/10 da luz refletida pelas tintas pretas opacas. Que porcentagem de luz a nova cobertura absorve? A) 9,7 B) 90,3 C) 99,7 D) 99,9 E) 970 07) Considere a seguinte seqüência:

33327 ××= , 2333207 ××= , 223332007 ××= , 22233320007 ××= , ... Qual dos seguintes inteiros é um múltiplo de 81? A) 200.007 B) 20.000.007 C) 2.000.000.007 D) 200.000.000.007 E) 20.000.000.000.007 08) Qual dos inteiros positivos abaixo satisfaz a seguinte equação:

309456654

4

4

4

4

4

4

444=

−+

−+

−++++

nn

nn

nn

nnnL ?

A) 2007 B) 309 C) 155 D) 25 E) 5 09) O desenho abaixo mostra um semicírculo e um triângulo isósceles de mesma área. Qual é o valor de tg °x ?

xo

Page 74: Eureka 2008

Sociedade Brasileira de Matemática

EUREKA! N°28, 2008

12

A) 1 B) 23

C) 3

π D)

π2

E) 2π

10) Um episódio muito conhecido na Matemática foi quando ao visitar o grande matemático Ramanujam no hospital, o outro grande matemático Hardy disse que o número do táxi que o trouxe, 1729, era um número sem graça; Ramanujam respondeu prontamente: “Não diga isso, Hardy! 1729 é o menor número inteiro positivo que pode ser escrito como soma de dois cubos perfeitos positivos de duas maneiras diferentes!” De fato, 1729 = 103 + 93 = 123 + 13. Um outro episódio não muito conhecido na Matemática foi quando o pequeno matemático Muralijam foi visitado pelo outro pequeno matemático Softy, que disse que o número do lotação que o trouxe era um número sem graça. Muralijam responde imediatamente: “Não, Softy, ele é o menor inteiro positivo que pode ser escrito como soma de dois quadrados perfeitos positivos de duas maneiras diferentes!” A que número Muralijam e Softy se referem? A) 18 B) 41 C) 45 D) 50 E) 65 11) Dizemos que uma palavra Q é quase-anagrama de outra palavra P quando Q pode ser obtida retirando-se uma letra de P e trocando a ordem das letras restantes, resultando em uma palavra com uma letra a menos do que P. Um quase-anagrama pode ter sentido em algum idioma ou não. Por exemplo, RARO, RACR e ARCO são quase-anagramas de CARRO. Quantos são os quase-anagramas da palavra BACANA que começam com A? A) 48 B) 60 C) 72 D) 96 E) 120 12) As cidades Aópolis, Beópolis e Ceópolis são ligadas por estradas retas. Sabe-se a estrada que liga Aópolis e Beópolis é perpendicular à estrada que liga Aópolis e Ceópolis. Rubens mora em Beópolis e tem um compromisso em Ceópolis. Todavia, a estrada que liga Beópolis a Ceópolis está interditada, de modo que Rubens é obrigado a fazer o trajeto Beópolis-Aópolis-Ceópolis. Para chegar ao compromisso na hora certa, Rubens trafega com uma velocidade 24% maior do que trafegaria se utilizasse a estrada interditada. Se α é o menor ângulo do triângulo determinado pelas três estradas, então

A) 0 < tgα < 61

B) 61

< tgα < 51

C) 51

< tgα < 41 D)

41

< tgα < 31

E) 31

< tgα < 1

Page 75: Eureka 2008

Sociedade Brasileira de Matemática

EUREKA! N°28, 2008

13

13) Todo número real a pode ser escrito de forma única como { }aaa += , em que a é inteiro e { } 10 <≤ a . Chamamos a parte inteira de a e { }a parte fracionária de a. Se { } 2,4=++ zyx , { } 6,3=++ xzy e { } 2=++ yxz , quanto vale x – y + z? A) –1 B) –0,5 C) 0 D) 0,5 E) 1 14) Dizemos que um natural X é um repunit quando os seus algarismos são todos iguais a 1, ou seja, quando X é da forma 11…1. Sejam p, q e r inteiros, 0>p , tais que rqXpX ++2 é um repunit sempre que X é um repunit. Qual dos valores a seguir é um possível valor de q? A) –2 B) –1 C) 0 D) 1 E) 2

15) O conjunto dos valores de c para os quais a equação cxx += possui solução real está contido em: A) [;1[ ∞− B) ]1;] ∞− C) ]2;3[− D) ]3;2[− E) Z 16) No triângulo ABC, AD é a altura relativa ao lado BC. Se AB = DC = 1, assinale a alternativa que corresponde à área máxima do triângulo ABC.

A) 8

33 B)

23

C) 32

D) 22

E) 21

17) O número de pares (x, y) de inteiros positivos que satisfazem a equação

3248 43 yxyx =+ , com 1≤ y ≤ 2007, é igual a: A) 40 B) 41 C) 42 D) 43 E) 44 18) Sejam a, b e c números tais que

a2 – ab = 1 b2 – bc = 1 c2 – ac = 1

O valor de abc⋅(a + b + c) é igual a: A) 0 B) 1 C) 2 D) −1 E) −3 19) Veja o problema No. 19 do Nível 2. 20) Veja o problema No. 20 do Nível 2. 21) Veja o problema No. 21 do Nível 2.

Page 76: Eureka 2008

Sociedade Brasileira de Matemática

EUREKA! N°28, 2008

14

22) O máximo divisor comum entre os números 1221, 2332, 3443, 4554,........, 8998 é: A) 3 B) 33 C) 37 D) 11 E) 101 23) Veja o problema No. 23 do Nível 2. 24) Veja o problema No. 24 do Nível 2. 25) Veja o problema No. 25 do Nível 2.

GABARITO NÍVEL 1 (5ª. e 6ª. Séries)

1) E 6) A 11) E 16) D 2) D 7) E 12) B 17) B 3) D 8) B 13) C 18) C 4) E 9) D 14) D 19) D 5) B 10) E 15) C 20) E

NÍVEL 2 (7ª. e 8ª. Séries)

1) E 6) E 11) B 16) D 21) E 2) E 7) C 12) D 17) B 22) D 3) E 8) C 13) C 18) A 23) B 4) D 9) A 14) C 19) B 24) Anulada 5) B 10) E 15) A 20) B 25) Anulada

NÍVEL 3 (Ensino Médio)

1) A 6) C 11) B 16) A 21) E 2) C 7) E 12) D 17) E 22) D 3) A 8) E 13) B 18) D 23) B 4) A 9) E 14) E 19) B 24) Anulada 5) A 10) D 15) A 20) B 25) D

Page 77: Eureka 2008

Sociedade Brasileira de Matemática

EUREKA! N°28, 2008

15

XXIX OLIMPÍADA BRASILEIRA DE MATEMÁTICA Problemas e Soluções da Segunda Fase

PROBLEMAS – Nível 1 PARTE A (Cada problema vale 5 pontos) 01. O número N = 1010010100101... contém somente os algarismos 0 e 1, de modo que o número de algarismos 0 entre dois algarismos 1 é um ou dois, alternadamente. O número N tem exatamente 101 algarismos. Qual é a soma de todos os algarismos do número N? 02. Uma folha de papel tem 20 cm de comprimento por 15 cm de largura. Dobramos essa folha ao meio, paralelamente à sua largura. Em seguida, dobramos a folha retangular dupla, de modo que dois vértices opostos coincidam. Ao desdobrar a folha, as marcas da segunda dobra dividem a folha em duas partes, conforme mostrado na figura ao lado. Qual é a área da parte escura, em cm2?

03. Observe as igualdades a seguir: 1 2 1 41 2 3 2 1 91 2 3 4 3 2 1 16

1 2 3 2006 2007 2006 3 2 1 A

+ + =+ + + + =+ + + + + + =

+ + + + + + + + + =M

L L

Qual é o valor de 2223

A ?

04. Uma folha retangular de cartolina foi cortada ao longo de sua diagonal. Num dos pedaços restantes, na forma de um triângulo retângulo, foram feitos dois cortes, paralelos aos lados menores, pelos meios desses lados. Ao final sobrou um retângulo de perímetro 129 cm. O desenho abaixo indica a seqüência de cortes.

Page 78: Eureka 2008

Sociedade Brasileira de Matemática

EUREKA! N°28, 2008

16

Em centímetros, qual era o perímetro da folha antes do corte? 05. Um reservatório cúbico internamente tem 2 metros de lado e contém água até a sua metade. Foram colocados no reservatório 25 blocos retangulares de madeira, que não absorvem água, de dimensões 20 30 160× × centímetros. Sabendo que 80% do volume de cada bloco permanece submerso na água, calcule, em centímetros, a altura atingida pela água, no reservatório. 06. A adição ao lado está incorreta. Entretanto, se substituirmos somente um certo algarismo a, toda vez que ele aparece, por um certo algarismo b, a conta fica correta. Qual é o valor de ?ba

PARTE B (Cada problema vale 10 pontos) PROBLEMA 1 A área do quadrado ABCD é 300 cm2. Na figura, M é ponto médio de CD e o ponto F pertence à reta BC. a) Qual é a área do triângulo ABF ? b) Qual é a área do triângulo ADF ?

M

PROBLEMA 2 Esmeralda comprou seis discos de ferro para usar num aparelho de ginástica. Esses discos têm massas 1, 2, 3, 4, 5 e 6 quilogramas, respectivamente. Esmeralda pode combiná-los e obter outras massas, como por exemplo: 1 disco de 2 kg + 1 disco de 6 kg = 8 kg. Qual a maior quantidade de massas diferentes que ela pode obter?

Page 79: Eureka 2008

Sociedade Brasileira de Matemática

EUREKA! N°28, 2008

17

PROBLEMA 3 Observe como o quadriculado ao lado é preenchido. a) Qual é a soma dos elementos da diagonal 9? b) Qual é o resto da divisão por 100 da soma dos elementos da diagonal 2007?

3

3

3

3

PROBLEMAS – Nível 2 PARTE A (Cada problema vale 4 pontos) 01. Ludmilson descobriu que o produto da idade que tinha há 55 anos atrás pela idade que terá daqui a 55 anos é igual ao cubo de um número primo. Qual é a idade atual de Ludmilson?

02. Sendo f(x) = 100x + 3, calcule o valor de )1(1010

)10()10(38

38

−−−−

fff .

03. Na figura abaixo temos um pentágono regular, um quadrado e um triângulo eqüilátero, todos com a mesma medida de lado.

Q

P

C

B

E R

A T S D

Determine a medida, em graus, do ângulo ∠QCE. 04. Um inteiro positivo K tem n algarismos e é igual a 2608.n. Determine a soma dos algarismos de K 05. Em 1949 o matemático indiano D. R. Kaprekar, inventou um processo conhecido como Operação de Kaprekar. Primeiramente escolha um número de

Page 80: Eureka 2008

Sociedade Brasileira de Matemática

EUREKA! N°28, 2008

18

quatro dígitos (não todos iguais), em seguida escreva a diferença entre o maior e o menor número que podem ser formados a partir de uma permutação dos dígitos do número inicial. Repetindo o processo com cada número assim obtido, obtemos uma seqüência. Por exemplo, se o primeiro número for 2007, o segundo será 7200 – 0027 = 7173. O terceiro será 7731 – 1377 = 6354. Começando com o número 1998, qual será o 2007-ésimo termo da seqüência? PROBLEMAS – Nível 2 PARTE B (Cada problema vale 10 pontos) PROBLEMA 1 O triângulo ABC é retângulo em B. Sejam I o centro da circunferência inscrita em ABC e O o ponto médio do lado AC. Se ∠AOI = 45°, quanto mede, em graus, o ângulo ∠ACB? PROBLEMA 2 Sejam α e β as raízes da equação quadrática (x – 2)(x – 3) + (x – 3)(x + 1) + (x + 1)(x – 2) = 0.

Determine o valor de 1 1 1

.( 1)( 1) ( 2)( 2) ( 3)( 3)α β α β α β

+ ++ + − − − −

PROBLEMA 3 a) Determine a quantidade de divisores do número N = 235 – 23. b) Mostre que para todo número natural n , n5 – n é múltiplo de 30. PROBLEMA 4 Um quadrado 4 × 4 é dividido em 16 quadrados unitários. Cada um dos 25 vértices desses quadrados deve ser colorido de vermelho ou azul. Ache o número de colorações diferentes tais que cada quadrado unitário possua exatamente dois vértices vermelhos. PROBLEMAS – Nível 3 PARTE A (Cada problema vale 4 pontos) 01. Quantos divisores positivos do número 123456 são menores que 2007?

Page 81: Eureka 2008

Sociedade Brasileira de Matemática

EUREKA! N°28, 2008

19

02. Considere o conjunto A dos pares ordenados (x;y) de reais não negativos tais que x + y = 2. Se a probabilidade de um elemento de A escolhido aleatoriamente estar a uma distância da origem menor ou igual a 5

3 é p, quanto vale 2535p2?

03. Qual é a soma dos algarismos do inteiro mais próximo de1000 uns

111...1?14243

04. Veja o problema 1 da parte B do nível 2. 05. Veja o problema 4 da parte B do nível 2. PROBLEMAS – Nível 3 PARTE B (Cada problema vale 10 pontos) PROBLEMA 1 Ache todos os pares (x, y) de inteiros positivos tais que

2(x + y) + xy = x2 + y2. PROBLEMA 2 Encontre todos os números n de seis algarismos da forma AAABBB, em que A e B são algarismos diferentes e não nulos e n + 1 é um quadrado perfeito. PROBLEMA 3 No quadrilátero convexo ABCD, ∠A + ∠B = 120°, AD = BC = 5 e AB = 8. Externamente ao lado CD, construímos o triângulo eqüilátero CDE. Calcule a área do triângulo ABE. PROBLEMA 4 Em um certo país há 21 cidades e o governo pretende construir n estradas (todas de mão dupla), sendo que cada estrada liga exatamente duas das cidades do país. Qual o menor valor de n para que, independente de como as estradas sejam construídas, seja possível viajar entre quaisquer duas cidades (passando, possivelmente, por cidades intermediárias)?

Page 82: Eureka 2008

Sociedade Brasileira de Matemática

EUREKA! N°28, 2008

20

Soluções Nível 1 – Segunda Fase – Parte A

Problema 01 02 03 04 05 06 Resposta 41 150 81 258 148 64 1. [41] O número é formado por blocos iguais, de 5 algarismos na forma

“10100”. Como o número tem 101 algarismos, concluímos que é formado por 20 desses blocos inteiros mais o primeiro algarismo de um bloco, que é 1. A soma dos algarismos de cada bloco é 1 + 0 +1 + 0 + 0 = 2, portanto a soma dos algarismos de N é 20 2 1 41× + = .

2. [150] O desenho abaixo à esquerda mostra como fica a folha após a primeira

dobra. À direita, mostra como fica a folha após as duas dobras.

Observamos que CE = EA e que CF = FA. Por uma propriedade da dobra, sabemos que o segmento FE é perpendicular ao segmento AC e esses segmentos se cruzam em seus pontos médios. Portanto, os quatro triângulos que compõem o quadrilátero AECF são congruentes; são congruentes também os triângulos EBC e FDA. Portanto, a dobra FE divide o retângulo ABCD em dois trapézios, EBCF e AEFD, de mesma área. Desdobrando inteiramente a folha, obtemos duas metades iguais. Portanto, a área do pentágono convexo BEFE’B’ é igual à área do pentágono não convexo AA’E’FE, ou seja, a área da parte escura é metade da área

da folha, portanto igual a 215 20150 cm

= .

Page 83: Eureka 2008

Sociedade Brasileira de Matemática

EUREKA! N°28, 2008

21

3. [81] Pelo padrão observado, as somas são iguais ao quadrado da parcela

central (aquela cujo número de parcelas à esquerda é igual ao número de parcelas à direita).

Portanto, 22007A = e, assim, 22

22 2

2007 20079 81

223 223 223A = = = =

.

4. [258] O retângulo que sobra após os cortes tem lados iguais às metades dos

lados da cartolina original, cujo perímetro, então, é o dobro do perímetro desse retângulo. Logo, o perímetro da cartolina antes do corte é 129 2 258× = cm.

5. [148] O volume de cada bloco de madeira é 30, 2 0,3 1,60 0,096 m× × = ; o

volume de cada bloco que fica submerso no líquido é 30,80 0,096 m× . O volume de líquido deslocado pelos 25 blocos é igual a

325 0,80 0,096 =1,92 m× × . Como o reservatório é um cubo de 2 m de lado, sua base é um quadrado de área 4 m2. Podemos pensar no líquido deslocado como se fosse um bloco cuja base é igual à base do reservatório, de altura h e volume acima.

Portanto 1,92

4 1,92 0, 48 m 48 cm4

h h= ⇔ = = = . Como a altura inicial

do líquido era 100 cm, a nova altura será 148 cm.

Page 84: Eureka 2008

Sociedade Brasileira de Matemática

EUREKA! N°28, 2008

22

6. [64] À primeira inspeção, podemos admitir que os três algarismos à direita de

todos os números estão corretos, isto é, estão corretamente escritos os algarismos 0, 1, 3, 4, 5, 6 e 8. Portanto, dentre os algarismos 2, 7 e 9, um deles está escrito incorretamente. O 9 está escrito corretamente, pois se o mudarmos, a soma com 2 não estará certa. Logo ou 2 ou 7 está errado. Se o 7 estiver errado, então 2 estará correto, mas isso não é possível pois a soma de 2 com 4 mais 1 não estaria certa. Logo, o 2 é que deve ser substituído; olhando novamente a soma de 2 com 4 mais 1 resultando 1 vemos que o resultado só dará certo se no lugar de 2 colocarmos 6. Fazendo a substituição, verificamos que o resto se encaixa. Teremos, então, 62 64ba = = .

Soluções Nível 1 – Segunda Fase – Parte B

1. Temos ˆ ˆ( ) ( )m FMC m AMD= (ângulos opostos pelo vértice), ˆˆ( ) ( )m ADM m FCM= (pois ABCD é quadrado, logo esses ângulos são

retos) e MC = MD (pois M é ponto médio de CD). Logo, os triângulos AMD e FMC são congruentes.

Page 85: Eureka 2008

Sociedade Brasileira de Matemática

EUREKA! N°28, 2008

23

a) Vemos que a ABCMFMCABF áreaáreaárea +∆=∆ .

Como AMDFMC ∆=∆ áreaárea , temos:

ABCMAMDABF áreaáreaárea +∆=∆ = área do quadrado ABCD = 300 cm2.

b) ADF AMD DMFárea área área∆ = ∆ + ∆ =área área áreaFMC DMF FCD= ∆ + ∆ = ∆

Como AD = FC, CD é lado comum e os ângulos ˆ ˆ e C D são retos, concluímos que os triângulos FCD e ADC são congruentes,

logoárea

área área 2ABCD

FCD ADC∆ = ∆ = . Portanto, a área do triângulo

ADF é igual a 2300150 cm

2= .

2. Dadas as massas de 1 a 6, podemos adicionar 1 a 6, 2 a 6, etc, até obter todos os pesos de 7 a 11; podemos adicionar 1 + 5 a 6, 2 + 5 a 6, etc, até obter todos os pesos de 12 a 15; podemos adicionar 1 + 4 + 5 a 6, etc, obtendo os pesos de 16 a 18; somando 1 + 3 + 4 + 5 a 6 obtemos 19; 2 + 3 + 4 + 5 a 6 obtemos 20 e, finalmente, somando 1 + 2 + 3 + 4 + 5 a 6 obtemos 21. Portanto, a quantidade de massas diferentes que Esmeralda pode obter é 21.

3. Pode-se concluir, examinando a tabela, que a soma dos elementos da

diagonal n é igual a 2n + (n – 1)k, onde k é o algarismo das unidades do número n. Por exemplo, na diagonal de número 4 a soma dos números é

Page 86: Eureka 2008

Sociedade Brasileira de Matemática

EUREKA! N°28, 2008

24

( )2 4 4 1 4 20⋅ + − ⋅ = , na diagonal de número 10 a soma dos números é

2 10 (10 1) 0 20⋅ + − ⋅ = , etc.

a) Na diagonal de número 9, a soma dos elementos é ( )2 9 9 1 9 90⋅ + − ⋅ = . De

outra forma, na diagonal 9 há 10 números 9; portanto a soma é 90910 =⋅ . b) Na diagonal 2007 a soma será ( )2 2007 2007 1 7 4014 14042 18056⋅ + − ⋅ = + = .

O resto da divisão desse número por 100 é 56. Soluções Nível 2 – Segunda Fase – Parte A

Problema 01 02 03 04 05 Resposta 66 197 174 8 6174 01. Seja x a idade de Ludmilson. Logo, 3( 55)( 55)x x p− + = , onde p é

primo. Temos então, duas possibilidades: i)

3

55 1

55

x

x p

− =

+ =

Nesse caso teríamos 56x = e 111p = , absurdo, pois 111 não é primo. ii)

2

55

55

x p

x p

− =

+ =

Com isso, 2110 ( 1) 11.10p p p p= − = − = . E assim teremos 11p = e 66x = . Logo, a idade de Ludmilson é 66 anos.

02. (100.10-8 + 3 – 100.10 3 – 3) / (10– 8 – 10 3) – 100.(–1) – 3 = 100(10–8 – 10 3) / (10–8 – 103) + 97 = 100 + 97 = 197. 03. Note que os triângulos PTA, ABD, BCE, e PQC são todos isósceles. Como ∠ STP = 108°, ∠ PTA = ∠ PAT = 72°. Assim, temos que ∠ TPA = 36° e ∠ BAD = ∠ BDA = 18°. Além disso, ∠ ABD = 144° e ∠ CBE = 66°. Como ∠ QPC = 126°, temos que ∠ QCP = 27° e ∠ ECB = 57°. Logo, ∠ QCE = 174°.

Page 87: Eureka 2008

Sociedade Brasileira de Matemática

EUREKA! N°28, 2008

25

04. Tente 1, 2, 3 ... e perceba que, somente com n = 5, K terá 5 algarismos. Assim, K = 2608 . 5 = 13040. Com isso, a soma dos algarismos de K é 8. 05. A partir do sétimo termo, todos serão iguais a 6174.

Soluções Nível 2 – Segunda Fase – Parte B SOLUÇÃO DO PROBLEMA 1:

Como ABC é um triângulo retângulo, então AO = BO = CO. Se

oAOIABI 45=∠=∠ e OAIBAI ∠=∠ , então ? ABI = ? AOI (ALA). Com isso, AB = AO = BO, e portanto, triângulo ABO é eqüilátero. Assim,

oACB 30=∠ . SOLUÇÃO DO PROBLEMA 2: É fácil ver que ( 2)( 3) ( 3)( 1) ( 1)( 2) 3( )( )x x x x x x x xα β− − + − + + + − = − − . Fazendo x = –1, 2 e 3, nesta igualdade, temos que,

( 1)( 1) 4α β+ + = , ( 2)( 2) 1α β− − = − , 4

( 3)( 3)3

α β− − = .

Com isso, 1 1 1 1 3

1 0.( 1)( 1) ( 2)( 2) ( 3)( 3) 4 4α β α β α β

+ + = − + =+ + − − − −

SOLUÇÃO DO PROBLEMA 3:

a) 4 2 2 223 (23 1) 23 (23 1)(23 1) 23 (23 1)(23 1)(23 1)N = ⋅ − = ⋅ + − = ⋅ + + − =

Page 88: Eureka 2008

Sociedade Brasileira de Matemática

EUREKA! N°28, 2008

26

523 530 24 22 2 3 5 11 23 53⋅ ⋅ ⋅ = ⋅ ⋅ ⋅ ⋅ ⋅ O número de divisores (positivos) de N é 6 2 2 2 2 2 192× × × × × = . b) 5 2( 1)( 1)( 1).N n n n n n n= − = + + − Necessariamente, n ou n + 1 é par. Logo, 2 divide N. Do mesmo modo, um dos números n– 1, n ou n + 1 é múltiplo de 3. Logo 3 também divide N. Finalmente, se nenhum dos 3 números n –1, n ou n + 1 é múltiplo de 5, então n é da forma 5k + 2 ou 5k + 3. No primeiro caso, temos 2 21 25 10 5n k k+ = + + e, no segundo,

2 21 25 15 10n k k+ = + + , ambos múltiplos de 5. Portanto, um dos números , 1, 1n n n− + ou 2 1n + é múltiplo de 5.

Assim N é, simultaneamente, múltiplo dos números primos entre si 2, 3 e 5, o que prova que N é múltiplo de 30. SOLUÇÃO DO PROBLEMA 4: Vamos começar colorindo a primeira linha de vértices. Cada coloração dessa linha é uma seqüência de letras “A” e “V”, por exemplo, A V V A V. Observe que, uma vez colorida a primeira linha, se aparecerem duas letras consecutivas iguais, o restante dos vértices do tabuleiro já estão determinados. De fato, ao aparecer dois V’s consecutivos, os dois vértices imediatamente abaixo deles deverão ser coloridos com dois A’s, os que estão mais abaixo deverão ter dois V’s, e assim por diante. Isto completa a coloração dessas duas colunas. Dessa forma, cada coluna vizinha também estará determinada, pois em cada retângulo teremos três vértices previamente coloridos, o que obriga o quarto vértice a ter sua cor determinada. Então, para cada seqüência de A’s e V’s na primeira linha que contém pelo menos duas letras iguais consecutivas, há exatamente uma maneira de colorir o tabuleiro. Como há 25 – 2 = 30 de tais seqüências, contamos 30 colorações possíveis.

A V V A V A A V V A A V V

Falta-nos analisar um segundo caso, em que não há duas letras consecutivas iguais na primeira linha. Há duas possibilidades de seqüências: começando com A ou começando com V.

Page 89: Eureka 2008

Sociedade Brasileira de Matemática

EUREKA! N°28, 2008

27

A V A V A V

Para cada uma dessas seqüências, há duas maneiras de escolhermos a primeira letra da segunda linha. Uma vez escolhida esta letra, a segunda linha inteira também estará determinada. Para a primeira letra da terceira linha também há 2 possibilidades. Com este raciocínio, cada vez que escolhemos a primeira letra de uma linha, determinamos a coloração desta linha. Logo, como há duas maneiras de escolhermos a primeira letra de cada linha, há 25 = 32 maneiras de colorirmos o tabuleiro, neste segundo caso. Logo, o total de colorações é igual a 30 + 32 = 62. Observação: Veja que, no caso geral, para um quadrado n × n, o raciocínio é análogo. No primeiro caso, teremos 2n + 1 – 2 colorações; no segundo caso, mais 2n + 1. Logo, teremos 2⋅2n+1 – 2 = 2n + 2 – 2 colorações.

Soluções Nível 3 – Segunda Fase – Parte A

Problema 01 02 03 04 05 Resposta 17 3024 1500 30 62

01. Seja a fatoração de 64332123456 6 ⋅⋅= e seja d um de seus divisores menores do que 2007. Podemos analisar dois casos:

- d não é múltiplo de 643: então d é um divisor de 2007192326 <=⋅ . Portanto podemos contar todos os divisores de 192, que são 14)11)(16( =++ divisores. - d é múltiplo de 643: 6436431 =⋅ , 12866432 =⋅ e 19296433 =⋅ são menores que 2007, mas a partir de 25726434 =⋅ , eles são maiores que 2007. Portanto há 3 divisores neste caso. Portanto o total de divisores d de 123456 menores do que 2007 é 14 + 3 = 17.

02. Seja B o conjunto dos pontos de A cuja distância à origem é menor do que 35

e seja P = );( yx um ponto de B. Sabe-se que P está sobre o segmento

Page 90: Eureka 2008

Sociedade Brasileira de Matemática

EUREKA! N°28, 2008

28

0,;2 ≥=+ yxyx e que a distância 22 yx + de P à origem é menor ou igual a

35 . Portanto:

( ) 09

1142

2

925

44

2

35

2222222 ≤+−

−=⇔

≤+−+

−=⇔

≤+

=+

xx

xy

xxx

xy

yx

yx

As raízes de 09

1142 2 =+− xx são

614

14

911

8164

0 ±=−±

=x , que nos dá os

pontos extremos

+−=

614

1;614

11P e

−+=

614

1;614

12P de B. Pela

inequação, temos que os pontos de B estão na reta 2=+ yx , delimitados pelos

pontos 1P e 2P , logo B é o segmento de reta 21PP . Queremos a probabilidade p de escolher um ponto do conjunto A estar contido no segmento 21PP , que é a razão entre 21PP e o comprimento de A. Como A está delimitado pelos pontos )2;0( e )0;2( , seu comprimento vale

22)02()20( 22 =−+− . O comprimento de B vale

372

314

2614

1614

1614

1614

1222

=

=

−−−+

+−+ , portanto

p614

223

72

== e 30247323614

3232 3455255 =⋅⋅=⋅⋅=p .

03. Inicialmente, temos 9

1109

999111

1000noves 1000

uns 1000

−==

876…

321… . Portanto

9110

1111000

uns 1000

−=43421 … .

Com isso, observando que

3

1109

)110)(110(9

)110)(110(9

110 5005005005005001000 −=

−−>

+−=

− e

Page 91: Eureka 2008

Sociedade Brasileira de Matemática

EUREKA! N°28, 2008

29

310

910

9110 50010001000

=<−

, temos 3

10111

3110 500

uns 1000

500

<<−

43421 … .

Como 3

110500 − é inteiro e seu consecutivo,

3210500 +

, é maior do que 3

10500

, o

inteiro mais próximo de 43421 …uns 1000

111 é {

500 noves

500

500 três

10 1 99 933 3

3 3−

= =

678… … , cuja soma dos

dígitos é 15005003 =⋅ . 04. Veja a solução do problema 1 da parte B do nível 2. 05. Veja a solução do problema 4 da parte B do nível 2. Soluções Nível 3 – Segunda Fase – Parte B SOLUÇÃO DO PROBLEMA 1: Uma solução: Multiplicando a equação dada por 2, obtemos 2x2 + 2y2 – 2xy – 4x – 4y = 0, ou ainda,

(x2 – 4x + 4) + (y2 – 4y + 4) + (x2 – 2xy + y2) = 8. Daí, (x – 2)2 + (y – 2)2 + (x – y)2 = 8. A única maneira de escrevermos 8 como a soma de três quadrados é 8 = 0 + 4 + 4, em alguma ordem. Logo (x – 2, y – 2) = (0, 2), (2, 0) ou (2, 2), de onde concluímos que as soluções são (x, y) = (2, 4), (4, 2) ou (4, 4). Outra solução: Escrevendo a equação dada como uma equação do segundo grau em x, temos:

x2 – (y + 2)x + (y2 – 2y) = 0. O discriminante desta equação é ∆ = (y + 2)2 – 4(y2 – 2y) = −3y2 + 12y + 4.

Resolvendo a inequação ∆ ≥ 0, ainda obtemos 3

342

334

2 +≤≤− y .

Como y é inteiro positivo, as únicas possibilidades são y = 1, 2, 3 ou 4. • Se y = 1, ficamos com ∆ = 13, que não é quadrado perfeito. Logo, este caso não tem solução.

Page 92: Eureka 2008

Sociedade Brasileira de Matemática

EUREKA! N°28, 2008

30

• Se y = 2, obtemos ∆ = 16 e =±

=2

44x 0 ou 4. Como x é inteiro positivo, a

única solução neste caso é (x, y) = (4, 2). • Se y = 3, ficamos com ∆ = 13, absurdo!

• Se y = 4, obtemos ∆ = 4. Neste caso, =±

=2

26x 2 ou 4. Logo, (x, y) = (2, 4) ou

(4, 4). Portanto, o conjunto solução é {(2, 4), (4, 2), (4, 4)}. Mais uma solução: Observe que 8(x + y) = 4x2 – 4xy + 4y2 = (x + y)2 + 3(x – y)2 ≥ (x + y)2, de modo que

8(x + y) ≥ (x + y)2, ou seja, x + y ≤ 8. Além disso, note que x2 – xy + y2 = 2(x + y) é par, e portanto ao menos uma das parcelas do primeiro membro é par (se todos forem ímpares, x2 – xy + y2 é ímpar), o que implica que x ou y é par. Suponha, sem perda de generalidade, que x é par. Então y2 = 2(x + y) + xy – x2 é par e, assim, y também é par. Logo, dos dois fatos acima, conclui-se que as únicas possibilidades para os pares (x, y) são (2, 2), (2, 4), (2, 6), (4, 2), (4, 4) e (6, 2). Substituindo os pares, vemos que as únicas soluções são (2, 4), (4, 2) e (4, 4). SOLUÇÃO DO PROBLEMA 2: Seja k inteiro positivo tal que k2 = n + 1. Primeiro, notemos que o algarismo das unidades dos quadrados perfeitos são 0, 1, 4, 5, 6 e 9, de modo que B é igual a 9, 3, 4, 5 ou 8. Porém, podemos eliminar alguns casos: • Se B = 9, pois nesse caso k2 = AAABBB + 1 terminaria com exatamente três zeros (note que A não pode ser igual a 9, pois é diferente de B); • Se B = 3, k2 terminaria com 34, e seria par e não múltiplo de 4, já que os dois últimos algarismos de todo múltiplo de 4 formam outro múltiplo de 4, um absurdo. • Se B = 4, k2 terminaria com 45, e seria múltiplo de 5 mas não de 25, já que os dois últimos algarismos de um múltiplo de 25 são 25, 50, 75 ou 00. Outro absurdo.

Page 93: Eureka 2008

Sociedade Brasileira de Matemática

EUREKA! N°28, 2008

31

Sobram somente os casos B = 5 e B = 8. Observe que n = k2 – 1 = (k – 1)(k + 1) = AAABBB = 111(1000A + B) é múltiplo de 373111 ⋅= e, portanto, os primos 3 e 37 dividem k + 1 ou k – 1, de modo que k é da forma 1111 ±x ou 38111 ±x . Além disso,

10003001000000111556 2 <<⇒<≤ kk , de modo que 93 ≤≤ x . • 1111 ±= xk :

Temos xxBAxxkAAABBB 211110002221111 2222 ±=+⇔±=−= . O dígito das unidades de 1000A + B é B. Note que 222 )55(22111 xxxxx +±=± tem a mesma paridade que x. Assim, se B = 5, x é ímpar, ou seja, é 3, 5, 7 ou 9. Se x = 3, 5, 7, 9, o algarismo das unidades de xx 2111 2 + é 5, 5, 3, 9, respectivamente, de modo que x = 3 ou x = 5, para o qual 1000A + B iguala 100569111 =+⋅ e

27851025111 =+⋅ , o que gera a solução x = 3, A = 1 e n = 111555. Além disso, se x = 3, 5, 7, 9, o algarismo das unidades de xx 2111 2 − é 3, 5, 5, 3, respectivamente, de modo que as únicas possibilidades são x = 5 ou x = 7, para os quais BA +1000 iguala 2765 e 54251449111 =−⋅ respectivamente, o que também não é possível. Se B = 8, x é par, ou seja, é 4, 6 ou 8. Se x = 4, 6, 8, o algarismo das unidades de

xx 2111 2 + é 4, 8, 0, respectivamente, de modo que obtemos x = 6 e 1000A + B =

40081236111 =+⋅ , ou seja, A = 4. Obtemos assim a solução n = 444888. Além disso, se x = 4, 6, 8, o algarismo das unidades de xx 2111 2 − é 8, 4, 8 respectivamente, de modo que obtemos x = 4 ou x = 8, para os quais 1000A + B igual a 1768816111 =−⋅ e 70881664111 =−⋅ , respectivamente, o que não é possível. • 38111 ±= xk : Temos 1383811121111 2222 −+⋅⋅±=−= xxkAAABBB

xx 76111111 22 ⋅±= 13761111000)1376111(1113937 22 +±=+⇔+±=⋅+ xxBAxx . Estudemos,

como no caso anterior, o dígito das unidades de 1376111 2 +± xx . Se B = 5, x é par, ou seja, é igual a 4, 6 ou 8. Se x = 4, 6, 8, o algarismo das unidades de

1376111 2 ++ xx é 3, 5, 5, respectivamente, de modo que x = 6 ou 8, para os quais 1000A + B iguala respectivamente 44651367636111 =+⋅+⋅ e

77251387664111 =+⋅+⋅ , nenhum dos dois gerando solução. Além disso, se

Page 94: Eureka 2008

Sociedade Brasileira de Matemática

EUREKA! N°28, 2008

32

x = 4, 6, 8, o algarismo das unidades de 1376111 2 +− xx é 5, 3, 9, respectivamente, de modo que x = 2 e 1000A + B igual a

14851347616111 =+⋅−⋅ , o que não é possível. Se B = 8, x é ímpar, ou seja, é igual a 3, 5, 7 ou 9. Se x = 3, 5, 7, 9 o algarismo das unidades de 1376111 2 ++ xx é 0, 8, 4, 8, respectivamente, de modo que x = 5 ou x = 9, para os quais 1000A + B = 31681357625111 =+⋅+⋅ e k = 1000389111 >+⋅ , o que não é possível. Além disso, se x = 3, 5, 7, 9 o algarismo das unidades de

1376111 2 +− xx é 4, 8, 0, 0, respectivamente, de modo que x = 5, para o qual 1000A + B = 24081357625111 =+⋅−⋅ , o que não é possível. Portanto os únicos números n que satisfazem o enunciado são 111555 e 444888. SOLUÇÃO DO PROBLEMA 3: Uma solução:

F

E

A

B

D C

Prolongue AD e BC até se encontrarem no ponto F. Veja que ∠AFB = 60° = ∠DEC. Com isso, o quadrilátero FECD é inscritível. Temos: (i) ∠FDE = ∠FCE = α ⇒ ∠ADE = ∠BCE = 180° − α. (ii) AD = BC e ED = EC. De (i) e (ii), concluímos que ∆ADE ≡ ∆BCE. Portanto, EA = EB.

Page 95: Eureka 2008

Sociedade Brasileira de Matemática

EUREKA! N°28, 2008

33

Além disso, ∠DEA = ∠CEB, de onde concluímos que ∠AEB = ∠DEC = 60°. Dessa forma, o triângulo ABE é eqüilátero de lado 8 e sua área é igual a

3164

382

= cm2.

Outra solução: Considere os pontos no plano complexo. Representaremos o número complexo correspondente ao ponto X com a letra correspondente minúscula x. Fixemos o ponto médio de AB como origem e sejam a = –4 e b = 4. Assim, sendo

BAD∠=α e ABC∠=β , ambos no sentido anti-horário, podemos encontrar as coordenadas de C e D:

)cis(54)cis()(85

ββ −−=⇔−−=− cbabc

αα cis54cis)(85

+−=⇔−=− dabad

Sendo 3

cisπ

ω = a raiz sexta da unidade e raiz da equação 012 =+− xx ,

iie

ie

ie

e

dccdedcde

343

2cis

32

cis534

32

cis3

2cis534

32

cis3

23

cis512

3124

32

cis3

cis5)1(4

cis54)cis(54)1()( 222

=

++

+−−=⇔

++

++−=⇔

++

+−−

+⋅=⇔

++

−−−+=⇔

−+−−=−=+−=⇔−=−

απ

απ

απ

παπ

απ

αππ

απ

βπ

ωω

αωωβωωωωωωω

Assim, o triângulo ABE, com pontos de coordenadas A = (–4, 0), B = (4, 0) e

)34,0(=E , é eqüilátero e tem área 3162

348=

⋅ cm2.

Page 96: Eureka 2008

Sociedade Brasileira de Matemática

EUREKA! N°28, 2008

34

SOLUÇÃO DO PROBLEMA 4: Escolha 20 das cidades do país. Ligando duas quaisquer delas por uma estrada,

utilizaremos 190219.20

220

==

estradas, e a cidade restante não poderá ser

alcançada de automóvel. Logo se deve construir pelo menos 191 estradas. Vamos mostrar que com essa quantidade é possível atingir nosso objetivo.

Suponha que n = 191, mas que seja possível dividir as cidades do país em dois grupos A e B, digamos com a e b cidades, respectivamente, de tal sorte que nenhuma cidade de A possa ser alcançada de automóvel a partir de qualquer

cidade de B. Então o número de estradas no país é no máximo

+

22ba

, de

modo que

+

22ba

≥ 191, ou ainda, (a2 + b2) – (a + b) ≥ 2⋅191 = 382.

Como a + b = 21, segue da inequação acima que a2 + b2 ≥ 282 + 21 = 403. Logo

ab = .192

4034412

)()( 222

=−

≤+−+ baba

Mas, como a + b = 21 e a e b são naturais, temos ab ≥ 1⋅20 = 20, uma contradição. Logo, se n = 191, sempre é possível viajar entre quaisquer duas cidades.

Page 97: Eureka 2008

Sociedade Brasileira de Matemática

EUREKA! N°28, 2008

35

XXIX OLIMPÍADA BRASILEIRA DE MATEMÁTICA Problemas e Soluções da Terceira Fase

PROBLEMAS – NÍVEL 1 PROBLEMA 1 Parte das casas de um quadriculado com o mesmo número de linhas (fileiras horizontais) e colunas (fileiras verticais) é pintada de preto, obedecendo ao padrão apresentado pelo desenho ao lado. a) Quantas casas serão pintadas num quadriculado com 14 linhas e 14 colunas, de acordo com esse padrão? b) Quantas linhas tem um quadriculado com 199 casas pintadas?

PROBLEMA 2 Uma sala quadrada com 81 m2 de área tem o seu piso inteiramente coberto por dois tapetes retangulares A e B, que não se superpõem, conforme mostrado na figura (1) abaixo. Em certo momento, o tapete B é deslocado, o tapete A é girado de 90o e colocado sobre o tapete B, conforme indicado na figura (2).

Sabendo que a área do tapete B é o dobro da área do tapete A, calcule a área da parte do piso que ficou descoberta. PROBLEMA 3 Em uma face de cada um de três cartões foi escrito um número inteiro positivo. Em seguida, os cartões foram colocados lado a lado sobre uma mesa, com a face numerada para baixo.

Page 98: Eureka 2008

Sociedade Brasileira de Matemática

EUREKA! N°28, 2008

36

Arnaldo, Bernaldo e Cernaldo sabem que:

I. Os números escritos nos cartões são todos diferentes. II. A soma dos três números é 13. III. Os números crescem da esquerda para a direita.

a) Considerando as condições I, II e III, escreva todas as possibilidades de numeração dos cartões. b) Agora é hora de descobrir os números que foram escritos nos cartões. Primeiramente, Arnaldo olha o número do primeiro cartão à esquerda e diz que não tem informações suficientes para descobrir os outros dois números sem levantar os outros cartões. Depois, Bernaldo levanta o último cartão à direita, olha o número e diz também que não consegue descobrir os dois números à esquerda, sem levantar todos os cartões. E o mesmo acontece com Cernaldo, que levanta o cartão do meio, olha seu número e afirma que não consegue descobrir os números nos outros dois cartões. Sabendo que todos ouvem o que os demais dizem, mas não vêem o cartão que o outro olhou, qual número está escrito no cartão do meio? PROBLEMA 4 Considere a tabela a seguir com quatro linhas (fileiras horizontais) e quatro colunas (fileiras verticais) a qual está preenchida com números naturais, ocorrendo repetições de números:

1 0 0 3 5 1 2 4 1 1 2 3 6 1 4 0

Ao somarmos os números de cada uma de suas linhas (L1, L2, L3 e L4) e colunas (C1, C2, C3 e C4) obtemos 8 números distintos: 3, 4, 7, 8, 10, 11, 12, 13. Veja:

C1 C2 C3 C4 Soma da linha

L1 1 0 0 3 4 L2 5 1 2 4 12 L3 1 1 2 3 7 L4 6 1 4 0 11

Soma da coluna 13 3 8 10

Page 99: Eureka 2008

Sociedade Brasileira de Matemática

EUREKA! N°28, 2008

37

Apresente, se for possível: a) uma tabela com 4 linhas e 4 colunas, formada por números naturais, podendo ocorrer repetições de números, na qual apareçam como somas de linhas ou colunas os números de 1 a 8. b) uma tabela com 8 linhas e 8 colunas, formada por números naturais, podendo ocorrer repetições de números, na qual apareçam como somas de linhas ou colunas os números de 1 a 16. c) uma tabela com 9 linhas e 9 colunas, formada por números naturais, podendo ocorrer repetições de números, na qual apareçam como somas de linhas ou colunas os números de 1 a 18. Atenção: caso seja impossível montar alguma tabela, você deve explicar porque. PROBLEMA 5 Sendo 43421 L43421 L

dois2007cincos2007

22222225555555A ×= , calcule a soma dos algarismos de

A9 × . Não se esqueça de justificar a sua resposta. PROBLEMAS – NÍVEL 2 PROBLEMA 1

Seja ABC um triângulo e O seu circuncentro. Seja ainda P a intersecção das retas BO e AC e S a circunferência circunscrita a AOP. Suponha que BO = AP e que a medida do arco OP em S que não contém A é 40°. Determine a medida do ângulo ∠OBC.

Obs: A circunferência circunscrita de um triângulo é a circunferência que passa pelos seus vértices e seu centro é chamado de circuncentro.

Page 100: Eureka 2008

Sociedade Brasileira de Matemática

EUREKA! N°28, 2008

38

PROBLEMA 2 Considere a tabela a seguir com quatro linhas (fileiras horizontais) e quatro colunas (fileiras verticais) a qual está preenchida com números naturais, ocorrendo repetições de números:

1 0 0 3 5 1 2 4 1 1 2 3 6 1 4 0

Ao somarmos cada uma de suas linhas (L1, L2, L3 e L4) e colunas (C1, C2, C3 e C4) obtemos 8 números distintos: 3, 4, 7, 8, 10, 11, 12, 13. Veja:

C1 C2 C3 C4 Soma da Linha

L1 1 0 0 3 4 L2 5 1 2 4 12 L3 1 1 2 3 7 L4 6 1 4 0 11

Soma da Coluna

13 3 8 10

Apresente, se for possível: a) uma tabela com 4 linhas e 4 colunas, formada por números naturais, podendo ocorrer repetições de números, na qual apareçam como somas de linhas ou colunas os números de 1 a 8. b) uma tabela com 8 linhas e 8 colunas, formada por números naturais, podendo ocorrer repetições de números, na qual apareçam como somas de linhas ou colunas os números de 1 a 16. c) uma tabela com 9 linhas e 9 colunas, formada por números naturais, podendo ocorrer repetições de números, na qual apareçam como somas de linhas ou colunas os números de 1 a 18. Atenção: caso seja impossível montar alguma tabela, você deve explicar porque. PROBLEMA 3

Mostre que existe um inteiro positivo a tal que 29 1

1aa

−−

tem pelo menos 2007

fatores primos distintos.

Page 101: Eureka 2008

Sociedade Brasileira de Matemática

EUREKA! N°28, 2008

39

SEGUNDO DIA PROBLEMA 4 Prove que não existem soluções inteiras e positivas para a equação

.133 2tnm =++ PROBLEMA 5 Seja ABC um triângulo retângulo isósceles. K e M são pontos sobre hipotenusa AB, com K entre A e M, e o ângulo ∠ KCM = 45o. Prove que AK2 + MB2 = KM2. PROBLEMA 6 Quadradinhos iguais estão arrumados formando um tabuleiro n × n. Ludmilson e Ednalva jogam o seguinte estranho jogo. Cada jogada de Ludmilson consiste em retirar 4 quadradinhos que formem um quadrado 2 × 2. Cada jogada de Ednalva consiste em retirar apenas 1 quadradinho. Ludmilson e Ednalva jogam alternadamente, sendo Ludmilson o primeiro a jogar. Quando Ludmilson não puder fazer sua jogada, então Ednalva fica com todas as peças restantes do tabuleiro. Ganha o jogo aquele que possuir mais quadradinhos no final. Diga se é possível que Ednalva ganhe o jogo, não importando como Ludmilson jogue, em cada um dos seguintes casos: a) n = 10. b) Caso geral (n qualquer). TERCEIRA FASE – NÍVEL 3 (Ensino Médio) PRIMEIRO DIA PROBLEMA 1 Seja f(x) = x2 + 2007x + 1. Prove que, para todo n inteiro positivo, a equação

0))))(((( vezes

=…43421 … xfffn

tem pelo menos uma solução real

PROBLEMA 2 Para quantos números inteiros c, 20072007 ≤≤− c , existe um inteiro x tal que x2 + c é múltiplo de 22007

Page 102: Eureka 2008

Sociedade Brasileira de Matemática

EUREKA! N°28, 2008

40

PROBLEMA 3 São dados n pontos no plano, os quais são os vértices de um polígono convexo. Prove que o conjunto das medidas dos lados e das diagonais do polígono tem pelo menos 2/n elementos distintos. Observação: x denota o maior número inteiro que não excede x. Por exemplo,

25,2 = , 33 = e 22,1 −=− . SEGUNDO DIA PROBLEMA 4 Arrumam-se 20072 quadradinhos iguais, formando um tabuleiro 20072007 × . Arnaldo e Bernaldo disputam o seguinte jogo: cada jogada de Arnaldo consiste em retirar 4 quadradinhos que formem um quadrado 22× . Cada jogada de Bernaldo consiste em retirar apenas 1 quadradinho. Os jogadores jogam alternadamente, sendo Arnaldo o primeiro a jogar. Quando Arnaldo não puder fazer sua jogada, Bernaldo fica com todas as peças restantes do tabuleiro. Ganha o jogo aquele que possuir mais quadradinhos no final. É possível que Bernaldo ganhe o jogo, não importando como Arnaldo jogue? PROBLEMA 5 Seja ABCD um quadrilátero convexo, P a interseção das retas AB e CD, Q a interseção das retas AD e BC e O a interseção das diagonais AC e BD. Prove que se ∠POQ é um ângulo reto então PO é bissetriz de ∠AOD e QO é bissetriz de ∠AOB. PROBLEMA 6 Dados números reais x1 < x2 < … < xn, suponha que todo número real ocorre no máximo duas vezes entre as diferenças xj – xi, com nji ≤<≤1 . Prove que há pelo menos 2/n números reais que ocorrem exatamente uma vez entre tais diferenças. Observação: caso você tenha se esquecido da prova de ontem, x denota o maior número inteiro que não excede x. Por exemplo, 25,2 = , 33 = e

22,1 −=− .

Page 103: Eureka 2008

Sociedade Brasileira de Matemática

EUREKA! N°28, 2008

41

SOLUÇÕES – NÍVEL 1 SOLUÇÃO DO PROBLEMA 1: LIARA GUINSBERG (SÃO PAULO – SP) Considerando a figura, conseguimos ver um padrão (de cima para abaixo e da esquerda para a direita). Número de quadrados pintados: 2 × 2 : 2 3 × 3 : 7 4 × 4 : 8 5 × 5 : 17 6 × 6 : 18 7 × 7 : 31 8 × 8 : 32 Podemos perceber que, do 3 × 3 (7 pintados) para o 4 × 4 (8 pintados) que o número aumentou 1 unidade pintada. O fato se deve à seqüência de quadrados pintados, do 2 × 2 para o 3 × 3, o número de quadrados pretos cresceu em 5 unidades enquanto o branco permaneceu igual, mas do 3 × 3 para o 4 × 4, o número de brancos aumentou 6, enquanto o preto somente 1. Em geral, se n é par, do n × n para o (n + 1) × (n + 1) o número de quadrados pretos cresce em 2n + 1 unidades, mas se n é ímpar cresce em apenas 1 unidade. Para o caso do quadrado n × n, com n par, como a quantidade de casas pretas é

igual à quantidade de casas brancas, a quantidade de casas pretas será 2

.2n

Para o

caso do quadrado n × n, com n ímpar, percebemos que, a quantidade de casas

pretas será 2( 1)

12

n +− (devido às descobertas anteriores). Com efeito, para n par,

( )22 22 1 1,

2 2nn

n+

+ + = − e , para n ímpar, 22( 1) 1

1 1 .2 2

n n+ + − + =

Usando estes fatos:

a) Num quadriculado de 14 × 14, usamos o padrão para pares: 214

2= número de

casas pretas 196

982

= = casas pretas.

b) Para descobrirmos quando o quadrado tem 199 casas pintadas, vamos testar os casos:

Page 104: Eureka 2008

Sociedade Brasileira de Matemática

EUREKA! N°28, 2008

42

Usando o padrão para n par, temos: 2

2199 398,2n

n= ⇔ = mas e equação não tem

solução inteira. Usando o padrão para n ímpar, vemos que:

2 2( 1) ( 1)1 199 200,

2 2n n+ +

− = ⇔ = achamos (n + 1) = 20, donde n = 19, portanto

o número de linhas será igual a 19. SOLUÇÃO DO PROBLEMA 2: CAROLINA RODRIGUES SILVA (FORTALEZA – CE)

B

A

(2)

B A

(1)

Na figura 1 chamamos a área de A de x e a de B de 2x. Teremos então 3x = 81 m2 e x = 27 m2, então a área de A = 27 m2 e seus lados são: 3 e 9; área de B = 54m2 e seus lados 6 e 9. Na figura 2, vemos que se juntarmos as áreas descobertas teremos como largura 3 e altura 9 – 3 = 6. Obtemos assim como área do piso que ficou descoberta o seguinte valor: 6 × 3 = 18m2. SOLUÇÃO DO PROBLEMA 3: FELIPE BELLIO DA NÓBREGA (RIO DE JANEIRO – RJ)

, , : números nos cartõesx y z vamos supor x y z< < 13x y z+ + =

a) 1 + 2 + 10 2 + 3 + 8 3 + 4 + 6 1 + 3 + 9 2 + 4 + 7 1 + 4 + 8 2 + 5 + 6 1 + 5 + 7 b) Quando Arnaldo olha, pode-se eliminar o 3 + 4 + 6, pois ele saberia, já que é o único que começa com 3.

Page 105: Eureka 2008

Sociedade Brasileira de Matemática

EUREKA! N°28, 2008

43

Quando Bernaldo olha, pode-se eliminar o 1 + 2 + 10, o 1 + 3 + 9 e o 2 + 5 + 6. O primeiro porque é o único que acaba com 10. O segundo com 9. E o último, já que não pode ser o 3 + 4 + 6 graças a Arnaldo é o único que acaba com 6. Quando Cernaldo olha, pode-se eliminar o 1 + 5 + 7 e o 2 + 3 + 8. Já que o 2 + 5 + 6 foi eliminado por Bernaldo, o 1 + 5 + 7 é o único com 5 no meio. E já que Bernaldo também eliminou o 1 + 3 + 9, o 2 + 3 + 8 é o único com 3 no meio. Resposta: Assim sobraram apenas o 1 + 4 + 8 e o 2 + 4 + 7. Então o 4 está no cartão do meio. SOLUÇÃO DO PROBLEMA 4: RAFAEL KAZUHIRO MIYAZAKI (SÃO PAULO – SP) a)

C1 C2 C3 C4 Soma da linha

L1 0 0 0 1 1 L2 5 0 0 0 5 L3 2 4 1 1 8 L4 0 2 2 0 4 Soma da coluna 7 6 3 2 b)

C1 C2 C3 C4 C5 C6 C7 C8 Soma da linha

L1 0 0 0 1 0 0 0 0 1 L2 0 0 0 0 4 0 1 0 5 L3 0 3 0 1 0 0 0 0 4 L4 3 0 2 0 0 0 3 0 8 L5 3 0 3 0 2 1 0 0 9 L6 0 0 3 0 1 0 2 7 13 L7 1 0 2 0 3 3 0 7 16 L8 0 0 0 0 1 11 0 0 12 Soma da coluna 7 3 10 2 11 15 6 14 c) Não é possível. Para que seja possível montar uma tabela, a soma das somas das colunas e das somas das linhas deve ser igual ao dobro da soma dos números ”internos”(números preenchendo a tabela, exceto os de soma). 1 + 2 + 3 +...+ 16 + 17 + 18 = 171

Page 106: Eureka 2008

Sociedade Brasileira de Matemática

EUREKA! N°28, 2008

44

2n = 171 ⇒ n =2

171, onde n é a soma dos números “internos” e estes devem ser

naturais, mas 2

171não é natural. Portanto não podemos montar a tabela pedida.

SOLUÇÃO DO PROBLEMA 5: SOLUÇÃO DA BANCA Observamos inicialmente que 9 5 2 9 10 90× × = × = 9 55 22 9 1210 10890× × = × = 9 555 222 9 123210 1108890× × = × = 9 5555 2222 9 12343210 111088890× × = × = 9 55555 22222 9 1234543210 11110888890× × = × = Isso nos leva a conjecturar que

2007 cincos 2007 dois 2006 uns 2006 oitos

9 9 555...555 222...22 111...1110888...8890A× = × × =14243 14243 14243 14243

Para mostrar que nossa conjectura é verdadeira, devemos garantir que, ao continuar as multiplicações acima, o padrão se repete. Digamos que você já tenha feito n multiplicações e tenha obtido

cincos dois -1 uns -1 oitos

9 555...555 222...22 111...1110888...8890.n n n n

× × =14243 14243 14243 14243 Então

1 cincos 1 dois cincos dois

9 555...555 222...22 9 555...55550 5 222...220 2n n n n+ +

× × = × + × + =

14243 14243 14243 14243

cincos dois cincos dois

9 555...55550 222...220 9 555...55550 2 9 5 222...220 9 5 2n n n n

× × + × × + × × + × × =14243 14243 14243 14243

cincos dois uns uns

9 555...5555 222...2200 9 111...111100 9 111...111100 90n n n n

× × + × + × + =14243 14243 14243 14243

1 uns 1 oitos noves

111...1110888...889000 2 999...9900 90n n n− −

+ × + =14243 14243 14243

1 uns 1 oitos 1 noves

111...1110888...889000 1999...9800 90n n n− − −

+ + =14243 14243 123

uns oitos

111...1110888...8890n n

14243 14243

Portanto, nossa conjectura é verdadeira. Logo, a soma dos algarismos de 9 × A é igual a 2006 1 2006 8 9 2006 9 9 2007 9 18063.× + × + = × + = × =

Page 107: Eureka 2008

Sociedade Brasileira de Matemática

EUREKA! N°28, 2008

45

SOLUÇÕES – NÍVEL 2 PROBLEMA 1: SOLUÇÃO DE HERMANO HENRIQUE DA SILVA (FORTALEZA – CE)

B

C A P

O

20°

30°

20° 30°

80° 100°

120° 80° 60°

Propriedade do circuncentro: Está a igual distância dos vértices! Como O é o circuncentro, ,AO BO AP= = logo APO∆ é isósceles e como o » 40 20 , 80 .OP OAP AOP APO= ° ⇒ ∠ = ° ∠ = ∠ = °

Daí, 100 , 20 , 60 .OPC OCP POC∠ = ° ∠ = ° ∠ = ° Logo 120 ,BOC∠ = ° mas BOC∆ é isosceles, daí 30 .OBC OCB∠ = ∠ = °

PROBLEMA 2: Veja a solução do problema No. 4 do Nível 1.

PROBLEMA 3: SOLUÇÃO DA BANCA Observe a seguinte fatoração

2 29 29 29

2

( ) 1 1 1.

1 1 1a a aa a a

− + −= ⋅

− + −

Sabemos que 29 28 27 261 ( 1)( ... 1)a a a a a a+ = + − + − − + e 29 28 271 ( 1)( ... 1).a a a a a− = − + + + + Dessa forma cada uma das frações 29 291 1

e 1 1

a aa a

+ −+ −

é inteira.

Além disso, se a for par, pelo lema de Euclides: 29 29 29( 1, 1) ( 1,2) 1.mdc a a mdc a+ − = + =

Page 108: Eureka 2008

Sociedade Brasileira de Matemática

EUREKA! N°28, 2008

46

Assim, 29 291 1

, 1.1 1

a amdc

a a + −

= + − Com isso, podemos concluir que, se a for

maior que 1, 2 29

2

( ) 11

aa

−−

possui pelo menos um divisor primo a mais do que

29 1.

1aa

−−

Portanto, o número 200723a = satisfaz às condições do problema.

PROBLEMA 4: SOLUÇÃO DE MATHEUS SECCO TORRES DA SILVA (RIO DE JANEIRO – RJ)

.133 2tnm =++ Sabe-se que todos os números da forma 3k são ímpares. Assim, 3 3m n+ é um número par obrigatoriamente. Logo, 3 3 1m n+ + é um ímpar. Sendo 2t um número ímpar, t também deve ser ímpar, então podemos escrever 2 1,t k= + onde k é inteiro positivo. Voltando à equação original, obtemos: 23 3 1 4 4 1; 3 3 4 ( 1).m n m nk k k k+ + = + + + = + Pelo princípio da Casa dos Pombos, k(k + 1) é um número par necessariamente, fazendo com que 4k (k + 1) seja múltiplo de 8. Devemos ter então 3 3 0 (mod8).m n+ ≡ Porém, 3 1 ou 3 (mod8)m ≡ e

3 1 ou 3 (mod8).n ≡ Assim, 3 3 0 (mod8)m n+ ≡ é um absurdo!

Por isso, a equação 23 3 1m n t+ + = não tem soluções nos inteiros positivos (c.q.d.)

PROBLEMA 5: SOLUÇÃO DE DEBORAH BARBOSA ALVES (SÃO PAULO – SP)

C 45°

45°

K

ß

a

M

45°

A

´B A≡ a

Page 109: Eureka 2008

Sociedade Brasileira de Matemática

EUREKA! N°28, 2008

47

Girando o AKC∆ em torno de C, até ´ ,A B≡ temos o ´ ´CK A∆ (ou ´ )CK B∆ em que ´ ´ 45 ,CA K CAK∠ = ∠ = ° então ´MBK∠ é reto. Sendo ACK α∠ = e ,MCB β∠ = ´ .BCK ACK α∠ = ∠ = Como ABC∠ é retângulo, e a hipotenusa é AB, ACB∠ é reto. Então

45 90 45 .α β α β+ + ° = ° ⇔ + = ° Como ´KC K C= ; ´ 45MCK KCMα β∠ = + = ° = ∠ e como MCK∆ e

´MCK∆ são congruentes (caso LAL) Então, todos os seus ladas e ângulos são iguais. Assim, ´ .KM K M=

ACK∆ é congruente com ´ ,́A CK∆ por construção. Então ´ ´ .́AK A K BK= =

´MK B∆ (ou ´ )́MK A∆ é retângulo. Então, pelo teorema de Pitágoras, temos:

2 2 2´ ´MB BK MK+ = e como ´ ´ ´BK A K AK= = e 2 2 2´ , .MK KM AK MB KM= + = PROBLEMA 6: BASEADA NA SOLUÇÃO DE JOÃO MENDES VASCONCELOS (FORTALEZA – CE)

a) Se n é par, dividimos o tabuleiro em 2

4n

quadrados 2 × 2. Em cada jogada,

Ludmilson retira um quadrado 2 × 2 desses em que dividimos o tabuleiro. Nas

primeiras 2

8n

jogadas, Ednalva retirou quadrados pertencentes a, no máximo,

2

8n

desses quadrados 2 × 2. Assim, se 2

1 ,8n

k − < no momento de Ludmilson

fazer a k-ésima jogada, foram tocados no máximo 2 2 2 2

18 8 8 4n n n n

k

− + < + =

desses quadrados 2 × 2, e portanto sobra algum desses quadrados para Ludmilson

retirar. Assim, Ludmilson consegue retirar pelo menos 2

8n

desses quadrados,

que contêm 2 2

48 2n n

quadrados 1 × 1, ficando com pelo menos a metade dos

quadradinhos do tabuleiro.

Page 110: Eureka 2008

Sociedade Brasileira de Matemática

EUREKA! N°28, 2008

48

Se n = 10, 2 210

4 4 13 52 ,8 2n

= ⋅ = >

e Ludmilson de fato ganha o jogo.

Obs.: x denota o menor inteiro que é maior ou igual a x

b) Para fazermos o caso geral, dividiremos em casos: Primeiro caso: n é par: Como vimos acima, Ludmilson consegue retirar pelo menos metade dos quadradinhos do tabuleiro, e logo Ednalva não consegue ganhar o jogo. Na verdade Ludmilson ganha se n for da forma 4k + 2 e o jogo empata se n for da forma 4k. Segundo caso: n é ímpar. Nós faremos uma pintura como segue: A cada duas linhas, uma ficará em branco e outra será pintada em um quadradinho sim e um não. Veja a figura para melhor compreensão:

. . .

. . .

. . .

M M M

n

n

Como n é ímpar, as linhas pintadas terão um quadradinho pintado a menos que os não pintados. Pelo mesmo motivo, o número de linhas pintadas será uma unidade menor que o de não pintadas. Isso garante que o número de casas pintadas seja mínimo e nós possamos ter ao mesmo tempo todos os quadrados 2 × 2 com uma casa pintada. Agora vamos contar o número de quadrados pintados:

Em cada linha pintada, nós temos 1

2n −

quadrados pintados.

Como são 1

2n −

linhas pintadas, o total de quadradinhos pintados será 2( 1)

.4

n −

Page 111: Eureka 2008

Sociedade Brasileira de Matemática

EUREKA! N°28, 2008

49

A estratégia de Ednalva se resume a retirar, a cada jogada, um quadradinho preto até que não reste mais nenhum. Percebemos também que a cada jogada de Ludmilson ele também retira um quadradinho preto obrigatoriamente, já que todos os quadrados 2 × 2 do tabuleiro estão pintados em uma casa.

Desse modo, após 2( 1)

8n −

jogadas de Ludmilson, e 2( 1)

8n −

jogadas de

Ednalva, são retiradas 2 2 2( 1) ( 1) ( 1)

8 8 4n n n − − −

+ =

casas pintadas, ou seja,

todas as casas pintadas, e Ludmilson não consegue mais jogar. Como, ao final,

Ludmilson tem ( )22 2 21( 1) 1 ( 1)

4 4 28 8 2 2 2

nn n n − − − ≤ + = + <

quadradinhos

(pois 3n ≥ nesse caso), Ednalva vence sempre nesse caso. SOLUÇÕES – NÍVEL 3 PROBLEMA 1: BASEADA NA SOLUÇÃO DE LEANDRO FARIAS MAIA (FORTALEZA – CE) Sejam 1( ) ( )f x f x= e para cada 1,n ≥ 1( ) ( ( )).n nf x f f x+ =

Sejam 21 2007 4,∆ = − 1

1

2007.

2x

− + ∆=

Temos 1( ) 0.f x = Vamos mostrar por indução que existe uma seqüência de reais

positivos ( )n∆ tal que, definindo 2007

,2n

nx

− + ∆= temos 1( ) ,n nf x x+ = para

todo n , donde 11( ) ( ) 0.n n

n nf x f x++ = =

Para isso, note que a maior raiz de 2 2007 1 nx x x+ + = é 12007,

2n+− + ∆

onde

2 21 2007 4 4 2007 4018 2 0,n n nx+∆ = − + = − + ∆ > c.q.d.

PROBLEMA 2: SOLUÇÃO DE RAMON MOREIRA NUNES (FORTALEZA – CE) Vamos provar que todo número da forma 8q + 1 é resíduo quadrático módulo 2n (usaremos no que segue a palavra resíduo significando resíduo quadrático) e que são os únicos resíduos ímpares para n maior ou igual a 3. Temos que 1 é o único resíduo ímpar módulo 8. De fato,

2(2 1) 4 ( 1) 1 1(mod8), .k k k k+ = + + ≡ ∀ ∈¢ Assim, se 3n ≥ então todo número ímpar que é resíduo módulo 2n é congruente a 1 módulo 8.

Page 112: Eureka 2008

Sociedade Brasileira de Matemática

EUREKA! N°28, 2008

50

Mostraremos, por indução que todo número da forma 8q + 1 é resíduo mod 2k, para todo 3.k ≥ Caso inicial k = 3: 8q +1 é resíduo mod 8 porque 1 é resíduo mod 8. Passo: Todo número da forma 8q + 1 é resíduo mod 2k; tome x dessa forma. Então, existe y ∈¢ com 2 (mod 2 ).ky x≡ Se 2 1(mod 2 ),ky x +≡ acabou. Senão,

2 12 (mod2 ),k ky x +≡ + e ( )21 2 2 2 2 12 2 2 2 (mod 2 ),k k k k ky y y y x− − ++ = + + ≡ + ≡

donde x é resíduo módulo 12 ,k + e concluímos a demonstração. Aprendemos a contar os números ímpares resíduos quadráticos. Como

2 20070(mod2 )x c+ ≡ para algum x é o mesmo que 2 2007(mod2 )c x− ≡ para algum x, queremos saber o número de c´s tais que – c é resíduo quadrático; bem, entre os ímpares temos: 2001,..., 2007.− + Quantos números temos entre eles? Como 2001 8(250) 1− =− − e 2007 = 8 ⋅ 251 – 1, temos 502 ímpares. Agora para os pares: é claro que c tem que ser múltiplo de 4, pois

2 2007 20(mod2 ) 0(mod4)x c x c+ ≡ ⇒ + ≡ , que só acontece para c múltiplo de 4. Bem, claro também que x deve ser par, ou seja, x = 2y; queremos

2 2007 2 20054 4 0(mod2 ) 0(mod2 );y d y d+ ≡ ⇔ + ≡ novamente, sabemos contar os y ímpares. Tínhamos os seguintes múltiplos de 4: – 2004, .., 2004; dividindo por 4, ficamos com: – 501, – 500, ..., 500, 501. Os ímpares da forma –8q – 1 são –497,..., 495; como –497 = – 8 (62) – 1 e 495 = –8 (–62) – 1, temos 125 ímpares aqui. Agora, seguindo o algoritmo, pegamos os múltiplos de 4: – 500,...,500 e vemos quais deles são simétricos de resíduos 2005mod2 . Dividindo por 4, vemos que eles correspondem aos elementos de – 125,...,125 que são simétricos de resíduos mod 22003. Encontramos agora os números –8q – 1 entre esses: Veja que – 121 = –8 (15) – 1 e 119 = –8(–15) – 1; são 31 números aqui. Múltiplos de 4: – 124, ..., 124, dividindo por 4: – 31,..., 31 (mod 22001 agora) – 25 = –8 (3) – 1 e 31 = –8(–4) –1; temos 8 números aqui. Múltiplos de 4: – 28, ..., 28, dividindo por 4: –7,..., 7 (módulo 21999 agora). –1 = –8(0) – 1 e 7 = –8(–1) –1; temos 2 números aqui. Múltiplos de 4: –4, 0, 4, dividindo: –1, 0, 1 (mod 21997 agora); desses números somente –(–1) e –(0) (1 e 0) são resíduos, –(1) não é, logo temos 2 números aqui. Total: 502 + 125 + 31 + 8 + 2 + 2 = 670 números.

Page 113: Eureka 2008

Sociedade Brasileira de Matemática

EUREKA! N°28, 2008

51

PROBLEMA 3: SOLUÇÃO DA BANCA Primeiro considere dois pontos P e Q do polígono cuja distância é máxima. Tome Q de modo que PQ separe o polígono em dois polígonos, um deles com PQ como única distância máxima. Em cada um desses dois polígonos vamos aplicar o seguinte Lema: Seja 1 2... kA A A um polígono convexo tal que a maior distância entre dois de seus vértices, incluindo diagonais, é 1 .kA A Então esse polígono tem k – 2 distâncias diferentes; caso 1 kA A seja a única distância máxima, então há k – 1 distâncias diferentes. Demonstração: Sejam pA e ,qA 1 p q k< < < dois vértices do polígono. Vamos

provar que, para quaisquer m e n com p m n q< ≤ < um dos segmentos ,p n q mA A A A

é menor do que .p qA A Em seguida, conseguiremos uma seqüência de k – 2

distâncias diferentes. Como conseguir distâncias menores? Ou, de modo mais geral, como compara segmentos? Muitas vezes é melhor transferir tudo para ângulos, para que possamos fazer...isso mesmo, um arrastão! Sejam 1 1 1 2 3, , , ,m k m k p m q q p mA A A A A A A A A A A A Aα α α α= ∠ = ∠ = ∠ = ∠ a interseção de

p qA A e 1 mA A (note que, como o polígono é convexo, A está no interior do

segmento p qA A ) e 4 .m qA AAα =∠

A1 Ak

An

Am

Ap

α

α4 Aq B

A

ß4

Suponha que .p q m qA A A A≤ Então, no triângulo ,m p qA A A 2 3.α α≤ Além disso, pelo

teorema do ângulo externo no triângulo 3 4, .p mAA A α α< Ademais, 1 2α α< e, sendo

Page 114: Eureka 2008

Sociedade Brasileira de Matemática

EUREKA! N°28, 2008

52

1 kA A a maior distância de todas (e esse é o passo decisivo da demonstração e mostra o poder do princípio do extremo), no triângulo 1 1, .m kA A A α α< Logo

1 2 3 4 4α α α α α α α< < ≤ < ⇒ < . Definindo os ´sβ analogamente e supondo que ,p q n pA A A A≤ obtemos 4.β β<

Porém, observando os quadriláteros 1 k n mA A A A e ,n mABA A temos que

1 4 4 4 4360 .m n k n m m n n mA A A A A A AA A BA Aα β α β α β α β+ +∠ +∠ = + +∠ +∠ = °⇒ + = + Mas

44 4

4

,α α

α β α ββ β

<⇒ + < +

<

contradição. O caso em que m = n fica a cargo do leitor. Para terminar, basta fazer uma espécie de “zigue-zague”. Comece com 2 1,kA A − que é menor do que 1 kA A (por quê?). Pelo que acabamos de provar, 2 2kA A − ou

3 1kA A − é menor do que 2 1.kA A − Suponha, por exemplo, que 3 1kA A − seja menor. Então, aplicando o nosso fato de novo, 4 1kA A − ou 3 2kA A − é menor do que 3 1.kA A − Continuamos assim, até acabar o polígono, e assim conseguimos k – 2 distâncias diferentes. No caso em que 1 kA A é a única distância máxima, fica para você provar (use o poder do arrastrão novamente!) que, no quadrilátero 1 2 1 ,k kA A A A− uma das diagonais (na verdade as duas) é menor do que 1 kA A (bem, isso é imediato) e maior do que

2 1kA A − , de modo que ganhamos mais uma distância, totalizando k– 1. Agora, vamos terminar o problema. Lembre que cortamos o polígono original do problema em dois por uma diagonal PQ com medida máxima. Suponha que os polígonos obtidos tenham k + 1 e n – k + 1 lados, sendo que o de k + 1 lados tem a distância máxima única. Nele, obtemos (k + 1) – 1 = k distâncias diferentes, e no outro, (n – k + 1) – 2 = n – k – 1. Então conseguimos { , 1}d máx k n k= − −

distâncias. Mas ( 1) 1

.2 2 2

k n k n nd d

+ − − − ≥ = ⇒ ≥

PROBLEMA 4: SOLUÇÃO DE HENRIQUE PONDÉ DE OLIVEIRA PINTO (SALVADOR – BA) Numeremos as casas do tabuleiro de acordo com o seguinte padrão:

Page 115: Eureka 2008

Sociedade Brasileira de Matemática

EUREKA! N°28, 2008

53

A

C

A

C

A

B A B

D C D

B B A

D C D

B A B

A

C

A

C

B

D

B

D

A B

C D C D C D

É fácil ver que cada quadrado 2 × 2 de Arnaldo ocupa exatamente uma casa de cada tipo (A; B; C e D). Agora uma contagem simples nos mostra a quantidade de casas de cada tipo.

A

B

C

D

Tipo Quantidade

10042

1004 ⋅ 1003

1003 ⋅ 1004

10032

Veja que a soma total é de fato 20072. Veja que as casas tipo D são as menos numerosas. Agora suponha que Bernaldo só jogue em casas tipo D. Teremos que a cada jogada de cada um dos jogadores exatamente uma casa tipo D é ocupada. Assim após 10032 jogadas Arnaldo não

poderá mais jogar. Como Arnaldo começa, quando ele fizer sua 21003 1

2+

-ésima

jogada acabarão as casas D ̧ então Bernaldo pode escolher qualquer casa que

Arnaldo não poderá jogar novamente. Então assim Arnaldo terá 2(1003 1)

42

+⋅

casas. Ele só ganha se pegar mais que 22007

2 ou seja

2 22 24(1003 1) 2007

2006 4 2007 .2 2

+> ⇒ + > Absurdo.

Logo jogando assim Bernaldo ganha independentemente de como Arnaldo jogar. PROBLEMA 5: SOLUÇÃO DE RAFAEL TUPYNAMBÁ DUTRA (BELO HORIZONTE - MG) Sejam E a interseção de OC e PQ e F a interseção de BD e PQ.

Page 116: Eureka 2008

Sociedade Brasileira de Matemática

EUREKA! N°28, 2008

54

F

B

Q E P

D O

A

C

Pelo teorema de Ceva aplicado ao triângulo CPQ, 1=⋅⋅CBQB

PDCD

QEPE

.

Pelo teorema de Menelaus aplicado ao triângulo CPQ, 1=⋅⋅CBQB

PDCD

QFPF

.

Assim, temos QFPF

QEPE

= e, portanto, P, Q, E, F formam uma quádrupla

harmônica. Assim sendo, OP, OQ, OE, OF formam um feixe harmônico. Portanto qualquer reta intersecta esse feixe em uma quádrupla harmônica. Vamos criar uma reta r que passa por Q e é perpendicular a OQ.

' ∞ P

F

F ’ E ’ Q

E

P O

r

Supondo PÔQ = 90o, provaremos que OQ é bissetriz de FÔE, o que mostra que OQ é bissetriz de AÔB. Analogamente, teremos OP bissetriz de AÔD (as duas bissetrizes das retas AC e BD são perpendiculares). Como PÔQ = 90o, temos OP // r. Assim, r intersecta o feixe harmônico na quádrupla harmônica '

∞P , Q, F’, E’, sendo '∞P o ponto do infinito

correspondente ao feixe de retas paralelas a r. Dessa forma, precisamos ter QE’ = QF’, ou seja, Q é o ponto médio de E’F’. Assim, pelo teorema de Pitágoras,

temos OF’ = OE’ = 22 'QEOQ + e, como o triângulo OE’F’ é isósceles, a altura OQ também é bissetriz de E’ÔF’, de EÔF e de AÔB, como queríamos demonstrar.

Page 117: Eureka 2008

Sociedade Brasileira de Matemática

EUREKA! N°28, 2008

55

Observação: a maioria das soluções utilizou trigonometria ou geometria analítica, eventualmente com algumas aplicações dos teoremas de Menelaus e de Ceva. A demonstração de Rafael é bastante interessante por explorar o potencial da Geometria Projetiva, evitando cálculos. Veja a edição 8 da Eureka! para ver a teoria utilizada nesse problema. PROBLEMA 6: SOLUÇÃO DE RÉGIS PRADO BARBOSA (FORTALEZA - CE) Seja },{ njixxA iji ≤<−= . Note que, se ,a b≠ ibia xxxx −≠− pois

ba xx ≠ ; assim, inAi −= .

Considere agora mk AA ∩ com k > m. Se 2≥∩ mk AA , então existem a, b, c, d

distintos tais que mkbambka xxxxxxxx −=−⇔−=− e

mkdcmdkc xxxxxxxx −=−⇔−=− . Assim, mkdcba xxxxxx −=−=− , ou seja, um real aparece três vezes como diferença, um absurdo. Logo

1≤∩ mk AA . Vamos contar os reais que aparecem duas vezes do seguinte modo: se ele pertence a kA e mA , k m> , a contagem é registrada na linha de kA (ou seja, no conjunto de maior índice). Façamos então tal contagem, começando de nA e indo até 1A . Isto quer dizer que se o número aparece outra vez em outro conjunto, ele o faz em um conjunto de índice menor. Para n par:

Índice Quantidade de elementos Reais que aparecem duas vezes n 0=nA 0

1−n 11 =−nA 1≤

M M M

12

+n

1212/ −=+n

An 12

−≤n

2n

22/n

An = 12

−≤n

12

−n

1212/ +=−n

An 22

−≤n

M M M 2 22 −= nA 1≤

1 11 −= nA 0

Page 118: Eureka 2008

Sociedade Brasileira de Matemática

EUREKA! N°28, 2008

56

Justificando a contagem acima: note que há kAk ≤−1 conjuntos com índice

menor do que k. Como 1≤∩ mk AA para m < k, há no máximo k – 1 números que podem se repetidos nos conjuntos de índice menor; ou seja, a quantidade de novos números de kA que aparecem duas vezes é menor ou igual a k – 1; os outros podem aparecer duas vezes, mas eles já foram contados nos conjuntos de índice maior. Além disso, a quantidade de números de kA que aparecem duas

vezes é menor ou igual à quantidade total de elementos de kA . Logo a quantidade de novos números que aparecem duas vezes é no máximo

{ }1,min −kAk .

Com isso, a quantidade de números que aparecem duas vezes é menor ou igual a ( )

42

2

121

23212

222 nnn nn −

=⋅−

⋅=

−++++ L .

A quantidade dos números que aparecem uma vez pode ser obtida tomando o

total de elementos 2

)1(

1

−=∑

=

nnA

n

ii e subtraindo dele duas vezes a quantidade

de números que aparecem duas vezes. Sendo 1d a quantidade de números que

aparecem uma vez e 2d a quantidade de números que aparecem duas vezes,

então

==

−⋅−

−≥−

−=

2242

22

)1(2

2)1( 2

21nnnnnn

dnn

d .

Analogamente, para n ímpar, prova-se que

=

−≥

221

1nn

d .

Nota dos editores: Régis fez o estudo completo do caso n ímpar; porém, o procedimento é totalmente análogo e foi decidido não colocá-lo aqui.

Page 119: Eureka 2008

Sociedade Brasileira de Matemática

EUREKA! N°28, 2008

57

XXIX OLIMPÍADA BRASILEIRA DE MATEMÁTICA Problemas e Soluções da Primeira Fase – Nível Universitário

PROBLEMA 1: Joãozinho joga repetidamente uma moeda comum e honesta. Quando a moeda dá cara ele ganha 1 ponto, quando dá coroa ele ganha 2 pontos. Encontre a probabilidade (em função de n) de que Joãozinho em algum momento tenha exatamente n pontos. PROBLEMA 2:

Dados números reais 1 2, ,..., na a a não todos nulos, encontre o (menor) período da função

1

( ) cos( ).n

kk

f x a kx=

= ∑

PROBLEMA 3: Calcule o volume do sólido definido pelas desigualdades abaixo:

2 2 2 2 23 2 , 3 2 5 1z x y x y z≥ + + + ≤ PROBLEMA 4: Seja a um inteiro não nulo. Prove que se a é uma n-ésima potência modulo 4a2, ou seja, existe um inteiro b tal que a − bn é múltiplo de 4a2, então a é uma n-ésima potência. PROBLEMA 5: Calcule os autovalores da matriz (n + 1) × (n + 1) abaixo:

01 0 1

2 00 1

0

nn

M

n

− =

OO

Em outras palavras, , 1 1 ,i iM n i+ = + − 1, ,i iM i+ = 0ijM = se 1.i j− ≠

Obs: Os autovalores de M são as raízes da seguinte equação em x: det(M − xI) = 0.

Page 120: Eureka 2008

Sociedade Brasileira de Matemática

EUREKA! N°28, 2008

58

PROBLEMA 6: Seja y(t) uma função real de variável real tal que

2

´́ ( ) (́ ) 3 ( ) 2sen( ) tg( ),ty t e y t ty t t t+ + = + (0) 1,y = (́0) 0.y = Calcule o limite:

0

(́ )lim .

( ) 1t

ty ty t→ −

Soluções Nível Universitário SOLUÇÃO DO PROBLEMA 1:

Seja np a probabilidade pedida. Claramente 0 1

11, .

2p p= =

A probabilidade de que ele nunca tenha n pontos é 1 .np− Por outro lado, a única forma de nunca ter n pontos é completar n – 1 pontos e depois tirar coroa. Assim:

112n

n

pp −− =

donde 1

1

2 1 1 23 2 3 2 3

nn n

pp p−

− − = − = −

e portanto

0

2 2 13 3 2

n

np p − = − ⋅ −

2 1 13 3 2

n

np = + −

.

SOLUÇÃO DO PROBLEMA 2:

Seja { }0 .km mdc k a= ≠

Claramente 2mπ

é um período de f: afirmamos que este é o menor período.

Escreva 1

( ) ,2 2

nk k ixk k

k

a af x z z z e−

=

= + =

Page 121: Eureka 2008

Sociedade Brasileira de Matemática

EUREKA! N°28, 2008

59

1

( ) ,2 2

k knk k ipk k

k

a w a wf x p z z w e

−−

=

+ = + =

Duas funções racionais só são iguais (ou iguais para números complexos de módulo 1) se seus coeficientes forem iguais. Assim, se p é um período, temos

kk ka w a= para 1,..., .k n= Em outras palavras ou 0ka = ou .

2pk

π∈¢ .

Equivalentemente, p deve ser um múltiplo inteiro de 2

.mπ

SOLUÇÃO DO PROBLEMA 3:

Seja ( )A a a área da elipse 2 23 2 .x y a+ ≤

Os semieixos da elipse são 2a

e 3a

donde ( )6

aA a

π= .

O sólido do problema pode ser descrito como a união disjunta de

2 2 21 13 2 ,0 ,

10x y z z b b

−+ ≤ ≤ < =

2 2 2 13 2 1 5 ,

5z y z b z+ ≤ − ≤ ≤

donde 1

250

( ) (1 5 )b

bv A z dz A z dz= + −∫ ∫ ( )

125

01 5

6 6

b

b

zdz z dz

π π= + −∫ ∫

231 1 5

2 36 5 3 5

bb b

π = + − − +

31 2 5 7 21300 15 1006

π = + −

SOLUÇÃO DO PROBLEMA 4:

Suponha a uma n-ésima potência mod 24a

Escreva 322 3 ... ...peeea p= ⋅ ⋅ ⋅

Vamos provar que o expoente pe é múltiplo de n.

Segue da hipótese que pea b p= ⋅ é n-ésima potência módulo 2 pep onde mdc(b,

p) = 1.

Page 122: Eureka 2008

Sociedade Brasileira de Matemática

EUREKA! N°28, 2008

60

Assim existem c, d com mdc(c, p) = 1, 2(mod )p pe en ndc p b p p≡ ⋅ donde .pnd e= Assim a é uma n-ésima

potência. Falta provar que se n é par então a > 0. Suponha por absurdo o contrário: n par, a < 0.

Escreva 2nda b= − % , b% ímpar, 0.b >%

Assim a e – a são ambos n-ésimas potências módulo 22 :nd + 2 2

2 2

2 2 (mod 2 )

2 2 (mod 2 )

n nd nd nd

n nd nd nd

c b

c b

+

+

⋅ ≡ − ⋅

⋅ ≡ ⋅

%% % ⇒

2

2

(mod 2 )

(mod 2 )

n nd

n nd

c b

c b

+

+

≡ −

%% % ⇒ b% e b−% são

quadrados módulo 4 ⇒ –1 é quadrado módulo 4 ⇒ Absurdo! SOLUÇÃO DO PROBLEMA 5:

Os autovalores são , 2, 4,..., 2, ,n n n n n− − − + − ou seja, 2k n− para 0,1,..., .k n=

Vamos exibir os autovetores de .tM

Interprete o vetor ( ) 10 1, ,..., n

na a a +∈ ¡ como o polinômio 1

0 1 ... .n n nnP a x a x y a y−= + + +

O polinômio correspondente a 0( ,..., )tnM a a é .

p py x

x y∂ ∂

+∂ ∂ Se expandirmos

os polinômios em u x y= + e v x y= − este operador passa a ser

.p p

u vu v

∂ ∂−

∂ ∂

Mas ( ) ( )2 .k n k k n ku v u v k n u vu v

− −∂ ∂ − = − ∂ ∂

Assim este é o autovetor associado ao autovalor (2k – n). SOLUÇÃO DO PROBLEMA 6:

Expanda as funções y, 2te e 2 ( ) ( )sen t tg t+ em series de potências:

20 1 2( ) ...y t a a t a t= + + +

Page 123: Eureka 2008

Sociedade Brasileira de Matemática

EUREKA! N°28, 2008

61

2 20 1 2 ...te b b t b t= + + +

20 1 22 ...sent tgt c c t c t+ = + + +

Temos

21 2

20 01 2

2 ... ...´lim lim

1 ... ...

nnnt t

n

a t a t na ttyy a t a t a t→ →

+ + + +=

− + + + +

donde este limite é igual a N se 0Na ≠ e 0ma = para 0 .m N< < Substituindo as séries de potências na EDO:

2 3 2

20 1 1 1 0 2 1 1 2 0 1 0 1 0 1

21 0 1 2

2 6 ... ( 1)( 2) ...

( 2 ) ... ( 2 ... ... ( 1) ) 3 3 ...

3 ... ... ... donde

nn

nn n k k n

n nn n

a at n n a t

ba ba ba t b a b a kb a n ba t a t at

a t c ct c t c t

+

− + − +

+ + + + + +

+ + + + + + + + + + + + + + +

+ + = + + + + +

( )2 1 1 2 0 1 1

12 ... ( 1) 3

( 1)( 2)n n n n n na c b a b a n b a an n+ − + −= − − − − + −

+ +

Segue facilmente que 1 2 3 0.a a a= = =

Se 1 2 1... 0na a a += = = = e 0nc = temos 2 0na + = e se

1 2 1... 0na a a += = = = e 0nc ≠ temos 2 0.na + ≠ Devemos portanto procurar °N tal que ° 0

Nc ≠ e 0mc = para °1 m N< < .

Temos 3 5

...6 120t t

sent t= − + +

3 52tan ...

3 15t t

t t= + + +

532 tan 3 ...

20t

sent t t+ = + +

Assim ° 5N = donde 0

´lim 7

1t

tyy→

=− .

Page 124: Eureka 2008

Sociedade Brasileira de Matemática

EUREKA! N°28, 2008

62

XXIX OLIMPÍADA BRASILEIRA DE MATEMÁTICA Problemas e Soluções da Segunda Fase – Nível Universitário

PRIMEIRO DIA PROBLEMA 1: Considere a função de R em R dada por f(x) = ax2 + bx + c, com , ,a b c ∈¡ e ac < 0. Prove que, para todo n inteiro positivo, a equação 0))))((((

vezes

=…43421 … xfffn

tem

pelo menos uma solução real. PROBLEMA 2: Dado um inteiro positivo n, mostre que existe um inteiro positivo N com a seguinte propriedade: se A é um subconjunto de },,2,1{ N… com pelo menos N/2 elementos, então existe um inteiro positivo nNm −≤ tal que

2},,2,1{

kkmmmA ≥+++∩ …

para todo k = 1, 2, …, n. PROBLEMA 3: Considere o conjunto nP dos polinômios mônicos de grau n > 0 e coeficientes

complexos 01

1)( axaxxp nn

n +++= −− L satisfazendo

121

21

20 =+++ −naaa L .

Para nPxp ∈)( , seja ( ))(xpr o máximo entre os módulos das raízes de p(x) e

( ))(sup)()(

xprnsnPxp ∈

= .

Determine )(lim ns

n ∞→.

SEGUNDO DIA PROBLEMA 4:

Seja :f →¡ ¡ uma função contínua tal que xexff =))(( para todo x ∈¡ . Prove que, para todo n inteiro positivo,

Page 125: Eureka 2008

Sociedade Brasileira de Matemática

EUREKA! N°28, 2008

63

( )lim nx

f xx→+∞

= +∞ .

PROBLEMA 5: Seja A uma matriz real quadrada simétrica de ordem n, e 1 2 nλ λ λ≤ ≤ ⋅⋅⋅ ≤ seus autovalores (contados com multiplicidade). Determine, em função de

1 2, , , nλ λ λ⋅ ⋅ ⋅ :

a) O número de matrizes reais B simétricas de ordem n tais que 2B A= . b) O número de matrizes reais B de ordem n tais que 2B A= . PROBLEMA 6: Para ,a b ∈¤ , definimos o conjunto

2 2( , ) { | , }S a b ax by x y= + ∈¤

dos números racionais que podem ser escritos na forma 22 byax + com ,x y ∈¤ . Dados a, b, c, d racionais não nulos, mostre que S(a,b) = S(c,d) se, e

somente se, cdab

é o quadrado de um racional e existe um racional não nulo

),(),( dcSbaSq ∩∈ , SOLUÇÕES PROBLEMA 1: SOLUÇÃO DE MAURÍCIO RODRIGUES COLLARES NETO (ARACAJU – SE) 1 ) Provemos a afirmação por indução. Para n = 1 o discriminante da equação do segundo grau 2 4b ac− é positivo (pois

0)ac < e, portanto, a equação possui raiz real. Suponhamos agora a afirmação válida para n. Para provar a afirmação para n + 1, vejamos que

1 vezes

( (...( ( ))...))n

f f f x+

14243 é um polinômio cujo coeficiente do termo líder

é uma potência ímpar de a (para n = 1 isto é verdade; se isto é válido para n, temos que 2 1 2 1 2 2 1

1 vezes

(...( ( ))) ( ...) ( ...) ( ...)r k r k r k

n

f f x f a x a a x b a x c+ + +

+

= + = + + + +123

que possui termo líder igual a 2kx com coeficiente 4 3).ra + Assim, se

0(resp. 0),a a< >1vezes

lim ( (...( ( )) (resp. )x

n

f f f x→+∞

+

= −∞ + ∞14243 (basta colocar o termo

líder em evidência e verificar que os outros termos vão a zero). Mas

Page 126: Eureka 2008

Sociedade Brasileira de Matemática

EUREKA! N°28, 2008

64

vezes

( (...( ( ))...))n

f f f x14243 tem uma raiz r, por hipótese, e

1 vezes

( (...( ( ))...)) (0) 0 (resp. < 0). n

f f f r f c+

= = >14243 (Pois ac < 0). Assim, pelo

Teorema do Valor Intermediário, como a função passa de um valor positivo (resp. negativo) para um valor negativo (resp. positivo) e é contínua, ela tem raiz real: PROBLEMA 2: SOLUÇÃO ADAPTADA DA SOLUÇÃO DE RAFAEL DAIGO HIRAMA (S.J. DOS CAMPOS – SP)

Fixe N. Para cada 1 ,i N≤ ≤ defina {1,..., } .ia A i= ∩

Temos

{ 1,..., } 0.2 2 2m k m m k m

k m k mA m m k a a a a+ +

+ ∩ + + = − ≥ ⇔ − − − ≥

Isso nos induz a definir ,2i i

ib a= − e portanto a desigualdade anterior equivale a

.m k mb b+ ≥ Assim, queremos mostrar que, se N é suficientemente grande, então

existe m N n≤ − tal que

1,..., .m m m nb b b+ +≤ (I) É claro que

1

1 1 1 1 1( 1) ,

2 2 2 2 2i i i i i

i i ib a a a b+

+ + +− = − ≤ − ≤ + − = + ou seja:

1

1 1, , .

2 2i i i ib b b b+ ∈ − +

Tome ( 2)N n n> + e suponha que (I) não ocorra para cada .m N n≤ − Em

particular, para m = 1, existe 1 {2,..., 1}i n∈ + tal que 1 11 1

1 1.

2 2i ib b b b< ⇒ ≤ − <

Por indução, construímos uma seqüência 1 1,..., ni i + de índices tal que

1 { 1,..., }j j ji i i n+ ∈ + + e 1

.j ji ib b+

< Podemos fazer isso pois ( 1).N n n n− > +

Daí, 1 1

...n ni i ib b b

+< < < , e, como

1 1

1,

2j j ji i i ijb b b b+ +

< ⇒ ≤ − para todo ,j n≤

temos 1 1

1 1 1( 1) ( 1) .

2 2 2 2ni i

nb b n n

+≤ − + ≤ − + ⋅ = −

Page 127: Eureka 2008

Sociedade Brasileira de Matemática

EUREKA! N°28, 2008

65

Assim, 1

.2ni

nb

+≤ −

Como {1,..., } , ,2N

NA N A a A∩ = = ≥ donde 0,

2N N

Nb a= − ≥ e a

desigualdade anterior garante que 1 .ni N n+ ≤ − Assim, se 1min{ ,..., },m Nb b b=

então .2m

nb ≤ − (pois

2N m

nb b− ≥ e 1

12j jb b+ − ≤ para todo j), o que garante

que m N n≤ − e 1,..., .m m m nb b b+ +≤

PROBLEMA 3: SOLUÇÃO DE FÁBIO DIAS MOREIRA (RIO DE JANEIRO – RJ)

Por Cauchy- Schwarz, temos que 22 21 1 1

0 0 0

.in n n

ii i

i i i

a p a p− − −

= = =

⋅ ≥

∑ ∑ ∑

Chame ;pα = então 22 1

20

1.

1

n ni

ii

a pαα

=

−≥

− ∑ Se p é raiz de p(x) e 1,p > então,

como 1

0

ni n

ii

a p p−

=

=−∑ , temos 2

2 2 2 2 2 2 22

11 (2 ) 1,

1

nn n n n nα

α α α α α αα

+−≥ ⇔ − ≥ − ⇔ − ≥

logo, como 0, 2.α α≥ <

Por outro lado, se 2 1 (0 1),α ε ε= + < < a desigualdade de Bernoulli diz que

2 1 ,n nα ε≥ + e portanto, para que 2 2 2 1(2 ) 1 ,

1n nα α α

ε− ≥ ⇔ ≥

− é suficiente

que ( ) ( )1 1 1nε ε+ − ≥ ⇔ ( ) 21 1 1n nε ε+ − − ≥ ⇔ 1

.n

−≤

Chame 1

1n

−= + e

2

2

1;

1nkλλ

−=

−defina p(x) tal que ,i

ia k λ= − ⋅ para

0 1.i n≤ ≤ −

É fácil verificar que 2

1.ia =∑ Ademais, 21

22

0

1 1( ) 0,

1

nnn i n n

i

p k kk

λλ λ λ λ λ

λ

=

−= − ⋅ = − ⋅ = − ≤

−∑ já que

Page 128: Eureka 2008

Sociedade Brasileira de Matemática

EUREKA! N°28, 2008

66

22 2 2

2

1 10 (2 ) 1,

1

nn n n

λ λ λ λλ

−− ≤ ⇔ ≤ ⇔ − ≥

− o que é verdadeiro pela

definição de λ (temos 2 1 ,λ ε= + com 1n

−= ). Logo ( )p x possui uma raiz

maior ou igual a λ (já que lim ( ) ).x

p x→+∞

= +∞

Assim, 1 1

( ) 1 liminf ( ) lim 1 2.n n

n ns n s n

n n→∞ →∞

− −≥ + ⇒ ≥ + = Mas 2α <

implica ( ) 2 limsup ( ) 2n

s n s n→∞

≤ ⇒ ≤ e, finalmente, lim ( ) 2.n

s n→+∞

=

PROBLEMA 4: SOLUÇÃO DE FÁBIO DIAS MOREIRA (RIO DE JANEIRO – RJ)

Note inicialmente que ( )( ( ( )) ( ).f x xf f f x e f e= = Fazendo as substituições

, , ,y z wx e y e z e= = = temos ( ) lim ( )( )

lim lim .y

f y f y ny

n nyx y

f x ee

x e→∞

→∞ →∞= =

( ) ( ) ( )( ) ( )lim ( ) lim lim .f z z z f z z

y z zf y ny e ne e e n−

→∞ →∞ →∞ − = − = −

Logo basta provar que ( )lim ( ) .z

f z z→∞

− = +∞

Mas ( ) ( )( )lim ( ) lim 1 ,w f w w

z wf z z e e −

→∞ →∞ − = − ou seja, basta que

( )f w w c≥ + para 0c > fixo e todo w suficientemente grande.

Como ( ) ( ) ( ( )) ( ( )) ,x yf x f y f f x f f y e e x y= ⇒ = ⇒ = ⇒ = f é injetora e, em virtude de ser contínua, é monótona. Se f fosse decrescente, como ] [0, Im( ),f+∞ ⊆ teríamos lim ( )

xf x

→−∞= +∞ mas,

por outro lado, lim lim ( ( )) 0,x

x xe f f x

→−∞ →−∞= = logo lim ( ) 0.

xf x

→+∞= Isso implica

] [Im( ) 0, ,f = +∞ mas então ] [ ] [( ) ] [( ( )) 0, 0, 0, (0) ,f f f f= +∞ = +∞ =¡

absurdo! Logo f é crescente. Se lim ( ) ,

xf x

→−∞= −∞ então lim ( ( ))

xf f x

→−∞= −∞ mas lim 0,x

xe

→−∞= logo

lim ( ) ,x

f x A→−∞

= onde 0A < é uma constante (veja que se 0A ≥ então

( ( )) (] ; [) ] ( ); [ ( ) 0 ( ) 0 ( ( )) (0),Af f f A f A f A f A e f f A f= +∞ = +∞ ⇒ = ⇒ = < = =¡absurdo).

Page 129: Eureka 2008

Sociedade Brasileira de Matemática

EUREKA! N°28, 2008

67

Defina ] ]0 ,I A= −∞ e 1 ( );n nI f I+ = assim ] ]1 ,0 ,I A=

2 30, , ,1 ...A AI e I e = = . É fácil ver que os ´iI s formam uma partição de ¡ ,

e logo ( ) , .f x x x> ∀ ∈¡

Considere ( ) ( ) .g x f x x= − No intervalo ] [, 2 ,A−∞ ( ) 2 .g x A A A> − = No

intervalo compacto ] [2 ,1 ,A a função g assume um mínimo positivo. Logo

] ]: ,1g −∞ → ¡ assume um mínimo positivo, digamos k.

Com isso provamos que ( )f x x k≥ + para 0 1 2 3 ;x I I I I∈ ∪ ∪ ∪ vamos

provar por indução que isso vale para 2 2 1k kx I I +∈ ∪ para todo 2 :k ≥

de fato, ( ( )) yx f f y e= = para 2 2 2 1k ky I I− −∈ ∪ e portanto ( )( ) ( ( ( ))) f y y kf x f f f y e e += = ≥ = (1 ) ,y k y ye e e k e k⋅ > ⋅ + > + já

que 0 1.yy e> ⇒ > Mas ( ( ))ye f f y x= = e portanto ( ) ,f x x k> + concluindo a demonstração.

PROBLEMA 5: SOLUÇÃO DA BANCA Primeiro afirmamos que o autoespaço V de A associado a um autovalor λ

{ | }V v Av vλ= = é invariante por B. Suponha .v V∈ Afirmamos que .Bv V∈

3( ) ( ) ( ) .A Bv B v B Av B v Bvλ λ= = = = Assim devemos em cada tal autoespaço definir B. Note que a definição de B em cada autoespaço é independente. (a) Se existir algum autovalor λ negativo então não existe B pois seus autovalores µ deveriam satisfazer 2 0 ,µ λ µ= < ⇒ ∉¡ o que contradiz a hipótese de B ser simétrica. Se existir autovalor zero ( 0)λ = com multiplicidade k então B restrita a V deve ser igual a 0 pois B é diagonalizável e todos os seus autovalores são iguais a 0. Se existir autovalor positivo ( 0)λ > com multiplicidade 1 então há duas

possibilidades correspondentes a µ λ= e .µ λ= − Se existir autovalor positivo ( 0)λ > com multiplicidade maior que 1 então há infinitas possibilidades pois podemos escolher de infinitas maneiras subespaços complementares para corresponderem a λ e .λ− Assim, o número pedido é:

Page 130: Eureka 2008

Sociedade Brasileira de Matemática

EUREKA! N°28, 2008

68

0, se algum for negativo.

2 , se nenhum for negativo, se todo positivo for simples e se houver autovalores positivos. , se nenhum for negativo e houver pelo menos um autovalor posi

k k

λ

λ λ

λ∞ tivo com multiplicidade maior que 1.

(b) Os casos 0λ > são como no item (a). Se existir autovalor zero com multiplicidade 1 então ainda há apenas uma opção. Por outro lado, se existir autovalor zero com multiplicidade maior do que 1 então há infinitas possibilidades para B (pois há infinitas matrizes B com 2 0)B = .

Se existir autovalor negativo com multiplicidade par (2k) então há infinitas

soluções: basta tomar ,B Jλ= ⋅ 2 .J I= − Há infinitas tais matrizes J pois

basta definir J em uma base 1 2 1 2, ,..., , ,...,k k kw w w w w+ por

, , 1,..., .j k j k j jJw w Jw w j k+ += = − =

Note que neste caso B tem autovalores iλ e iλ− com multiplicidades k e k.

Se existir autovalor negativo com multiplicidade ímpar então é impossível pois não há como os autovalores de B virem aos pares conjugados. Assim, o número pedido é:

0, se existe autovalor negativo com multiplicidade ímpar,

2 , se não existe autovalor negativo, todos os autovalores são simples, e há autovalores positivos. , se todos os autovalores nega

k

k∞ tivos têm multiplicidade par e existe pelo menos um autovalor com multiplicidade maior do que 1.

PROBLEMA 6: SOLUÇÃO DA BANCA

2 2 ax by+ pode ser visto como ( )( )x a y b x a y b⋅ + ⋅ − ⋅ − ⋅ − , o que

funciona como uma espécie de "norma" de .x a y b⋅ + ⋅ − Vamos usar o fato de que o produto ou a razão de dois números dessa forma são da forma

, z w ab+ ⋅ − e assim a raiz quadrada que aparece só depende do produto ab.

Page 131: Eureka 2008

Sociedade Brasileira de Matemática

EUREKA! N°28, 2008

69

Se 2 ,ab

rcd

= com *,r ∈¤2

2 2 2 ,z

cz dw c dwr

+ = +

% onde 2 ,c cr=% donde

2cd r cd ab= =% e ( , ) ( , ).S c d S c d=% Assim, se abcd

é o quadrado de um

racional, podemos supor sem perda de generalidade que ab = cd.

Se 2 2 2 2 ab

ax by tz wt

+ = +

é não nulo (se abcd

é quadrado, podemos

juntar esse quadrado com uma variável e supor ab = cd), e queremos provar que

qualquer número da forma 2 2abtu v

t +

também é da forma 2 2 ,ar bs+

escrevemos

( )2 2 2 2 2 2 2 2

2 2

2 2 2 2

.

ab ab abtz w tu v ax by tu v

t t tabtu v

t ab abtz w tz w

t t

+ ⋅ + + ⋅ + + = = + + Abusando de notação, isso é Acontece que

( ) ( )2 2 2 2

2 2

.

abab N x a y b N u t vax by tu vtt

ab abtz w N z t wt t

− ⋅ + ⋅ − ⋅ ⋅ + ⋅+ ⋅ + = −+ ⋅ + ⋅

2 2

( ),

ab vwabu t v uzt vz uw abt tabab tz wz t w tt

−⋅ + ⋅ + + − − = − +⋅ + ⋅

que, multiplicado por

x a y b⋅ + ⋅ − , dá um número da forma , k a b⋅ + ⋅ −l a saber

Page 132: Eureka 2008

Sociedade Brasileira de Matemática

EUREKA! N°28, 2008

70

( ) ( )

2 2

,

vwab vwabx uzt by vz uw a y uzt ax vz uw b

t tab

tz wt

+ − − + + + − − +

cuja "norma" 2 2 ,ak bl+ ou seja,

( ) ( )2 2

2 22 2 2 2

vwab vwabx uzt by vz uw y uzt ax vz uw

t ta b

ab abtz w tz w

t t

+ − − + + − + + +

é igual a 2 2 ,ab

tu vt

+

como queríamos (isso pode ser verificado diretamente,

mas chutar essa última expressão seria um pouco de sorte...). (Note que a condição de que há um valor comum não nulo das formas 2 2ax by+

e 2 2 cu dv+ é importante. Não é o caso, por exemplo, se a = 2, b = 3, c = 6 e d = 1).

Vamos agora mostrar a outra implicação. Queremos provar que se cdab

não é o

quadrado de um racional então as imagens de 2 2 ax by+ e de 2 2 cx dy+ (quando x e y percorrem os racionais) são diferentes. A imagem de

2 2 ax by+ com x e y racionais não muda se multiplicarmos a ou b pelo

quadrado de um racional não nulo. Assim, podemos supor que em 2 2 ax by+ e 2 2 cx dy+ temos a, b, c, d inteiros livres de quadrados. É claro que, se os sinais

de ab e cd forem diferentes, as imagens não podem ser iguais. Assim, nos casos

interessantes, 0.abcd

> Basta ver então que cada primo p divide um número par

de números dentre a, b, c, d para concluir que abcd

é o quadrado de um racional.

Se p divide exatamente 3 deles, digamos a, b e c, ou seja, a = pk, b = pl, c = pm,

temos que 2 2 2 2 ( )ax by p kx ly+ = + e 2

2 2 2 2 2 y

cx dy pmx dy p mx pdp

+ = + = +

Page 133: Eureka 2008

Sociedade Brasileira de Matemática

EUREKA! N°28, 2008

71

têm a mesma imagem se e só se 2 2 kx ly+ e 2 2 mz pdw+ têm a mesma imagem, e agora p divide exatamente um número dentre k, l, m e pd. Assim, reduzimos o problema a provar que, se um primo p divide exatamente um dentre os números a, b, c, d, digamos a, então 2 2 ax by+ e 2 2 cx dy+ não têm a mesma imagem. Suponhamos por absurdo que tenham. Notemos primeiro que para quaisquer u e v inteiros, existiriam x e y racionais com

2 2 2 2 = .ax by cu dv+ + Como |p a e a é livre de quadrados, a maior

potência de p que divide 2ax é ímpar e a maior potência de p que divide 2by é par, donde x e y não podem ter p no denominador, senão a maior potência de p que dividiria 2 2 ax by+ seria negativa e logo 2 2 2 2 = ax by cu dv+ + não poderia ser inteiro. Assim, x e y podem ser vistos como inteiros módulo p, e

2 2 2 (mod ),cu dv by p+ ≡ donde 2 2 2 ( ) (mod ) bcu bdv by p+ ≡ é quadrado mod p para quaisquer u, v inteiros. Fazendo u = 1, v = 0 temos que bc é quadrado mod p. Fazendo v = 1, temos que 2 bcu bd+ é quadrado mod p para todo u inteiro, donde, como bc é quadrado mod p, u2 + d/c é quadrado mod p para todo u inteiro (note que b, c e d são invertíveis mod p). Ou seja, se r é quadrado

mod p então d

rc

+ também é, mas isso implica por indução que kd

rc

+ é

quadrado mod p para todo k natural, donde todo inteiro é quadrado mod p. Isso só é possível se p = 2. Se p = 2 dividir um número par de números dentre a, b, c, d teremos que todo primo p divide um número par de números dentre a, b, c, d, como queríamos. Caso contrário, teremos ainda algum trabalho extra, que realizaremos a seguir. Podemos supor como antes que 2 divide a mas não divide bcd. Sejam

( , )r mdc c d= e K o produto dos primos ímpares que dividem ab mas não

dividem cd. Sejam e .c d

m nr r

= = Se n (que, como d, é ímpar) for

congruente a 3 ou –3 módulo 8, tomaremos 2 (4 ) , R rK c d= ⋅ + e se n for

congruente a 1 ou –1 módulo 8, tomaremos 2 (2 ) . R rK c d= ⋅ + Temos em

qualquer caso que R pertence à imagem de 2 2 .cx dy+ Além disso, Rr

é

congruente a 3 ou –3 módulo 8 e é primo com m, n, r e K, e portanto é primo com

a, b, c e d. Rr

tem que ter algum fator primo q congruente a 3 ou –3 módulo 8

que aparece com expoente ímpar em sua fatoração (pois um produto de números

Page 134: Eureka 2008

Sociedade Brasileira de Matemática

EUREKA! N°28, 2008

72

que são 1 ou –1 módulo 8 ainda é dessa forma). Temos que R é um número da forma 2 c x d⋅ + com x inteiro, e portanto 2 0(mod ), c x d R q⋅ + = =

donde 2 ( ) (mod ).cd cx q− = Por outro lado, se as imagens são iguais, existem

u e v racionais com 2 2 .au bv R+ = Podemos escrever U

uD

= e V

vD

=

onde D é o menor denominador comum de u e v. Temos então 2 2 2 0(mod ).aU bV R D q+ = ⋅ = Se jq é a maior potência de q que divide

U e V simultaneamente, escrevemos jU q T= ⋅ e jV q S= ⋅ obtendo então

2 2 2 2 ,jaT bS R q D−+ = ⋅ ⋅ que ainda é múltiplo de q. Como , R

q qr

é primo

com a e b, e logo q não pode dividir T, caso contrário q dividiria bS2, donde q dividiria também S, contradizendo a escolha de j. Assim,

2 2 ( ) (mod ) ab T bS q− ⋅ = implica que –ab é um quadrado módulo q.

Portanto, ( )

( )

ab abcd cd

−=

− também é um quadrado módulo q, mas, pelas

considerações anteriores, 2 2 ,ab

wcd

= ⋅ para algum racional w, e daí seguiria

que 2 é quadrado módulo q, o que é um absurdo, pois q é congruente a 3 ou –3 módulo 8 (veja o artigo “ Reciprocidade quadrática”, de Carlos Gustavo Moreira e Nicolau Saldanha, na Eureka! No. 15). Errata: Na Eureka! No. 27, no artigo “Substituições envolvendo números complexos”, de Diego Veloso Uchoa, na página 21, o trecho entre as linhas 12 e 15 deveria ser: Fazendo 2 1n m= + e igualando as partes imaginárias, temos:

2 2 12 1

2 1 2 1((2 1) )(cos ) (cos ) ... ( 1)

1 3m m m

m

m msen m tt t

sen t−

+

+ + += − + + −

. (*)

Agora podemos tratar essa igualdade por meio do polinômio

12 1 2 1( ) ... ( 1) .

1 3m m m

m

m mP x x x −+ +

= − + + −

Page 135: Eureka 2008

Sociedade Brasileira de Matemática

EUREKA! N°28, 2008

73

XXIX Olimpíada Brasileira de Matemática Nível 1 (5ª. e 6ª. Séries)

Nome Cidade – Estado Prêmio Rafael Kazuhiro Miyazaki São Paulo – SP Ouro Guilherme Renato Martins Unzer São Paulo – SP Ouro Marina Pessoa Mota Fortaleza – CE Ouro Ivan Tadeu Ferreira Antunes Filho Lins – SP Ouro Danilo Hikari Motoyama Watanabe São Paulo – SP Ouro

Arthur Oenning Fagundes Palmas – TO Prata Breno Levi Correa Campo Belo – MG Prata Thomás Rincon Reis Belo Horizonte – MG Prata Lucas Finger Roman Florianópolis – SC Prata Lucas Nishida Pedreira – SP Prata Ana Cristina Barreto Sabino de Araújo Itapissuma – PE Prata Gabriel Santa Rosa Cavaresi Birigüi – SP Prata Victor Kioshi Higa São Paulo – SP Prata Ana Beatrice Bonganha Zanon Santo André – SP Prata Maria Paula Silva Serrasqueiro Brasília – DF Prata

Débora Barreto Ornellas Salvador – BA Bronze

Igor Araújo Rio de Janeiro – RJ Bronze Pedro Ivo Coelho de Araújo Caucaia – CE Bronze Ramon Silva de Lima São Paulo – SP Bronze Dênnis Dantas de Souza Campina Grande – PB Bronze Nathália Roscoe e Firace Belo Horizonte – MG Bronze Renan Fernandes Moreira Taubaté – SP Bronze Nicolas Seoane Miquelin Mauá – SP Bronze Nicolas Fernandez Leitão Florianópolis – SC Bronze Tiago Sueda Limone Jundiaí – SP Bronze Gabriel Pacianotto Gouveia São Paulo – SP Bronze Jonathan Henrique de Oliveira Cordeirópolis – SP Bronze Murilo Dória Guimarães São Paulo – SP Bronze Julio Barros de Paula Taubaté – SP Bronze Cesar Nobuo Moniwa Ishiuchi Campinas – SP Bronze Danilo Kenji Shido São Paulo – SP Menção Honrosa Francisco Markan Nobre de Souza Filho Fortaleza – CE Menção Honrosa Natália Rodrigues Parrode Goiânia – GO Menção Honrosa João Felipe Ribeiro Soares Brasília – DF Menção Honrosa Sofia Sayuri Yamamura Araçatuba – SP Menção Honrosa Paula Dias Garcia Brasília – DF Menção Honrosa Lara Timbó Araújo Fortaleza – CE Menção Honrosa Nathalia Novello Fernandes Ribeiro Rio de Janeiro – RJ Menção Honrosa Eric Luiz Rodrigues de França Recife – PE Menção Honrosa Pedro Ducci Serafim Campinas – SP Menção Honrosa Lucas Bitran Giestas Vitória – ES Menção Honrosa

Wederson Santos Silva Massaranduba – PB Menção Honrosa

Ayrton Barros de Lira Recife – PE Menção Honrosa Leonardo Kazunori Tsuji São Paulo – SP Menção Honrosa Lucas Guedes de Almeida Rocha Maceió – AL Menção Honrosa Liang Wei Dong Salvador – BA Menção Honrosa Rafael Wingester Ribeiro de Oliveira Belo Horizonte – MG Menção Honrosa Hugo Diehl de Souza Criciúma – SC Menção Honrosa Matheus de Oliveira Leão Teresina – PI Menção Honrosa Rodolfo Vieira Fontenele Cocal dos Alves – PI Menção Honrosa Henrique Gasparini Fiúza do Nascimento Brasília – DF Menção Honrosa Victor Venturi Campinas – SP Menção Honrosa Gabrielle Macanhan Guimarães Anápolis – GO Menção Honrosa Reinaldo Abad Junior Guarulhos – SP Menção Honrosa Henrique Vieira G. Vaz São Paulo – SP Menção Honrosa Gabriela Loiola Vilar Fortaleza – CE Menção Honrosa Igor Tetsuo Boninsenha Kunizaki Taubaté – SP Menção Honrosa Marcelo Cargnelutti Rossato Santa Maria – RS Menção Honrosa

Arthur Ferreira do Nascimento São Paulo – SP Menção Honrosa Liara Guinsberg São Paulo – SP Menção Honrosa Filipe Bellio da Nóbrega Rio de Janeiro – RJ Menção Honrosa Israel Rodrigues Soares Goiânia – GO Menção Honrosa Matheus Carneiro Campagnani Niterói – RJ Menção Honrosa

Page 136: Eureka 2008

Sociedade Brasileira de Matemática

EUREKA! N°28, 2008

74

Nível 2 (7a. e 8a. Séries)

Nome Cidade – Estado Prêmio João Mendes Vasconcelos Fortaleza – CE Ouro Matheus Barros de Paula Taubaté – SP Ouro Gabriel Militão Vinhas Lopes Fortaleza – CE Ouro Thiago Saksanian Hallak São Paulo – SP Ouro Paulo Henrique Dias Vieira Rio de Janeiro – RJ Ouro

João Lucas Camelo Sá Fortaleza – CE Prata Ana Beatriz Prudêncio de Almeida Rebouças Fortaleza – CE Prata Hanon Guy Lima Rossi São Paulo – SP Prata Danilo Silva de Albuquerque Fortaleza – CE Prata Felipe Vieira de Paula Fortaleza – CE Prata Leonardo Ferreira Patrício Rio de Janeiro – RJ Prata Deborah Barbosa Alves São Paulo – SP Prata Vinicius Cipriano Klein Venda Nova do Imigrante – ES Prata Fernando Fonseca Andrade Oliveira Belo Horizonte – MG Prata Maria Clara Mendes Silva Pirajuba – MG Prata Ruan Alves Pires Rio de Janeiro – RJ Bronze Natan Lima Viana Fortaleza – CE Bronze Gleycianne Arruda de Freitas Silva Fortaleza – CE Bronze Matheus Secco Torres da Silva Rio de Janeiro – RJ Bronze Felipe Mostavenco Carmo Rio de Janeiro – RJ Bronze Jonas Rocha Lima Amaro Fortaleza – CE Bronze Gustavo Lisbôa Empinotti Florianópolis – SC Bronze Guilherme da Rocha Dahrug Santo André – SP Bronze Victorio Takahashi Chu São Paulo – SP Bronze Itamar Sales de Oliveira Filho Cedro – CE Bronze Francisco Vagner Dantas Leite Filho Fortaleza – CE Bronze Kayo de França Gurgel Fortaleza – CE Bronze Rodrigo Rolim Mendes de Alencar Fortaleza – CE Bronze Igor Rosiello Zenker São Paulo – SP Bronze Daniel Lucas Filgueira Fortaleza – CE Bronze Mario Valney Pereira de Andrades Fortaleza – CE Bronze Rafael Dias da Fonseca Maceió – AL Bronze Matheus Cordeiro Wilhelm da Costa Rio de Janeiro – RJ Menção Honrosa Elder Massahiro Yoshida São Paulo – SP Menção Honrosa Lucas de Freitas Smaira Guaxupé – MG Menção Honrosa Léo Nunes Benevides Fortaleza – CE Menção Honrosa Felipe Bento Vargas de Moraes Rio de Janeiro – RJ Menção Honrosa Rubens Cainan Sabóia Monteiro Fortaleza – CE Menção Honrosa Alessandro Macêdo de Araújo Fortaleza – CE Menção Honrosa Sandoel de Brito Vieira Cocal dos Alves – PI Menção Honrosa Rafael Ferreira Antonioli S. B. do Campo – SP Menção Honrosa Leonardo Victor Maciel Pontes Fortaleza – CE Menção Honrosa Bryan Levy Salinas Carrillo São Paulo – SP Menção Honrosa Filipe José Oliveira Sabóia Fortaleza – CE Menção Honrosa Débora Jun Portugheis Campinas – SP Menção Honrosa Kelve Torres Henrique Recife – PE Menção Honrosa Nicolás Francisco E. C. Hespanhol Santos Bauru – SP Menção Honrosa André Austregesilo Scussel Fortaleza – CE Menção Honrosa Álvaro Lopes Pedroso Santa Isabel – SP Menção Honrosa Wellington Biing Jung Lee São Paulo – SP Menção Honrosa Bruno César da Silva Guedes Recife – PE Menção Honrosa Luiz Filipe Martins Ramos Niterói – RJ Menção Honrosa Jéssica Kazumi Okuma São Paulo – SP Menção Honrosa Pedro Vieira Rodrigues Serradas Rio de Janeiro – RJ Menção Honrosa Leonardo Henrique Caldeira Pires Ferrari Rio de Janeiro – RJ Menção Honrosa Carlos Henrique de Andrade Silva Fortaleza – CE Menção Honrosa Gregory Cosac Daher Rio de Janeiro – RJ Menção Honrosa

Page 137: Eureka 2008

Sociedade Brasileira de Matemática

EUREKA! N°28, 2008

75

Nível 3 (Ensino Médio)

Nome Cidade – Estado Prêmio Rafael Tupynambá Dutra Belo Horizonte – MG Ouro Régis Prado Barbosa Fortaleza – CE Ouro Ramon Moreira Nunes Fortaleza – CE Ouro Henrique Pondé de Oliveira Pinto Salvador – BA Ouro Henrique Hiroshi Motoyama Watanabe São Paulo – SP Ouro Adenilson Arcanjo de Moura Junior Fortaleza – CE Prata Renan Henrique Finder São Paulo – SP Prata Guilherme Philippe Figueiredo São Paulo – SP Prata Marco Antonio Lopes Pedroso Santa Isabel – SP Prata Rafael Sampaio de Rezende Fortaleza – CE Prata Giuliano Pezzolo Giacaglia Santo André – SP Prata Jorge Henrique Craveiro de Andrade Rio de Janeiro – RJ Prata Marcelo Matheus Gauy S.J. do Rio Preto – SP Prata Mateus Oliveira de Figueiredo Fortaleza – CE Prata Paulo Sérgio de Castro Moreira Fortaleza – CE Prata Robério Soares Nunes Ribeirão Preto – SP Prata Marlen Lincoln da Silva Fortaleza – CE Bronze Esdras Muniz Mota Fortaleza – CE Bronze Grazielly Muniz da Cunha Fortaleza – CE Bronze Davi Lopes Alves de Medeiros Fortaleza – CE Bronze Gabriel Luís Mello Dalalio S. J. dos Campos – SP Bronze José Airton Coêlho Lima Filho Fortaleza – CE Bronze Leandro Farias Maia Fortaleza – CE Bronze Marcos Victor Pereira Vieira Fortaleza – CE Bronze Alfredo Roque de Oliveira Freire Filho S. J. dos Campos – SP Bronze Francisco Osman Pontes Neto Fortaleza – CE Bronze Leonel Lopes Lima Neto Maceió – AL Bronze Renan Braz Parente Fortaleza – CE Bronze Alex Atsushi Takeda Londrina – PR Bronze Marcelo Tadeu de Sá Oliveira Sales Salvador – BA Bronze Thiago Ribeiro Ramos Varginha – MG Bronze Luiz Paulo Freire Moreira Fortaleza – CE Menção Honrosa Antônio Felipe Cavalcante Carvalho Fortaleza – CE Menção Honrosa Luca Mattos Möller Niterói – RJ Menção Honrosa Hugo Fonseca Araújo Juiz de Fora – MG Menção Honrosa Fernando Nascimento Coelho Fortaleza – CE Menção Honrosa Filipe de Almeida Araujo Vital Rio de Janeiro – RJ Menção Honrosa Illan Feiman Halpern Itatiaia – RJ Menção Honrosa Alexandre Nobuo Kunieda São Paulo – SP Menção Honrosa Alysson Espíndola de Sá Silveira Fortaleza – CE Menção Honrosa Thiago S. Pinheiro São Paulo – SP Menção Honrosa Orlando Alencar Lustosa Neto Fortaleza – CE Menção Honrosa Ricardo Turolla Bortolotti Rio Claro – SP Menção Honrosa Gustavo Pacianotto Gouveia São Paulo – SP Menção Honrosa Felipe Holanda Moreira Fortaleza – CE Menção Honrosa Artur de Almeida Losnak São Paulo – SP Menção Honrosa Rafael Parpinel Cavina São Paulo – SP Menção Honrosa Filipe Alves Tomé Fortaleza – CE Menção Honrosa Custodio Moreira Brasileiro Silva Caém – BA Menção Honrosa Marília Valeska Costa Medeiros Fortaleza – CE Menção Honrosa Pollyanna Stéfani Borges Freitas Fortaleza – CE Menção Honrosa Gustavo Sampaio Sousa Fortaleza – CE Menção Honrosa Joas Elias dos Santos Rocha Muribeca – SE Menção Honrosa Raphael Luiz França Greco Rio de Janeiro – RJ Menção Honrosa Rafael Morioka Oda São Paulo – SP Menção Honrosa

Page 138: Eureka 2008

Sociedade Brasileira de Matemática

EUREKA! N°28, 2008

76

Nível Universitário

Nome Cidade – Estado Prêmio Fábio Dias Moreira Rio de Janeiro – RJ Ouro Rafael Marini Silva Vila Velha – ES Ouro Guilherme Rodrigues Nogueira de Souza São Paulo – SP Ouro José Marcos Andrade Ferraro São Paulo – SP Ouro Rafael Daigo Hirama Campinas – SP Ouro Eduardo de Moraes Rodrigues Poço São Paulo – SP Prata Felipe Rodrigues Nogueira de Souza São Paulo – SP Prata Murilo Vasconcelos Andrade Maceió – AL Prata Leonardo Ribeiro de Castro Carvalho São Paulo – SP Prata Luty Rodrigues Ribeiro S.J. dos Campos – SP Prata André Linhares Rodrigues Campinas – SP Prata Maurício de Lemos Rodrigues Collares Neto Aracaju – SE Prata Henry Wei Cheng Hsu São Paulo – SP Prata Kellem Correa Santos Rio de Janeiro – RJ Prata Levi Maximo Viana Rio de Janeiro – RJ Bronze Ronaldo Rodrigues Pelá São Carlos – SP Bronze Luís Daniel Barbosa Coelho Rio de Janeiro – RJ Bronze Thiago Costa Leite Santos São Paulo – SP Bronze Helder Toshiro Suzuki São Paulo – SP Bronze Raphael Constant da Costa Rio de Janeiro – RJ Bronze Rafael Sabino Lima Rio de Janeiro – RJ Bronze Erick Costa e Silva Talarico Rio de Janeiro – RJ Bronze Rodrigo Aguiar Pinheiro S.J. dos Campos – SP Bronze Renato Rebouças de Medeiros S.J. dos Campos – SP Bronze José Armando Barbosa Filho S.J. dos Campos – SP Bronze Evandro Makiyama de Melo São Paulo – SP Bronze Tiago Barbin Batalhão São Carlos – SP Bronze Gabriel Ponce São Carlos – SP Bronze Vitor Gabriel Kleine S.J. dos Campos – SP Bronze Alexandre Hideki Deguchi Martani São Paulo – SP Bronze Vitor Humia Fontoura Rio de Janeiro – RJ Bronze Ana Maria Menezes de Jesus Itabaiana – SE Bronze Eduardo Fischer Encantado – RS Bronze Anderson Hoshiko Aiziro São Paulo – SP Bronze Daniel Lopes Alves de Medeiros S.J. dos Campos – SP M. Honrosa Paulo André Carvalho de Melo Rio de Janeiro – RJ M. Honrosa Pedro Meira de Vasconcellos Bezerra Recife – PE M. Honrosa Willy George do Amaral Petrenko Rio de Janeiro – RJ M. Honrosa Gustavo Antônio da Silva Amaro São Carlos – SP M. Honrosa Ricardo Monteiro da Silva Lanna Belo Horizonte – MG M. Honrosa Felipe Gonçalves Assis Campina Grande – PB M. Honrosa Elder Rodrigo Barbosa Campos Rio de Janeiro – RJ M. Honrosa Matheus Pimentel Rodrigues Rio de Janeiro – RJ M. Honrosa Rafael Montezuma Pinheiro Cabral Fortaleza – CE M. Honrosa Nivan Roberto Ferreira Júnior Olinda – PE M. Honrosa Elton Gomes Coriolano Campinas – SP M. Honrosa Thomás Yoiti Sasaki Hoshina Rio de Janeiro – RJ M. Honrosa Samir Rodrigues Vieira Fortaleza – CE M. Honrosa Frederico de Souza Frydman S.J. dos Campos – SP M. Honrosa Jordan Freitas Piva Rio de Janeiro – RJ M. Honrosa Rodrigo Viana Soares Fortaleza – CE M. Honrosa Bruno Euzébio dos Santos Malhados – SE M. Honrosa Antonia Taline de Souza Mendonça Rio de Janeiro – RJ M. Honrosa

Page 139: Eureka 2008

Sociedade Brasileira de Matemática

EUREKA! N°28, 2008

77

AGENDA OLÍMPICA

XXX OLIMPÍADA BRASILEIRA DE MATEMÁTICA

NÍVEIS 1, 2 e 3 Primeira Fase – Sábado, 14 de junho de 2008

Segunda Fase – Sábado, 13 de setembro de 2008 Terceira Fase – Sábado, 25 de outubro de 2007 (níveis 1, 2 e 3)

Domingo, 26 de outubro de 2008 (níveis 2 e 3 - segundo dia de prova).

NÍVEL UNIVERSITÁRIO Primeira Fase – Sábado, 13 de setembro de 2008

Segunda Fase – Sábado, 25 e Domingo, 26 de outubro de 2008

XIV OLIMPÍADA DE MAIO 10 de maio de 2008

XIX OLIMPÍADA DE MATEMÁTICA DO CONE SUL Temuco – Chile

18 a 23 de junho de 2008

XLIX OLIMPÍADA INTERNACIONAL DE MATEMÁTICA 10 a 22 de julho de 2008

Madri – Espanha

XIV OLIMPÍADA INTERNACIONAL DE MATEMÁTICA UNIVERSITÁRIA 25 a 31 de julho de 2008

Blagoevgrad, Bulgária

XXIII OLIMPÍADA IBEROAMERICANA DE MATEMÁTICA 18 a 28 de setembro de 2008

Salvador, Bahia – Brasil ♦

XI OLIMPÍADA IBEROAMERICANA DE MATEMÁTICA UNIVERSITÁRIA

Page 140: Eureka 2008

Sociedade Brasileira de Matemática

EUREKA! N°28, 2008

78

COORDENADORES REGIONAIS

Alberto Hassen Raad (UFJF) Juiz de Fora – MG Américo López Gálvez (USP) Ribeirão Preto – SP Amarísio da Silva Araújo (UFV) Viçosa – MG Andreia Goldani FACOS Osório – RS Antonio Carlos Nogueira (UFU) Uberlândia – MG Ali Tahzibi (USP) São Carlos – SP Benedito Tadeu Vasconcelos Freire (UFRN) Natal – RN Carlos Alexandre Ribeiro Martins (Univ. Tec. Fed. de Paraná) Pato Branco – PR Carmen Vieira Mathias (UNIFRA) Santa María – RS Claus Haetinger (UNIVATES) Lajeado – RS Cleonor Crescêncio das Neves (Inst. de Tec. e Educ. Galileo da Amazônia) Manaus – AM Cláudio de Lima Vidal (UNESP) S.J. do Rio Preto – SP Denice Fontana Nisxota Menegais (UNIPAMPA) Bagé – RS Edson Roberto Abe (Colégio Objetivo de Campinas) Campinas – SP Élio Mega (Faculdade Etapa) São Paulo – SP Eudes Antonio da Costa (Univ. Federal do Tocantins) Arraias – TO Fábio Brochero Martínez (UFMG) Belo Horizonte – MG Florêncio Ferreira Guimarães Filho (UFES) Vitória – ES Francinildo Nobre Ferreira (UFSJ) São João del Rei – MG Genildo Alves Marinho (Centro Educacional Leonardo Da Vinci) Taguatingua – DF Ivanilde Fernandes Saad (UC. Dom Bosco) Campo Grande – MS Jacqueline Rojas Arancibia (UFPB)) João Pessoa – PB Janice T. Reichert (UNOCHAPECÓ) Chapecó – SC João Benício de Melo Neto (UFPI) Teresina – PI João Francisco Melo Libonati (Grupo Educacional Ideal) Belém – PA Jose de Arimatéia Fernandes (UFPB) Campina Grande – PB José Luiz Rosas Pinho (UFSC) Florianópolis – SC José Vieira Alves (UFPB) Campina Grande – PB José William Costa (Instituto Pueri Domus) Santo André – SP Krerley Oliveira (UFAL) Maceió – AL Licio Hernandes Bezerra (UFSC) Florianópolis – SC Luciano G. Monteiro de Castro (Sistema Elite de Ensino) Rio de Janeiro – RJ Luzinalva Miranda de Amorim (UFBA) Salvador – BA Mário Rocha Retamoso (UFRG) Rio Grande – RS Marcelo Rufino de Oliveira (Grupo Educacional Ideal) Belém – PA Marcelo Mendes (Colégio Farias Brito, Pré-vestibular) Fortaleza – CE Newman Simões (Cursinho CLQ Objetivo) Piracicaba – SP Nivaldo Costa Muniz (UFMA) São Luis – MA Osnel Broche Cristo (UFLA) Lavras – MG Osvaldo Germano do Rocio (U. Estadual de Maringá) Maringá – PR Raul Cintra de Negreiros Ribeiro (Colégio Anglo) Atibaia – SP Ronaldo Alves Garcia (UFGO) Goiânia – GO Rogério da Silva Ignácio (Col. Aplic. da UFPE) Recife – PE Reginaldo de Lima Pereira (Escola Técnica Federal de Roraima) Boa Vista – RR Reinaldo Gen Ichiro Arakaki (UNIFESP) SJ dos Campos – SP Ricardo Amorim (Centro Educacional Logos) Nova Iguaçu – RJ Sérgio Cláudio Ramos (IM-UFRGS) Porto Alegre – RS Seme Gebara Neto (UFMG) Belo Horizonte – MG Tadeu Ferreira Gomes (UEBA) Juazeiro – BA Tomás Menéndez Rodrigues (U. Federal de Rondônia) Porto Velho – RO Valdenberg Araújo da Silva (U. Federal de Sergipe) São Cristovão – SE Vânia Cristina Silva Rodrigues (U. Metodista de SP) S.B. do Campo – SP Wagner Pereira Lopes (CEFET – GO) Jataí – GO